Category: PRELIMS

IAS Score

Civil Service UPSC as a Career

  • by IAS Score

Civil Service (UPSC) as a Career

George Lucas said that dreams are extremely important. You can’t do it unless you imagine it. Dreams keep you alive, make you different from the crowd; they invigorate energy, warmth and freshness and consciousness in you that keep you restless unless and until you don’t finish the post you aimed at.

The primary reason most aspirants choose Civil Services as a career option is:

 ·         Job security, perks, and retirement benefits.

·         Power, Prestige and Authority to frame policies that govern daily life or businesses.

·         Reputation and ability to reach out to the corridors of power to move matters.

·         A chance to develop and usher the country on a path of progress and bring about a systemic change for the betterment of future generations.

·         Unbridled access to knowledge and future education by way of Government policies.

Challenges

·         The path to becoming a civil services officer is strewn with immense hard work and sheer dedication.

·         Salaries may not be as high as compared to other professions.

·         Inter-job transfers may be difficult and sometimes inevitable, so it is like a double-edged sword.

Civil services as a career haveremained the beatings of hearts of thousands of youngmen and women across the country for a multitude of reasons. For some it is a passion to make it to the elite service to touch million lives and to some it offers job diversity and prestige. For some, it is a pompous show of power; and for the others, it is an instrument to bring in equity and equality by acting as champions for the underprivileged groups. The Indian Civil Services have remained one of the premier services which provide prestige, job security, higher salary package, opportunity of foreign tour, job satisfaction, etc., that attract talent as well motivation for the aspiring civil servants.

Civil servants, in away, decide the fate of the nation as the implementation of all developmental and other government policy rest with them.

  • The technical and advisory aids provided by civil servants in the policy-making are indispensable components in the making of policies.
  • The decisions of civil servants have deep ramifications as there is a diversity of work associated with civil services, like maintenance of law and order, developmental work, disaster management, representing India on international forum, administration, upliftment and empowerment of marginalised sections of society, etc.
  • Who would not like to get such a dream job that involves dynamism, challenges, teachings and building the nation through and through?
  • Johnny Carson well said, “Not education. Not being born with hidden talents. But desire is the one secret of every man’s career”.
  • No doubt desires and dreams are personal and innate but a proper shape to them requires coaching and guidance. GS SCORE apart from conventional and digital coaching has been helping students in designing their dreams as well as achieving them.

Career Prospects

The are many career prospects in civil services. A candidate who clears UPSC or the state level civil services examination and has completed his/her training is commissioned as an officer. In the Central Services, the first posting is usually as a Diplomatic attaché/ Under-secretary/ Probationary IPS officer, depending on the branch chosen. In the state civil services, it is referred to as Deputy Collector/Deputy Superintendent of Police/ Taxes Commissioner, etc.

Different Branches Names of posts Responsibilities
Indian Foreign Service Diplomat

Foreign Secretary

High Commissioner

The framing of foreign policies

Maintaining and regulating the Indian embassies in various countries of the world

Indian Administrative Service District Magistrate (collector)

Principal Secretary

Chief Secretary

responsible for the smooth functioning of administrative departments

framing of policies and acting as advisors to the ministers

executing appointed duties as the officers in charge of smaller administrative units

Indian Police Service Border Security Force

Department of Home Guards

Central Bureau of Investigation

Maintain law and order in our country

 

Proper planningis required to convert dreams into reality as goals without dreams are mere wishes.

Through hard work and determination, it is possible to achieve your goal. Experienced guidance and knowledge is available for all students at GS Score for converting all your dreams and potentialities into realities.

IAS Score

UPSC Prelims Answer Key 2017 with Explanation

  • by IAS Score

UPSC Prelims Answer Key 2017

Here we are providing the UPSC Prelims answer key 2017 of Set B with detailed explanation for the aspirants. This will help you in tallying the answers to know their scores and start preparation for Mains freely.

All The Best to our Prelims in 100 Days Program subscribers. 46+ answers (still counting is going on) out of 100 questions were from your program. Check the answers with the link of your program given with the explanation….

Download Answer Key PDF

Q 1. The object of the Butler Committee of 1927 was to
(a) Define the jurisdiction of the Central and Provincial Governments.
(b) Define the powers of the Secretary of State for India.
(c) Impose Censorship on national press.
(d) Improve the relationship between the Government of India and the Indian States.
Ans. D
Exp:  The Indian States committee appointed a committee under the chairmanship of Sir Harcourt Butler which was popularly known as the Butler Committee to investigate and clarify the relationship between the paramount power and Princes of Princely States in 1927 AD. Committee visited 16 states and submitted its report in 1929.

Q 2. The term ‘Domestic Content Requirement’ is sometimes seen in the news with reference to
(a) Developing solar power production in our country.
(b) Granting licenses to foreign T.V. channels in our country.
(c) Exporting our food products to other countries.
(d) Permitting foreign educational institutions to set up their campuses in our country.
Ans. A
Exp:  The policy of Domestic Content Requirement is with intent to promote the local manufacturing of the components of solar generation equipment which includes the cells and modules.
• With an objective to establish India as a global leader in solar energy , the Ministry of New and Renewable Energy Launched its National Solar Policy in 2010 named as Jawaharlal Nehru National Solar Mission (JNNSM). Phase 1 of the National policy was composed of 2 parts.
• Batch 1 of policy implementation: India required developers of solar photovoltaic projects employing crystalline silicon technology to use solar modules manufacture in India
• Batch 2 India expanded this domestic sourcing requirement to crystalline silicon solar cells as well.

Q 3. Consider the following statements:
1. The Nuclear Security Summits are periodically held under the aegis of the United Nations.
2. The International Panel on Fissile Materials is an organ of International Atomic Energy Agency.
Which of the statements given above is/are correct?
(a) 1 Only
(b) 2 Only
(c) Both 1 and 2
(d) Neither 1 nor 2
Ans. D
Exp:  The idea of Nuclear Security Summit (NSS) was mooted by the US President Barack Obama in 2009 and the first summit was held in 2010 in Washington. It is a signature event of global diplomacy in Obama’s presidency that aimed at preventing nuclear terrorism around the globe. India has been contributing its bit to strengthen nuclear security by taking an active part in the summit ever since its inception in 2010.
• The fourth edition of Nuclear Security Summit (NSS) began in Washington, United States to discuss a gamut of issues pertaining to nuclear security
• India is being represented by Prime Minister Narendra Modi in the two-day summit. The fourth summit is the last summit under the current format and also under the presidency of Barack Obama.

Source: http://iasscore.in/upsc-prelims/nuclear-security-summit

Q 4. Who among the following can join the National Pension System (NPS)?
(a) Resident Indian citizens only
(b) Persons of age from 21 to 55 only
(c) All State Government employees joining the services after the date of notification by the respective State Governments
(d) All Central Governments Employees including those of Armed Forces joining the services on or after 1st April, 2004
Ans. C
Exp:  The Central Government had introduced the National Pension System (NPS) with effect from January 1, 2004 (except for armed forces).
• Subsequently, various State Governments adopted this architecture and implemented NPS with effect from different dates.
• Eligibility – Any citizen of India, whether resident or non-resident, subject to the following conditions. Individuals who are aged between 18 – 60 years, including NRIs . After attaining 60 years of age, you will not be permitted to make further contributions to the NPS accounts.

Q 5.With reference to river Teesta, consider the following statements:
1. The source of river Teesta is the same as that of Brahmaputra but it flows through Sikkim.
2. River Rangeet originates in Sikkim and it is a tributary of river Teesta.
3. River Teesta flows into Bay of Bengal on the border of India and Bangladesh.
Which of the statements given above is/are correct?
(a) 1 and 3 only
(b) 2 only
(c) 2 and 3 only
(d) 1, 2 and 3
Ans B
Exp:  The Teesta River originates from the Pahunri glacier above 7,068 metres and flows southward through gorges and rapids in the Sikkim Himalaya. Pauhunri is a mountain in the Eastern Himalayas located on the border of Sikkim, India and Tibet, China, and is situated about 75 km northeast of Kangchenjunga.
 Tista River is a tributary of the Jamuna River (Brahmaputra), flowing through India and Bangladesh. Brahmaputra finally merges into Bay of Bengal
 The Rangeet is a tributary of the Teesta river, which is the largest river in the Indian state of Sikkim. The Rangeet river originates in the Himalayan mountains in West Sikkim district. The river also forms the boundary between Sikkim and Darjeeling district.

Q 6. Consider the following statements:
1. In tropical regions, Zika virus disease is transmitted by the same mosquito that transmits dengue.
2. Sexual transmission of Zika virus disease is possible.
Which of the statements given above is/are correct?
(a) 1 only
(b) 2 only
(c) Both 1 and 2
(d) Neither 1 nor 2
Ans. C
Exp: Dengue and Zika are both mosquito borne viruses spread especially by the Aedes Aegypti mosquito species.
Evidence has emerged that the Zika virus can actually be transmitted through sex and blood transfusion. This is an element that has also created some difference between the two even though the sexual transmissions are still on the low.
Apart from exercising mosquito related preventions, people need to exercise safer sex to reduce Zika infections. 

Source: http://iasscore.in/upsc-prelims/diseases-types-of-vaccines-latest-updates

Q 7. Consider the following statements:
1. The Standard Mark of Bureau of Indian Standards (BIS) is mandatory for automotive tyres and tubes.
2. AGMARK is a quality Certification Mark issued by the Food and Agriculture Organisation (FAO).
Which of the statements given above is/are correct?
(a) 1 only
(b) 2 only
(c) Both 1 and 2
(d) Neither 1 nor 2
Ans. A
Exp:  The Product Certification Scheme of BIS aims at providing Third Party Guarantee of quality, safety and reliability of products to the customer.
Presence of ISI certification mark, known as Standard Mark, on a product is an assurance of conformity to the specifications.
Under Product Certification , Mandatory certification is there for Automobile Accessories such as Automotive vehicles –Tubes and pneumatic tyres
AGMARK (Agricultural Mark) is a certification issued by Directorate of Marketing and Inspection for agricultural produce.

Q 8. What is/are the advantage/advantages of implementing the `National Agriculture Market’ scheme?
1. It is a pan-India electronic trading portal for agricultural commodities.
2. It provides the farmers access to nationwide market, with prices commensurate with the quality of their produce.
Select the correct answer using the code given below:
(a) 1 only
(b) 2 only
(c) Both 1 and 2
(d) Neither 1 nor 2
Ans. C
Exp:  National Agriculture Market (NAM) is a pan-India electronic trading portal which networks the existing APMC (Agricultural produce market Committee) mandis to create a unified national market for agricultural commodities.
NAM created a unified market through online trading platform, both, at State and National level and promotes uniformity, streamlining of procedures across the integrated markets, removes information asymmetry between buyers and sellers and promotes real time price discovery, based on actual demand and supply, promotes transparency in auction process, and access to a nationwide market for the farmer, with prices commensurate with quality of his produce and online payment and availability of better quality produce and at more reasonable prices to the consumer.

Source:  http://iasscore.in/upsc-prelims/budget

Q 9. With reference to the ‘National. Intellectual Property Rights Policy’, consider the following statements:
1. It reiterates India’s commitment to the Doha Development Agenda and the TRIPS Agreement.
2. Department of Industrial Policy and Promotion is the nodal agency’ for regulating intellectual property rights in India.
Which of the above statements is/are correct?
(a) 1 only
(b) 2 only
(c) Both 1 and 2
(d) Neither 1 nor 2
Ans. C
Exp:  The National IPR Policy recognizes that India has a well-established TRIPS-compliant legislative, administrative and judicial framework to safeguard IPRs, which meets its international obligations while utilizing the flexibilities provided in the international regime to address its developmental concerns.
 It reiterates India’s commitment to the Doha Development Agenda and the TRIPS agreement.
IPR Policy objectives are sought to be achieved through detailed action plan which is monitored by DIPP (the nodal department to coordinate, guide and oversee implementation and future development of IPRs in India) 

Source: http://iasscore.in/upsc-prelims/policies-after-independence

Q 10. According to the Wildlife (Protection) Act, 1972, which of the following animals cannot be hunted by any person except under some provisions provided by law?
1. Gharial
2. Indian wild ass
3. Wild buffalo
Select the correct answer using the code given below:
(a) 1 only
(b) 2 and 3 only
(c) 1 and 3 only
(d) 1, 2 and  3
Ans: D
Exp: The Government of India subsequently accorded the highest level of protection to Gharial by bringing it under Schedule I of the Wild Life Protection Act, 1972. In 1976, Project Crocodile was initiated with support from the United Nations Development Programme and Food and Agriculture Organization.
The Indian wild ass is a subspecies of the onager native to Southern Asia. As of 2016, it is listed as Near Threatened by IUCN.
Wild Buffalo is also listed under the Schedule I of the Wildlife Protection Act, 1972

Q 11. Which of the following statements is/are true of the Fundamental Duties of an Indian citizen?
1. A legislative process has been provided to enforce these duties.
2. They are, correlative to legal duties.
Select the correct answer using the code given below:
(a) only 1
(b) 2 only
(c) Both 1 and 2
(d) Neither 1 nor 2

Answer – D
Exp: Fundamental duties are neither enforceable, nor have legal recognition

Source: http://www.iasscore.in/upsc-prelims/directive-principles-state-policy-fundamental-duties

Q 12. Consider the following pairs:
1. Radhakanta Deb –
First President of the British Indian Association
2.Gazulu Lakshminarasu Chetty –
Founder of the Madras Mahajana Sabha
3. Surendranath Banerjee –
Founder of the Indian Association
Which of the above pairs is/are correctly matched?
(a) 1 only
(b) 1 and 3 only
(c) 2 and 3 only
(d) 1, 2 and3

Answer – B
Explanation:Madras Mahajana Sabha was established by M. Veeraraghavachariar, G. SubramaniaIyer and P. Anandacharluin May 1884

Source: http://iasscore.in/upsc-prelims/rise-of-nationalism-indian-national-congress-inc

Q 13. Which one of the following objectives is not embodied in the Preamble to the Constitution of India?
(a) Liberty of thought
(b) Economic liberty
(c) Liberty of expression
(d) Liberty of belief

Answer – B
Exp:Line 3 of the Preamble mentions,
LIBERTY of thought, expression, belief, faith and worship;

Source: http://iasscore.in/upsc-prelims/preamble-and-values-in-constitution

Q 14. With reference to `Quality Council of India (QCI)’, consider the following statements:
1. QCI was set up jointly by the Government of India and the Indian Industry.
2. Chairman of QCI is appointed by the Prime Minister on the recommendations of the industry to the Government.
Which of the above statements is/are correct?
(a) 1 only
(b) 2 only
(c) Both 1 and 2
(d) Neither 1 nor 2

Answer – C
Exp: Formed in 1997, the Quality Council of India was set-up by the Confederation of Indian Industry, the Federation of Indian Chambers of Commerce and Industry, and the Associated Chambers of Commerce and Industry.
QCI is governed by a Council comprising of 38 members including the Chairman and Secretary General. The Council has an equal representation of Government, Industry and other Stakeholders.
The chairman selection is approved by the PM upon suggestion from the body stakeholders.

Q 15. What is the purpose of setting up of Small Finance Banks (SFBs) in India?
1. To supply credit to small business units
2. To supply credit to small and marginal farmers
3. To encourage young entrepreneurs to set up business particularly in rural areas.
Select the correct answer using the code given below:
(a) 1 and 2 only
(b) 2 and 3 only
(c) 1 and 3 only
(d) 1, 2and 3

Answer – A
Exp: RBI’s guidelines for Small Banks
Eligibility – Professionals with 10 years of experience in banking / finance / Micro Finance Institutions.
The minimum capital requirement is Rs 100 crore (minimum paid-up equity capital).
Local focus and ability to serve smaller customers will be a key criterion in licensing such banks.
The bank shall primarily undertake basic banking activities of accepting deposits and lending to small farmers, small businesses, micro and small industries, and unorganized sector entities. It cannot set up subsidiaries to undertake non-banking financial services activities. After the initial stabilization period of five years, and after a review, the RBI may liberalize the scope of activities for Small Banks.
The area of operations would normally be restricted to few districts (near-by) of a state. However, if necessary, it would be allowed to expand its area of operations beyond contiguous districts in one or more states with reasonable geographical proximity.
Small Banks have to meet RBI’s norms and regulations regarding risk management. They have to meet CRR and SLR requirements, like any other commercial bank.
The maximum loan size and investment limit exposure to single/group borrowers/issuers would be restricted to 15 per cent of capital funds.
For the first three years, 25 per cent of branches should be in unbanked rural areas.
Of the loans issued by Small Banks, 75% should be to the so-called priority sector which includes agriculture and small businesses. And half the loan portfolio of the banks should be loans and advances of up to Rs.25 lakh to micro finance businesses.

Source: http://iasscore.in/upsc-prelims/types-of-banksclassification-of-banks

Q 16. With reference to `Asia Pacific’ Ministerial Conference on Housing and Urban Development (APMCHUD)’, consider the following statements:
1. The first APMCHUD was held in India in 2006 on the theme `Emerging Urban Forms – Policy Responses and Governance Structure’.
2. India hosts all the Annual Ministerial Conferences in partnership with ADB, APEC and ASEAN.
Which of the statements given above is/are correct?
(a) 1 only
(b) 2 only
(c) Both 1 and 2
(d) Neither 1 nor 2

Answer: D
Exp: The first conference was held in New Delhi in 2006, but its theme was, A vision for sustainable urbanisation in the Asia-Pacific by 2020.

Q 17. ‘Democracy’s superior virtue lies in the fact that it calls into activity
(a) the intelligence and character ordinary men and women.
(b) the methods for strengthening executive leadership
(c) a superior individual with dynamism and vision.
(d) a band of dedicated party workers.

Answer: A
Exp: A democracy is superior, because it allows a right to every adult citizen, without any educational and wealth criterion.

Source: http://iasscore.in/upsc-prelims/types-of-political-systemtypes-of-governmenttypes-of-elections

Q 18. Which of the following is a most likely consequence of implementing the `Unified Payments Interface (UPI)’?
(a) Mobile wallets will not be necessary for online payments.
(b) Digital currency will totally replace the physical currency in about two decades.
(c) FDI inflows will drastically increase.
(d) Direct transfer of subsidies to poor people will become very effective.

Answer: A

Source: http://iasscore.in/upsc-prelims/unified-payment-interface

Q 19. The terns `Event Horizon’, ‘Singularity’, `String Theory’ and `Standard Model’ are sometimes seen in the news in the context of
(a) Observation and understanding of the Universe
(b) Study of the solar and the lunar eclipses
(c) Placing satellites in the orbit of the Earth
(d) Origin and evolution of living organisms on the Earth

Answer: A
Exp: All these terms come from theoretical physics branch, which investigates into state of matter and composition, origin, and extent of universe.

Q 20. With reference to agriculture in India, how can the technique of `genome sequencing’, often seen in the news, be used in the immediate future?
1. Genome sequencing can be used to identify genetic markers far disease resistance and drought tolerance in various crop plants.
2. This technique helps in reducing the time required to develop new varieties of crop plants.
3. It can be used to decipher the host-pathogen relationships in crops.
Select the correct- answer using the code given below:
(a) 1 only
(b) 2 and 3 only
(c) 1 and 3 only
(d) 1, 2 and 3

Answer: D

Source: http://iasscore.in/upsc-prelims/latest-updates-in-biotech-sector

Q 21. The main advantage of the parliamentary form of government is that
(a) the executive and legislature work independently.
(b) it provides continuity of policy and is more efficient.
(c) the executive remains responsible to the legislature.
(d) the head of the government cannot be changed without election.

Ans. C
Exp: • Executive branch is directly responsible to the legislature.
• A parliamentary system is a system of democratic governance of a state where the executive branch derives its democratic legitimacy from its ability to command the confidence of the legislative branch.

Source: http://iasscore.in/upsc-prelims/types-of-political-systemtypes-of-governmenttypes-of-elections

Q 22. In the context of India, which one of the following is the correct relationship between Rights and Duties?
(a) Rights are correlative with Duties.
(b) Rights are personal and hence independent of society and Duties.
(c) Rights, not Duties, are important for the advancement of the personality of the citizen.
(d) Duties, not Rights, are important for the stability of the State.

Ans. A
Exp: • Rights and duties are two phases of the same thing. Rights are considered to be essential for the expansion of human personality. They offer to the individual a sufficient scope for free action and thus prepare ground for self-development. Although rights arc of great significance in a democratic stale yet they become meaningless in the absence of duties.
• Rights can be enjoyed only in the world of duties. For every right there is corresponding duty. When the people fail to discharge their duties properly, the rights all become meaningless.

Q 23. The mind of the makers of the Constitution of India is reflected in which of the following?
(a) The Preamble
(b) The Fundamental Rights
(c) The Directive Principles of State Policy
(d) The Fundamental Duties

Ans A
Exp: • Preamble is the key to open the mind of the makers.
• The Preamble to a Constitution embodies the fundamental values and the philosophy, on which the Constitution is based, and the aims and objectives, which the founding fathers of the Constitution enjoined the polity to strive to achieve. The importance and utility of the Preamble has been pointed out in several decisions of the Supreme Court of India.

Source: http://iasscore.in/upsc-prelims/preamble-and-values-in-constitution

Q 24. If you travel by road from Kohima to Kottayam, what is the minimum number of States within India through which you can travel, including the origin and the destination?
(a) 6
(b) 7
(c) 8
(d) 9

Ans B
Exp:  Nagaland (Kohima) – Assam – WestBengal – Orissa – Andhra Pradesh – Tamilnadu – Kerala(Kottayam)

Q 25. The Parliament of India exercises control over the functions of the Council of Ministers through
1. Adjournment motion
2. Question hour
3. Supplementary questions
Select the correct answer using the code given below:
(a) 1 only
(b) 2 and 3 only
(c) 1 and 3 only
(d) 2 and 3

Ans D
Exp: • The Parliament keeps a day-to-day watch over the activities of the Executive. As ours is a parliamentary system of Government, the Executive is responsible to the Parliament for all acts.
• Members of the Parliament have a right to ask questions and supplementary question to the Ministers. Any lapses or mishandling on the part of the Government can be exposed in the Parliament.
• Adjournment motions may be moved to discuss serious administrative lapses. Through adjournment motions, matters of public importance can be brought to the notice of the Government by the members of the Parliament.

Source: http://iasscore.in/upsc-prelims/parliamentary-functioning-passage-bill-budgeting

Q 26. With reference to the Parliament of India, consider the following statements:
1. A private member’s bill is a bill presented by a Member of Parliament who is not elected but only nominated by the President of India.
2. Recently, a private member’s bill has been passed in the Parliament of India for the first time in its history.
Which of the statements given above is/are correct?
(a) 1 only
(b) 2 only
(c) Both 1 and 2
(d) Neither 1 nor 2

Ans D
Exp: • Members of parliament other than ministers are called private members and bills presented by them are known as private member’s bills.
• Any MP can introduce a bill in parliament. Private member bills are bills introduced in parliament by MPs who are not ministers.
• Only 14 private members’ bills passed since Independence. The Rights of Transgender Persons Bill, 2014, passed by the Rajya Sabha on Friday is the first private member’s bill to get the ujpper house’s approval in the past 45 years.

Source: http://iasscore.in/upsc-prelims/parliamentary-functioning-passage-bill-budgeting

Q 27. With reference to .the difference between the culture of Rigvedic Aryans and Indus Valley people, which of the following statements is/are correct?
1. Rigvedic Aryans used the coat of mail and helmet in warfare whereas the people of Indus Valley Civilization did not leave any evidence of using them.
2. Rigvedic Aryans knew gold, silver and copper whereas Indus Valley people knew only copper and iron.
3. Rigvedic Aryans had domesticated the horse whereas there is no evidence of Indus Valley people having been aware of this animal.
Select the correct answer using the code given below:
(a) 1 only
(b) 2 and 3 only
(c) 1 and 3 only
(d) 1, 2 and 3

Ans C*
Exp: • Indus people were basically peace loving. Their arms (swords, daggers, arrow-heads, and spears) were primitive in nature. No evidence of armour, helmet, body armour or shield is available. The Aryans, on the contrary, were warlike people and were conversant with all kinds of traditional arms and armour and had devised a full-fledged ‘science of war’.
• The horse, which played a decisive role in the Aryan system of warfare, was not known to the Indus people. A few bones of horse and terracotta figure of a ‘horse-like animal’ have been unearthed from Surkotada (Gujarat) and still it has not been convincingly proved that the horse was employed by the Harappans.
• The Indus valley people knew the use of copper, bronze, silver, gold but not iron.

Source: http://iasscore.in/upsc-prelims/indus-valley-civilization-sites-harrapan-culture

Q 28. ‘Recognition of Prior Learning Scheme’ is sometimes mentioned in the news with reference to
(a) Certifying the skills acquired by construction workers through traditional channels.
(b) Enrolling the persons in Universities for distance learning programmes.
(c) Reserving some skilled jobs to rural and urban poor in some public sector undertakings.
(d) Certifying the skills acquired by trainees under the National Skill Development Programme.

Ans. D*
Exp: Pradhan Mantri Kaushal Vikas Yojana (PMKVY) is the flagship scheme of the Ministry of Skill Development & Entrepreneurship (MSDE) with the objective of this Skill Certification Scheme to enable a large number of Indian youth to take up industry-relevant skill training that will help them in securing a better livelihood.
Recognition of Prior Learning Scheme
Individuals with prior learning experience or skills shall be assessed and certified under the Recognition of Prior Learning (RPL) component of the Scheme. RPL aims to align the competencies of the unregulated workforce of the country to the NSQF (National Skills Qualification Framework).

Q 29. From the ecological point of view, which one of the following assumes importance in being a good link between the Eastern Ghats and the Western Ghats?
(a) Sathyamangalam Tiger Reserve
(b) Nallamala Forest
(c) Nagarhole National Park
(d) Seshachalam Biosphere Reserve

Ans. A
Exp: • Sathyamangalam Tiger Reserve Sathyamangalam Wildlife Sanctuary and Tiger Reserve is a protected area and tiger reserve along the Western Ghats in the Indian state of Tamil Nadu
• Sathyamangalam forest range is a significant wildlife corridor in the Nilgiri Biosphere Reserve between the Western Ghats and the rest of the Eastern Ghats

Q 30. One of the implications of equality in society is the absence of
(a) Privileges
(b) Restraints
(c) Competition
(d) Ideology

Ans. A
Exp: Equality in society is The Absence of Special Privileges. Thus no person, family or class or group or persons in a society can be granted special privileges if we have to achieve Equality and liberty in a society

Q 31. Consider the following statements in respect of Trade Related Analysis of Fauna and Flora in Commerce (TRAFFIC):
1. TRAFFIC is a bureau under United Nations Environment Programme (UNEP).
2. The Mission of TRAFFIC is to ensure that trade in wild plants and animals is not a threat to the conservation of nature.
Which of the above statements is/are correct?
(a) 1 only
(b) 2 only
(c) Both 1 and 2
(d) Neither 1 nor 2
Ans: B
Exp: TRAFFIC has an enviable reputation as a reliable and impartial organisation, a leader in the field of conservation as it relates to wildlife trade. TRAFFIC was established in 1976 and has developed into a global network, research-driven and action-oriented, committed to delivering innovative and practical conservation solutions based on the latest information.
TRAFFIC also works in close co-operation with the Secretariat of the Convention on International Trade in Endangered Species of Wild Fauna and Flora (CITES).
TRAFFIC, the wildlife trade monitoring network, works to ensure that trade in wild plants and animals is not a threat to the conservation of nature.

Q 32. Which principle among the following was added to the Directive Principles of State Policy by the 42nd Amendment to the constitution?
a) Equal pay for equal work for both men and women
b) Participation of workers in the management of industries
c) Right to work, education and public assistance
d) Securing living wage and human conditions of work to workers
Ans: B
Exp: 42nd amendment act added participation of workers in the management of industries.
New directives was added by new articles 39A, 43A, 48A which, respectively, provide for equal justice and free legal aid to economically backward classes, participation of workers in the management of industries, and protection and improvement of environment and safeguarding of forests and wildlife.

Source: http://iasscore.in/upsc-prelims/directive-principles-state-policy-fundamental-duties

Q 33. Which one of the following statements is correct?
(a) Right are claims of the State against the citizens.
(b) Rights are privileges which are incorporated in the Constitution of a State.
(c) Rights are claims of the citizens against the State.
(d) Rights are privileges of a few citizens against the many.
Ans: c
Exp: Rights are claims of citizen of India against the states. They prevent the establishment of an authoritarian and despotic rule in the country, and protect the liberties and freedoms of the people against the invasion by the State. They operate as limitations on the tyranny of the executive and arbitrary laws of the legislature. In short, they aim at establishing ‘a government of laws and not of men’.

Q 34. Which of the following gives ‘Global Gender Gap Index’ ranking to the countries of the world?
(a) World Economic Forum
(b) UN Human Rights Council
(c) UN Women
(d) World Health Organization
Ans: a
Exp: The Global Gender Gap Report was first published in 2006 by the World Economic Forum. The 2016 report covers 144 major and emerging economies. The Global Gender Gap Index is an index designed to measure gender equality.

Source: http://iasscore.in/upsc-prelims/india-ranking-different-indexes-2016

Q 35. Which of the following statements is/are correct regarding Smart India Hackathon 2017?
1. It is a centrally sponsored scheme for developing every city of our country into Smart Cities in a Decade.
2. It is an initiative to identify new digital technology innovations for solving many problems faced by our country.
3. It is a programme aimed at making all the financial transactions in our country completely digital in a decade.
Select the correct answer using the code given below:
(a) 1 and 3 only
(b) 2 only
(c) 3 only
(d) 2 and 3 only
Ans: b
Exp: Smart India Hackathon 2017 harnesses creativity & expertise of students, builds funnel for ‘Startup India, Standup India’ campaign, crowd sources solutions for improving governance and quality of life, and provides opportunity to citizens to provide innovative solutions to India’s daunting problems.

Source: http://www.iasscore.in/upsc-prelims/ugc-and-aicte

Q 36. Which of the following statements is/are correct regarding the Monetary Policy Committee (MPC)?
1. It decides the RBI’s benchmark interest rates.
2. It is a 12-member body including the Governor of RBI and is reconstituted every year.
3. It functions under the chairmanship of the Union Finance Minister.
Select the correct answer using the code given below:
(a) 1 only
(b) 1 and 2 only
(c) 3 only
(d) 2 and 3 only
Ans: a
Exp: The Monetary Policy Committee would be entrusted with the task of fixing the benchmark policy rate (repo rate) required to contain inflation within the specified target level.
It consists of 6 members. The Members of the Monetary Policy Committee appointed by the Central Government shall hold office for a period of four years.
The Governor of the Bank—Chairperson, ex officio;

Q. 37. With reference to Manipuri Sankirtana, consider the following statements:
1. It is a song and dance performance.
2. Cymbals are the only musical instruments used in the performance.
3. It is performed to narrate the life and deeds of Lord Krishna.
Which of the statements given above is/are correct?
(a) 1, 2 and 3
(b) 1 and 3 only
(c) 2 and 3 only
(d) 1 only
Ans: b
Exp: Manipuri Sankirtana is a form of performing art involving ritual singing, drumming and dancing performed in the temples and domestic spaces in Manipur State in India. Through the performances which exhibit unparalleled religious devotion and energy, the performers narrate the many stories of Krishna often moving the spectators to tears. Dances are very much based upon the cymbals (kartal or manjira) and the cylindrical drum known as Manipuri mridang or pung.

Q 38. Who among the following was/were associated with the introduction of Ryotwari Settlement in India during the British rule?
1. Lord Cornwallis
2. Alexander Read
3. Thomas Munro
Select the correct answer using the code given below:
(a) 1 only
(b) 1 and 3 only
(c) 2 and 3 only
(d) 1, 2 and 3
Ans: c
Exp: Lord Cornwallis is associated with zamindari system.

Source: http://www.iasscore.in/upsc-prelims/administration-before-1857-economic-policies

Q 39. In the context of solving pollution problems what is/are the advantage/advantages of bioremediation technique?
1. It is a technique for cleaning up pollution by enhancing the same biodegradation process that occurs in nature.
2. Any contaminant with heavy metals such as cadmium and lead can be readily and completely treated by bioremediation using microorganisms.
3. Genetic engineering can be used to create microorganisms specifically designed for bioremediation.
Select the correct answer using the code given below:
a) 1 only
b) 2 and 3 only
c) 1 and 3 only
d) 1, 2 and 3
Ans: D
Exp: Bioremediation provides a technique for cleaning up pollution by enhancing the same biodegradation processes that occur in nature. Depending on the site and its contaminants, bioremediation may be safer and less expensive than alternative solutions such as incineration or landfilling of the contaminated materials. It also has the advantage of treating the contamination in place so that large quantities of soil, sediment or water do not have to be dug up or pumped out of the ground for treatment.
However, not all contaminants are easily treated by bioremediation using microorganisms. For example, heavy metals such as cadmium and lead are not readily absorbed or captured by microorganisms, but occurs in phytoremediation which is part of bioremediation.
The use of genetic engineering to create organisms specifically designed for bioremediation has great potential.

Q 40. The Trade Disputes Act of 1929 provided for
a) the participation of workers in the management of industries.
b) arbitrary powers to the management to quell industrial disputes.
c) an intervention by the British Court in the event of a trade dispute.
d) a system of tribunals and a ban on strikes.
Ans: a

Q 41. Local self-government can be best explained as an exercise in
a) Federalism
b) Democratic decentralization
c) Administrative delegation
d) Direct democracy
Ans: b
Exp: The main purpose of democratic decentralization, however, is to bring fundamental changes in the traditional outlook about the power structure of the government. Thus, democratic decentralization means decentralization of power. The source from which this power is decentralized is based on the democratic structure and hence, such decentralization is called the democratic decentralization.

Source: http://iasscore.in/upsc-prelims/local-government-urban-government

Q 42. Consider the following statements:
With reference to the Constitution of India, the Directive Principles of State Policy constitute limitations upon
1. legislative function
2. executive function
Which of the above statements is/are correct?
a) 1 only
b) 2 only
c) Both 1 and 2
d) Neither 1 nor 2
Ans: d
Exp: DPSP does not place any limitation on legislative and executive function, it is simply a guideline.

Source: http://iasscore.in/upsc-prelims/directive-principles-state-policy-fundamental-duties

Q 43. The term ‘Digital Single Market Strategy’ seen in the news refers to
a) ASEAN
b) BRICS
c) EU
d) G20
Ans: c
Exp: The Digital Single Market strategy aims to open up digital opportunities for people and business and enhance Europe’s position as a world leader in the digital economy.

Q 44. At one of the places in India, if you stand on the seashore and watch the sea, you will find that the sea water recedes from the shore line a few kilometers and comes back to the shore, twice a day, and you can actually walk on the sea floor when the water recedes. This unique phenomenon is seen at
a) Bhavnagar
b) Bheemunipatnam
c) Chandipur
d) Nagapattinam
Ans: c
Exp: The beach is unique in that the water recedes up to 5 kilometers during the ebb tide. When the tide is in, locals trawl for small fish along the coast.

Q 45. With reference to the ‘Prohibition of Benami Property Transactions Act, 1988 (PBPT Act)’, consider the following statements:
1. A property transaction is not treated as a benami transaction if the owner of the property is not aware of the transaction.
2. Properties held benami are liable for confiscation by the Government.
3. The Act provides for three authorities for investigations but does not provide for any appellate mechanism.
Which of the statements given above is/are correct?
a) 1 only
b) 2 only
c) 1 and 3 only
d) 2 and 3 only
Ans: b
Exp: Benami Transactions (Prohibition) Act, 1988 is an Act of the Parliament of India that prohibits certain types of financial transactions. The act defines a ‘benami’ transaction as any transaction in which property is transferred to one person for a consideration paid by another person. Such transactions were a feature of the Indian economy, usually relating to the purchase of property (real estate), and were thought to contribute to the Indian black money problem. The act bans all benami transactions and gives the government the right to recover property held benami without paying any compensation.

Q 46. Due to some reasons, if there is a huge fall in the population of species of butterflies, what could be its likely consequence/consequences?
1. Pollination of some plants could be adversely affected.
2. There could be a drastic increase in the fungal infections of some cultivated plants.
3. It could lead to a fall in the population of some species of wasps, spiders and birds.
Select the correct answer using the code given below:
a) 1 only
b) 2 and 3 only
c) 1 and 3 only
d) 1, 2 and 3
Ans: c
Exp:
pollination is affected adversely and the predators of butterflies are also affected, do not have impact on fungal infection.

Q 47. It is possible to produce algae based biofuels, but what is/are the likely limitation(s) of developing countries in promoting this industry?
1. Production of algae based biofuels is possible in seas only and not on continents.
2. Setting up and engineering the algae based biofuel production requires high level of expertise/technology until the construction is completed.
3. Economically viable production necessitates the setting up of large scale facilities which may raise ecological and social concerns.
Select the correct answer using the code given below:
a) 1 and 2 only
b) 2 and 3 only
c) 3 only
d) 1, 2 and 3
Ans: b
Exp: Algae can be grown using land and water not suitable for plant and food production. The algal biofuel technology includes selection of specific species for production and extraction of valuable co-products. The algaes are bioengineered for achieving advanced photosynthetic efficiencies through continued development of production system.
To make the industry commercially viable, researchers have pursued biotech varieties, which could be particularly dangerous if released into nature. Some algae biodiesel proposals involve aquaculture-style operations in open ocean waters, which could have harmful ecological effects, especially if biotech algae is used.
Water and nutrient use would also be extreme, making any serious scaling up of algae biofuels quite unsustainable.

Source: http://iasscore.in/upsc-prelims/algae-fuel

Q 48. Which of the following are the objectives of ‘National Nutrition Mission’?
1. To create awareness relating to malnutrition among pregnant women and lactating mothers.
2. To reduce the incidence of anaemia among young children, adolescent girls and women.
3. To promote the consumption of millets, coarse cereals and unpolished rice.
4. To promote the consumption of poultry eggs.
Select the correct answer using the code given below:
a) 1 and 2 only
b) 1, 2 and 3 only
c) 1, 2 and 4 only
d) 3 and 4 only
Ans: a
Exp: The key objectives of these programmes are as under:
• To create awareness relating to malnutrition amongst pregnant women, lactating mothers, promote healthy lactating practices and importance of balanced nutrition;
• To improve maternal and child under-nutrition in 200 high burdened districts and to prevent and reduce the under-nutrition prevalent among children below 3 years;
• To reduce incidence of anaemia among young children, adolescent girls and women.

Source: http://www.iasscore.in/upsc-prelims/women-children-youth

Q 49. Consider the following statements:
1. The Factories Act, 1881 was passed with a view to fix the wages of industrial workers and to allow the workers to form trade unions.
2. N.M. Lokhande was a pioneer in organizing the labour movement in British India.
Which of the above statements is/are correct?
a) 1 only
b) 2 only
c) Both 1 and 2
d) Neither 1 nor 2
Ans: B
Exp: Narayan Meghaji Lokhande (1848-1897) was a pioneer of the labour movement in India.
During Lord Ripon’s time, the first Factories Act was adopted in 1881. Following this act , a Factory Commission was appointed in 1885. There was another Factories Act in 1891, and a Royal Commission on Labor was appointed in 1892. The result of these enactments was the limitation on the factory working hours. This was an answer of the Government to the pathetic conditions of the workers in the factory, wherein, only when a laborer exhausted, new laborer was to take his / her place.

Q 50. In the context of mitigating the impending global warming due to anthropogenic emission of carbon dioxide, which of the following can be the potential sites for carbon sequestration?
1. Abandoned and Uneconomic coal seams
2. Depleted oil and gas reservoirs
3. Subterranean deep saline formations
Select the correct answer using the code given below:
a) 1 and 2 only
b) 3 only
c) 1 and 3 only
d) 1, 2 and 3
Ans: d
Exp: Capturing CO2 from the major stationary sources and its storage into deep geological formations is considered as a potential mitigation option. Geological storage of CO2 can be undertaken in a variety of geological settings in sedimentary basins.
The options for CO2 are as follows:
• Depleted oil and gas reservoirs
• Use of CO2 in FOR
• Deep unmineable coal seams/enhanced coalbed methane (ECBM) recovery
• Oceans
• Deep unused saline water-saturated formations
• Other geological media are basalts, shales, and cavities
• Subterranean deep saline formations

Q 51. Consider the following statements:
1. In the election for Lok Sabha or State Assembly, the winning candidate must get at least 50 percent of the votes polled, to be declared elected.
2. According to the provisions laid down in the Constitution of India, in Lok Sabha, the Speaker’s post goes to the majority party and the Deputy Speaker’s to the Opposition.
Which of the statements given above is/are correct?
a) 1 only
b) 2 only
c) Both 1 and 2
d) Neither 1 nor 2
Ans: d
Exp: India follows First Past the Post system, there is no provision for 50% votes. There is no provision related to it in the Constitution.

Source: http://iasscore.in/upsc-prelims/types-of-political-systemtypes-of-governmenttypes-of-elections

Q 52. Which of the following has/have occurred in India after its liberalization of economic policies in 1991?
1. Share of agriculture in GDP increased enormously.
2. Share of India’s exports in world trade increased.
3. FDI inflows increased.
4. India’s foreign exchange reserves increased enormously.
Select the correct answer using the codes given below:
a) 1 and 4 only
b) 2, 3 and 4 only
c) 2 and 3 only
d) 1, 2, 3 and 4
Ans: b
Exp: Share of agriculture in GDP is decreasing

Q 53. What is the application of Somatic Cell Nuclear Transfer Technology?
a) Production of biolarvicides
b) Manufacture of biodegradable plastics
c) Reproductive cloning of animals
d) Production of organisms free of diseases
Ans: c
Exp: Somatic cell cloning is a technique in which thenucleus (DNA) of a somatic cell is transferred into an enucleated metaphase-II oocyte for the generation of a new individual, genetically identical to the somatic celldonor

Q 54. Consider the following statements:
1. National Payments Corporation of India (NPCI) helps in promoting the financial inclusion in the country.
2. NCPI has launched RuPay, a card Payment scheme.
Which of the statements given above is/are correct?
a) 1 only
b) 2 only
c) Both 1 and 2
d) Neither 1 nor 2
Ans: c
Exp: RuPay is an Indian domestic card scheme conceived and launched by the National Payments Corporation of India (NPCI). It was created to fulfil the Reserve Bank of India’s desire to have a domestic, open loop, and multilateral system of payments in India. National Payments Corporation of India (NPCI) helps in promoting the financial inclusion in the country

Source: http://iasscore.in/upsc-prelims/financial-inclusionself-help-groups

Q 55. The term ‘M-STrIPES’ is sometimes seen in the news in the context of
a) Captive breeding of Wild Fauna
b) Maintenance of Tiger Reserves
c) Indigenous Satellite Navigation System
d) Security of National Highways
Ans: b
Exp: The full form of M-STrIPES is Monitoring System for Tigers’-Intensive Protection and Ecological Status. It’s a software monitoring system launched by the Indian Government in 2010 in some tiger reserves. The aim is to reduce vulnerability of Tigers. The system would enable field managers to assist intensity and spatial coverage of patrols in a geographic information system (GIS) domain

Q 56. What is/are the most likely advantages of implementing ‘Goods and Services Tax (GST)’?
1. It will replace multiple taxes collected by multiple authorities and will thus create a single market in India.
2. It will drastically reduce the ‘Current Account Deficit’ of India and will enable it to increase its foreign exchange reserves.
3. It will enormously increase the growth and size of economy of India and will enable it to overtake China in the near future.
Select the correct answer using the code given below:
a) 1 only
b) 2 and 3 only
c) 1 and 3 only
d) 1, 2 and 3
Ans: a
Exp: Statements 2 and 3 are probable but not deterministic.

Source: http://iasscore.in/upsc-prelims/fiscal-policy-revenue

Q 57. ‘Broad-based Trade and Investment Agreement (BTIA)’ is sometimes seen in the news in the context of negotiations held between India and
a) European Union
b) Gulf Cooperation Council
c) Organization for Economic Cooperation and Development
d) Shanghai Cooperation Organization
Ans: a
Exp: The EU and India have negotiated an ambitious Broad-based Trade and Investment Agreement since 2007.

Q 58. Consider the following statements:
1. India has ratified the Trade Facilitation Agreement (TFA) of WTO.
2. TFA is a part of WTO’s Bali Ministerial Package of 2013.
3. TFA came into force in January 2016.
Which of the statements given above is/are correct?
a) 1 and 2 only
b) 1 and 3 only
c) 2 and 3 only
d) 1, 2 and 3
Ans: a
Exp: WTO members concluded negotiations at the 2013 Bali Ministerial Conference on the landmark Trade Facilitation Agreement (TFA), which entered into force on 22 February 2017 following its ratification by two-thirds of the WTO membership. The TFA contains provisions for expediting the movement, release and clearance of goods, including goods in transit. It also sets out measures for effective cooperation between customs and other appropriate authorities on trade facilitation and customs compliance issues. It further contains provisions for technical assistance and capacity building in this area. It came into being in Feb, 2017.

Source: http://iasscore.in/upsc-prelims/national-committee-trade-facilitation

Q 59. What is the importance of developing Chabahar Port by India?
a) India’s trade with African countries will enormously increase.
b) India’s relations with oil-producing Arab countries will be strengthened.
c) India will not depend on Pakistan for access to Afghanistan and Central Asia.
d) Pakistan will facilitate and protect the installation of a gas pipeline between Iraq and India.
Ans: c
Exp: The port will make way for India to bypass Pakistan in transporting goods to Afghanistan using a sea-land route. At present, Pakistan does not allow India to transport through its territory to Afghanistan. It has, however, recently allowed some Afghan shipments to come to India.

This will also give momentum to the International North-South Transport Corridor of which both are initial signatories along with Russia. Iran is the key gateway in this project. It entails the ship, rail, and road routes for moving freight between India, Russia, Iran, Europe and Central Asia. The route primarily involves moving freight from India, Iran, Azerbaijan and Russia. The objective of the corridor is to increase trade connectivity between major cities such as Mumbai, Moscow, Tehran, Baku, Astrakhan etc.

Q 60. In India, it is legally mandatory for which of the following to report on cyber security incidents?
1. Service providers
2. Data Centres
3. Body corporate
Select the correct answer using the code given below:
a) 1 only
b) 1 and 2 only
c) 3 only
d) 1, 2 and 3
Ans: d
Exp: The CERT Rules impose an obligation on service providers, intermediaries, data centres and body corporates to report cyber incidents within a reasonable time so that CERT-In may have scope for timely action.

Q 61. Right to vote and to be elected in India is a
a) Fundamental Right
b) Natural Right
c) Constitutional Right
d) Legal Right

Ans: c
Exp: In India, the right to vote is provided by the Constitution and the Representation of People’s Act, 1951, subject to certain disqualifications. Article 326 of the Constitution guarantees the right to vote to every citizen above the age of 18. Further “ to be elected” is also a constitutional right.

Q 62. What is the purpose of ‘evolved Laser Interferometer Space Antenna (ELISA)’ project?
a) To detect neutrinos
b) To detect gravitational waves
c) To detect the effectiveness of missile defence system
d) To study the effect of solar flares on our communication systems

Ans: b
Exp: eLISA’s primary objective is the detection and examination of gravitational waves emitted by the super-massive black holes that reside in the centres of many galaxies. In addition, eLISA will measure the signals of thousands of compact binary star systems in the Milky Way.

Q 63. What is the purpose of Vidyanjali Yojana’?
1. To enable the famous foreign campuses in India.
2. To increase the quality of education provided in government schools by taking help from the private sector and the community.
3. To encourage voluntary monetary contributions from private individuals and organizations so as to improve the infrastructure facilities for primary and secondary schools.
Select the correct answer using the code given below:
(a) 2 only
(b) 3 only
(c) 1 and 2 only
(d) 2 and 3 only

Ans: a
Exp: Vidyanjali is an initiative to enhance community and private sector involvement in Government run elementary schools across the country under the overall aegis of the Sarva Shiksha Abhiyan. Through this initiative people from the Indian Diaspora, retired teachers, retired government officials including retired defence personnel, retired professionals and women who are home makers can volunteer at a school that requests for one.
Vidyanjali will also cover initiatives under the Corporate Social Responsibility (CSR) and Public Private Partnership (PPP), with Public Sector Undertaking (PSU) Companies, private corporate and others.

Source: http://iasscore.in/upsc-prelims/healtheducation

Q 64. What is the aim of the programme ‘Unnat Bharat Abhiyan’ ?
a) Achieving 100% literacy by promoting collaboration between voluntary organizations and , government’s education system and local communities.
b) Connecting institutions of higher education with local communities to address development challenges through appropriate techhologies.
c) Strengthening India’s scientific regearch institutions in order to make India a scientific and technological Power.
d) Developing human capital by allocating special funds for health- care and education of rural and urban poor, and organizing skill development programmes and vocational training for them.

Ans: b
Exp: The objectives of Unnat Bharat Abhiyan are broadly two-fold:

i) Building institutional capacity in Institutes of higher education in research & training relevant to the needs of rural India.
ii) Provide rural India with professional resource support from institutes of higher education ,especially those which have acquired academic excellence in the field of Science, Engineering & Technology and Management

Source: http://iasscore.in/upsc-prelims/healtheducation

Q 65. Consider the following statements:
1. The Election Commission of India is a ‘ five-member body.
2. Union Ministry of Home Affairs decides the election schedule for the conduct of both general elections and bye-elections.
3. Election Commission resolves the disputes relating to splits/mergers of recognized political parties.
Which of the statements given above is/are correct ?
(a) 1 and 2 only
(b) 2 only
(c) 2 and 3 only
(d) 3 only
Ans: d
Exp: Originally in 1950, the commission had only a Chief Election Commissioner. The Election Commissioner Amendment Act, 1989 made the Commission a multi-member body. The concept of a 3-member Commission has been in operation since then.
Home Ministry is consulted for that, schedule is not decided by them.
Splits, mergers and alliances have frequently disrupted the compositions of political parties. This has led to a number of disputes over which section of a divided party gets to keep the party symbol, and how to classify the resulting parties in terms of national and state parties. The Election Commission has to resolve these disputes, although its decisions can be challenged in the courts.

Source: http://iasscore.in/upsc-prelims/constitutional-bodies

Q 66. In India, if a species of tortoise is declared protected under Schedule I of the Wildlife (Protection) Act, 1972, what does it imply ?
a. It enjoys the same level of protection as the tiger.
b. It no longer exists in the wild, a few individuals are under captive protection; and how it is impossible to prevent its extinction.
c. It is endemic to a particular region of India.
d. Both (b) and (c) stated above are correct in this context.

Ans: a

Q 67. In India, Judicial Review implies
(a) the power of the Judiciary to pronounce upon the constitutionality of laws and executive orders.
(b) the power of the Judiciary to question the wisdom of the laws enacted by the Legislatures. (c) the power of the Judiciary to review- all the legislative enactments before they – are assented to by the President.
(c) the power of the Judiciary to review its own judgements given earlier in similar or different cases?

Ans: a
Exp: Judicial Review refers to the power of the judiciary to interpret the constitution and to declare any such law or order of the legislature and executive void, if it finds them in conflict the Constitution of India.

Source: http://iasscore.in/upsc-prelims/judiciary-terms

Q 68. With reference to Indian freedom struggle, consider the following events :
1. Mutiny in Royal Indian Navy
2. Quit India Movement launched
3. Second Round Table Conference
What is the correct chronological sequence of the above events ?

(a) 1-2-3
(b) 2-1-3
(c) 3-2-1
(d) 3-1-2

Ans: c
Exp: Mutiny in Royal Indian Navy: 18 February 1946, Quit India Movement: 8 August 1942, Second Round Conference: 1931.

Source: http://iasscore.in/upsc-prelims/closer-to-freedom-1930-1947

Q 69. Consider the following statements :
1. Tax revenue as a percent of GDP of India has steadily increased in the last decade.
2. Fiscal deficit as a percent of GDP of India has steadily increased in the last decade.
Which of the statements given- above is/are correct ?
(a) 1 only
(b) 2 only
(c) Both 1 and 2
(d) Either 1 nor 2

Ans: a
Exp: Fiscal Deficit has decreased in the last decade.

Q 70. Recently there was a proposal to translocate some of the lions from their natural habitat in Gujarat to which one of the following sites?
(a) Corbett National Park
(b) Kuno Palpur Wildlife Sanctuary
(c) Mudumalai Wildlife Sanctuary
(d) Sariska National Park

Ans: b
Exp: An environment ministry’s expert committee has approved Kuno Palpur in Madhya Pradesh as the second home for Asiatic lions found only in Gir national park.

71. Which of the following are not necessarily the consequences of the proclamation of the President’s rule in a State?
1. Dissolution of the State Legislative Assembly
2. Removal of the Council of Ministers in the State
3. Dissolution of the local bodies
Select the correct answer using the code given below:
(a) 1 and 2 only
(b) 1 and 3 only
(c) 2 and 3 only
(d) 1, 2 and 3

Ans: b
Exp: During President’s rule, the Council of Ministers stands dissolved.
According to Article 365, if a state government fails to exercise its executive powers in compliance with the directions given by the Union government, the responsibility shifts to the governor of the state to assess the situation and recommend the dissolution of state Assembly to the Union cabinet. This proclamation is made by the President only when the governor is convinced that the state cannot function in accordance with the provisions of the Constitution.
Although the power of dissolution of state Assembly is vested with the Governor, yet such a power can be exercised only after both Houses of Parliament approve the decision. If the proclamation made by the President under Article 356 to dissolve the Assembly is approved by both the Houses within two months, the government does not revive on the expiry of period of dissolution.
It is not related to the dissolution of the local bodies.

Source: http://iasscore.in/upsc-prelims/emergency

72. Which of the following are envisaged by the Right against Exploitation in the Constitution of India?
1. Prohibition of traffic in human beings and forced labour
2. Abolition of untouchability
3. Protection of the interests of minorities
4. Prohibition of employment of children in factories and mines
Select the correct answer using the code given below :
(a) 1, 2 and 4 only
(b) 2, 3 and 4 only
(c) 1 and 4 only
(d) 1, 2, 3 and 4

Ans: c
Exp: Article 23 prohibits the traffic in human beings and forced labor such as begar.
Article 24 is against the child labour.
Article 17 is related to untouchability.

Source: http://iasscore.in/upsc-prelims/basic-fundamental-rights-for-citizens-and-aliens-military-laws

73. Which of the following is geographically closest to Great Nicobar ?
(a) Sumatra
(b) Borneo
(c) Java
(d) Sri Lanka

Ans: a

Source: http://iasscore.in/upsc-prelims/himalaya-plain-plateaus-coastal-plain-islands

74. Out of the’ ‘following statements, choose the one that brings out the principle underlying the Cabinet form of Government :
(a) An arrangement for mimmizing the criticism against the Government whose responsibilities are complex and hard to carry out to the satisfaction of all.
(b) A mechanism for speeding up the activities of the Government whose responsibilities are increasing day by day.
(c) A mechanism of parliamentary democracy for ensuring collective responsibility of the Government to the people.
(d) A device for strengthening the hands of the head of the Government whose hold over the people is in a state of decline.

Ans: c

Source: http://iasscore.in/upsc-prelims/council-minister-prime-minister-roles-other-executives

75. Which one of the following is not a feature of Indian federalism ?
(a) There is an independent judiciary in India.
(b) Powers have been clearly divided between the Centre and the States.
(c ) The federating units have been given unequal representation in the Rajya Sabha.
(d) It is the result of an agreement among the federating units.

Ans: d*
Source: http://iasscore.in/upsc-prelims/administrative-legislative-financial-relations

76. Which one of the following was a very important seaport in the Kakatiya kingdom ?
(a) Kakinada
(b) Motupalli
(c) Machilipatnam (Masulipatnam)
(d) Nelluru

Ans: d

77. With, reference to ‘Global Climate Change Alliance’ which of the following statements is/are correct?
1. It is an initiative of the European Union.
2. It provides technical and financial support to targeted developing countries to integrate climate change into their development policies and budgets.
3. It is coordinated by World Resources Institute (WRI) and World Business Council for Sustainable Development (WBCSD).
Select the correct answer using the code given below :
(a) 1 and 2 only
(b) 3 only
(c) 2 and 3 only
(d) 1, 2 and 3

Ans: a
Exp: The Global Climate Change Alliance (GCCA) is an initiative of the European Union. Its overall objective is to build a new alliance on climate change between the European Union and the poor developing countries that are most affected and that have the least capacity to deal with climate change. The GCCA does not intend to set up a new fund or governance structure, but is working through the European Commission’s established channels for political dialogue and cooperation at national and international level.
The GCCA+ aim is to boost the efficiency of its response to the needs of vulnerable countries and groups. Using ambitious and innovative approaches, it will achieve its goals by building on its two mutually reinforcing pillars:
• Under the first pillar, the GCCA+ serves as a platform for dialogue and exchange of experience between the EU and developing countries, focusing on climate policy and bringing renewed attention to the issue of international climate finance. The results feed into negotiations for a new climate deal under the United Nations Framework Convention on Climate Change (UNFCCC).
• Under the second pillar, the GCCA+ acts as a source of technical and financial support for the world’s most climate-vulnerable countries, whose populations need climate finance the most. Extra efforts will be made to strengthen the strategically important issues of ecosystems-based adaptation, migration and gender equality.

78. With reference to the religious history of India, consider the following statements :
1. Sautrantika and Sammitiya were the sects of Jainism.
2. Sarvastivadin held that the constituents of phenomena were not wholly momentary, but existed forever in a latent form.
Which of the statements given above is/are correct ?
(a) 1 only
(b) 2 only
(c) Both 1 and 2
(d) Neither 1 nor 2

Ans: b
Exp: The two major sects include – Digambar(दिगम्बर) And Shwetambar(श्वेताम्बर).95% of the practices followed are same in both the sects.

79. Mediterranean Sea is a border of which of the following countries ?
1. Jordan
2. Iraq
3. Lebanon
4. Syria
Select the correct answer using the code given below:
(a) 1, 2 and 3 only
(b) 2 and 3 only
(c) 3 and 4 only
(d) 1, 3 and 4 only

Ans: c
Exp: The countries with coastlines on the Mediterranean Sea are Albania, Algeria, Bosnia and Herzegovina, Croatia, Cyprus, Egypt, France, Greece, Israel, Italy, Lebanon, Libya, Malta, Morocco, Monaco, Montenegro, Slovenia, Spain, Syria, Tunisia and Turkey. In addition, the Gaza Strip and the British Overseas Territories of Gibraltar and Akrotiri and Dhekelia have coastlines on the sea.

Source: http://www.iasscore.in/upsc-prelims/europe

80. With reference to ‘National Investment and Infrastructure Fund’, which of the following statements is/are correct?
1. It is an organ of NITI Aayog.
2. It has a corpus of Rs. 4,00,000 crore at present.
Select the correct answer using the code given below :
(a) 1 only
(b) 2 only
(c) Both 1 and 2
(d) Neither 1 nor 2

Ans: d
Exp: NIIF has been structured as a fund of funds and set up as Category II Alternate Investment Fund (AIF) under the Securities and Exchange Board of India (SEBI) Regulations. Total corpus of the fund is Rs. 40000 Crore.

Source: http://iasscore.in/upsc-prelims/national-investment-and-infrastructure-fund

81. The Global Infrastructure Facility is a/an
(a) ASEAN initiative to upgrade infrastructure in Asia and financed by credit from the Asian Development Bank.
(b) World Bank collaboration that facilitates the preparation and structuring of complex infrastructure Public-Private Partnerships (PPPs) to enable mobilization of private sector and institutional investor capital.
(c) Collaboration among the major banks of the world working with the OECD and focused on expanding the set of infrastructure projects that have the potential to mobilize private vestment.
(d) UNCTAD funded initiative that seeks to finance and facilitate infrastructure development in the world.

Ans. B
Exp: • The World Bank (WB) has launched the GIF to specifically cater to the infrastructure needs of the emerging economies and developing countries.
• The GIF will channel money towards bankable infrastructure project in such countries.
• GIF also places importance on sustainable development. Its key focus will be on climate friendly infrastructure investments and projects that will boost trade.
• Key Objectives :
o Mobilizing the private sector: Concentrating on financing and implementation modalities that draw together the comparative advantages of the public and private sectors to expand investment and improve infrastructure services.
o Collaborating for best results: Mobilizing partners’ technical and financial resources in an innovative and flexible way to achieve greater leverage and address the complex infrastructure financing challenges that are beyond the resources of any individual partner.

82. For election to the Lok Sabha, a nomination paper can be filed by
(a) Anyone residing in India.
(b) A resident of the constituency from which the election is to be contested.
(c) Any citizen of India whose name appears in the electoral roll of a constituency.
(d) any citizen of India.

Ans. C
Exp: • For contesting an election as a candidate a person must be registered as a voter.Sec 4 (d) of Representation People Act, 1951 precludes a person from contesting unless he is an elector in any parliamentary constituency.

Source: http://iasscore.in/upsc-prelims/political-parties-electoral-system

83. Consider the following statements:
1. In India, the Himalayas are spread over five States only.
2. Western Ghats are spread over five States only.
3. Pulicat Lake is spread over two States only.
Which of the statements given above is/are correct ?
(a) 1 and 2 only
(b) 3 only
(c) 2 and 3 only
(d) 1 and 3 only

Ans. B
Exp: • Western Ghats – A chain of mountains running parallel to India’s western coast, approximately 30-50 km inland, the Ghats traverse the States of Kerala, Tamil Nadu, Karnataka, Goa, Maharashtra and Gujarat.
• Pulicat Lake is the second largest brackish water lake or lagoon in India, after Chilka Lake. It straddles the border of Andhra Pradesh and Tamil Nadu states.

Source: http://iasscore.in/upsc-prelims/himalaya-plain-plateaus-coastal-plain-islands

84. Biological Oxygen Demand (BOD) is a standard criterion for
(a) Measuring oxygen levels in blood
(b) Computing oxygen levels in forest ecosystems
(c) Pollution assay in aquatic ecosystems
(d) Assessing oxygen levels in high altitude regions

Ans. C
Exp: • Biochemical Oxygen Demand is an important water quality parameter which provides an index to assess the effect discharged wastewater will have on the receiving environment.
• The higher the BOD value, the greater the amount of organic matter or “food” available for oxygen consuming bacteria.

Source: http://iasscore.in/upsc-prelims/surface-and-groundwater-degradation

85. With reference to the role of UN-Habitat in the United Nations programme working towards a better urban future, which of the statements is/are correct ?
1. UN-Habitat has been mandated by the United Nations General Assembly to promote socially and environmentally sustainable towns and cities to provide adequate shelter for all.
2. Its partners are either governments or local urban authorities only.
3. UN-Habitat contributes to the overall objective of the United Nations system to reduce poverty and to promote access-to safe drinking water and bask sanitation.
Select the correct answer using the code given below :
(a) 1, 2 and 3
(b) 1 and 3 only
(c) 2 and 3 only
(d) 1 only

Ans. B
Exp: • UN-Habitat, the United Nations Human Settlements Programme, is mandated by the UN General Assembly to promote socially and environmentally sustainable towns and cities. It is the focal point for all urbanization and human settlement matters within the UN system.
• UN-Habitat envisions well-planned, well-governed, and efficient cities and other human settlements, with adequate housing, infrastructure, and universal access to employment and basic services such as water, energy, and sanitation

86. With reference to ‘National Skills Qualification Framework (NSQF)’, which of the statements given below is/are correct?
1. Under NSQF, a learner can acquire the certification for competency only through formal learning.
2. An outcome expected from the implementation of NSQF is the mobility between vocational and general education:
Select the correct answer using the code given below:
(a) 1 only
(b) 2 only
(c) Both 1 and 2
(d) Neither 1 nor 2

Ans. b
Exp: The National Skills Qualifications Framework (NSQF) is a competency-based framework that organizes all qualifications according to a series of levels of knowledge, skills and aptitude. These levels, graded from one to ten, are defined in terms of learning outcomes which the learner must possess regardless of whether they are obtained through formal, non-formal or informal learning.
Specific outcomes expected from implementation of NSQF are:
i. Mobility between vocational and general education by alignment of degrees with NSQF
ii. Recognition of Prior Learning (RPL), allowing transition from non-formal to organised job market
iii. Standardised, consistent, nationally acceptable outcomes of training across the country through a national quality assurance framework
iv. Global mobility of skilled workforce from India, through international equivalence of NSQF
v. Mapping of progression pathways within sectors and cross-sectorally
vi. Approval of NOS/QPs as national standards for skill training

87. In the context of Indian history, the-principle of ‘Dyarchy (diarchy)’ refers to
(a) Division of the central legislature into two houses.
(b) Introduction of double government i.e.; Central and State governments.
(c) Having two sets of rulers; one in London and another in Delhi.
(d) Division of the subjects delegated to the provinces into two categories.

Ans. d

Source: http://iasscore.in/upsc-prelims/constitution-and-its-typesconstitutional-development

88. Consider the following in respect of ‘National Career Service’ :
1. National Career Service is an initiative of the Department of Personnel and Training, Government of India.
2. National Career Service has been launched in a Mission Mode to improve the employment opportunities to uneducated youth of the country.
Which of the above statements is/are correct ?
(a) 1 only
(b) 2 only
(c) Both 1 and 2
(d) Neither 1 nor 2

Ans. b
Exp: The Government is implementing the National Career Service Project for linking employment exchanges and other institutions using technology to provide a variety of employment related services like job postings, career counselling, vocational guidance, skill courses, apprenticeship, etc.

89. Which of the following statements best describes the- term ‘Scheme for Sustainable Structuring of Stressed Assets (S4A)’, recently seen in the news?
(a) It is a procedure for considering ecological costs of developmental schemes formulated by the Government.
(b) It is a scheme of RBI for reworking the financial structure of big corporate entities facing genuine difficulties.
(c) It is a disinvestment plan of the Government regarding Central Public Sector Undertakings.
(d) It is an important provision in ‘The Insolvency and Bankruptcy Code’ recently implemented by the Government.

Ans. b
Exp: In order to strengthen the lenders’ ability to deal with stressed assets and to put real assets back on track by providing an avenue for reworking the financial structure of entities facing genuine difficulties, the Reserve Bank of India has issued guidelines on a ‘Scheme for Sustainable Structuring of Stressed Assets’.

90. Consider the following statements :
1. Climate and Clean Air Coalition (CCAC) to Reduce Short Lived Climate Pollutants is a unique initiative of G20 group of countries;
2. The CCAC focuses on methane, black carbon and hydrofluorocarbons.
Which of the statements given above is/are correct ?
(a) 1 only
(b) 2 only
(c) Both 1 and 2
(d) Neither 1 nor 2

Ans. B
Exp: The Climate and Clean Air Coalition to Reduce Short-Lived Climate Pollutants (CCAC) was launched by the United Nations Environment Programme (UNEP) and six countries—Bangladesh, Canada, Ghana, Mexico, Sweden, and the United States—on 16 February 2012.
It focuses on O3, CH4, Black Carbon, and HFCs

91. With reference to ‘Indian Ocean Dipole (IOD)’. sometimes mentioned in the news while forecasting Indian monsoon, which of the following statements is/are correct?
1. IOD phenomenon is characterized by a difference in sea surface temperature between tropical Western Indian Ocean and tropical Eastern Pacific Ocean.
2. An IOD phenomenon can influence an El Nino’s impact on the monsoon.
Select the correct answer using the code given below:
(a) 1 only
(b) 2 only
(c) Both 1 and 2
(d) Neither 1 nor 2

Ans. B
Exp: The phenomenon is concerned to temperature differentials in Western Indian Ocean and Eastern Indian Ocean

Source: http://iasscore.in/upsc-prelims/climate-india-climatic-division-region-wise

92. If you want to see gharials in their natural habitat, which one of the following is the best place to visit ?
(a) Bhitarkanika Mangroves
(b) Chambal River
(c) Pulicat Lake
(d) DeeporBeel

Ans. B
Exp: National Chambal Gharial Wildlife Sanctuary is one of the dedicated sanctuary for Gharials.

93. Consider the following in respect of Indian Ocean Naval Symposium (IONS) :
1. Inaugural IONS was held in India in 2015 under the chairmanship of the Indian Navy.
2. IONS is a voluntary initiative that seeks to increase maritime co-operationamong navies of the littoral states of the Indian Ocean Region.
Which of the above statements is/are Correct ?
(a) 1 only
(b) 2 only
(c) Both 1 and 2
(d) Neither 1 -nor 2

Ans. B
Exp: The inaugural IONS-2008 was held in New Delhi, India on 14 Feb 08.
There are 35 members – navies of the IONS which have been geographically grouped into the following four sub-regions:-
• South Asian Littorals – Bangladesh, India, Maldives, Pakistan, Seychelles and Sri Lanka
• West Asian Littorals – Bahrain, Iran, Iraq, Kuwait, Oman, Qatar, Saudi Arabia, UAE and Yemen
• East African Littorals – Comoros, Djibouti, Egypt, Eriteria, France, Kenya, Madagascar, Mauritius, Mozambique, Somalia, South Africa, Sudan and Tanzania
• South East Asian and Australian Littorals – Australia, Indonesia, Malaysia, Myanmar, Singapore, Thailand and Timor Leste

94. The painting of Bodhisattva Padmapani is one of the most famous and oft-illustrated paintings at
(a) Ajanta
(b) Badami
(c) Bagh
(d) Ellora

Ans. A

95. Consider the following pairs :
Traditions Communities
1. Chaliha Sahib Festival — Sindhis
2. Nanda Raj Jaat Yatra — Gonds
3. Wari-Warkari — Santhals
Which of the pairs given above is/are correctly matched?
(a) 1 only
(b) 2 and 3 only
(c) 1 and 3 only
(d) None of the above

Ans. A
Exp: Nanda Devi Raj Jaat Yatra is a festival of Gharwali and kumauoni people in Uttarakhand
Wari Warkari or Varkari(Pandharpur Wari) is a Maharashtrian festival celebrated by Marathas.

96. Which of the following practices can helpin water conservation in agriculture ?
1. Reduced or zero tillage of the land
2. Applying gypsum before irrigating the field
3. Allowing crop residue to remain in the field
Select the correct answer using the code given below :
(a) 1 and 2 only
(b) 3 only
(c) 1 and 3 only
(d) 1, 2 and 3

Ans. D

97. Consider the following statements:
The nation-wide ‘Soil Health Card Scheme’ aims at
1. expanding the cultivable area under irrigation.
2. enabling the banks to assess the quantum of loans to be granted to farmerson the basis of soil quality.
3. checking the overuse of fertilizers in farmlands.
Which of the above statements is/are correct ?
(a) 1 and 2 only
(b) 3 only
(c) 2 and 3 only
(d) 1, 2 and 3

Ans. B

Source: http://iasscore.in/upsc-prelims/agriculture-schemes

98. Consider the following pairs :
Commonly used materials / Unwanted or controversial chemicals likely to be found in them

1. Lipstick — Lead
2. Soft drinks — Brominated vegetable oils
3. Chinesefast food — Monosodiumglutamate
Which of the pairs given above is/are correctly matched ?
(a) 1 only
(b) 2 and 3 only
(c) 1 and 3 only
(d) 1, 2 and 3

Ans. C
Exp: Brominated vegetable oils are not used in soft drink manufacturing

99. Organic Light Emitting Diodes (OLEDs) are used to create digital display in many devices. What are the advantages of OLED displays over Liquid Crystal displays?
1. OLED displays can be fabricated on flexible plastic substrates.
2. Roll-up displays embedded in clothing can be made using OLEDs.
3. Transparent/displays are possible using OLEDs
Select the correct answer using the code givenbelow :
(a) 1 and 3 only
(b) 2 only
(c) 1, 2 and 3
(d) None of -the above statements is correct

Ans. C

100. Which of the following is/are famous for Sun temples?
1. Arasavalli
2. Amarakantak
3. Omkareshwar
Select the correct answer using the code given below :
(a) 1 only
(b) 2 and 3 only
(c) 1 and 3 only
(d) 1, 2 and 3

Ans. A

Note: In some * marked answers, explanation will be given later…

IAS Score

How to scan the NEWSPAPER for Civil Services Exam?

  • by IAS Score

A day for IAS aspirant usually starts by reading newspaper and scanning the important news. Few jot down important points in the register; few underline the points and paste cuttings in the register; and so on.

However, have you ever noticed that when exam approaches most of the aspirants fail to even revise that notes, not able to co-relate them and end up reading the ready-made materials which become hard to grasp in the last minute.

Why is this so? What’s the fault in the methodology?  Is there any one best method for preparing current affairs from the newspaper?

As all human beings are different from the other, definitely their learning and grasping power will be different and hence there cannot be any one best method.

Hence hereby, sharing one of the methodologies for scanning Newspaper:

1. One should be aware about the UPSC syllabus as it will help in scanning and analysing the important news.

2. Firstly, just go through the headlines in the morning to remain up-to-date and further read it for the civil services preparation in the evening. It is because till evening more content related to the important topics will be available under different heads.

3. Try to cover one newspaper only, not more than that. As most of the news will be similar. For editorials refer different newspapers as it will help in developing the thinking line.
I will suggest reading Indian Express and Mint newspapers.
In Indian Express the editorials and coverage of International News is much better. Whereas in Mint, the language is lucid, technical jargons are less and quality of content is good.

4. While reading newspaper note down the underline issue rather than the headline.

For example: there was news last month “Election Commission seeks funds for Paper Trail Units”.  The news simply states about the funds and how much they want and in brief about Paper Trail Units. If an aspirant is reading this, then in general he will note down the news in 3-4 lines. But for the civil services preparation this news can be utilised as
a) Election commission and its role in free and fair election
b) About Paper Trial Units and its working?
c) Will it be able to make EVM more tamper-proof?
d) Importance of fair and free election for maintaining democracy and so on.

5. When an aspirant try to list down different dimensions associated with the particular news the analytical skill improves and he can design material with interdisciplinary approach. At first it looks a tedious task but if you note down important topics of one day, it will not be more than 2 or 3.

6. Note down important points under each head. Make different registers for different subjects/GS papers so that before the Mains examination of particular subject, it can be revised easily. This will also be helpful in connecting different editorials at one place as each editorial display different dimension.
                                   Be an active reader rather than passive reader.

7. There are number of news that get stretch to days, so while preparing notes leave 2-3 pages, so that if new dimension come up in news it can be added and co-related easily. If it remains scattered in your notebook, revision cannot be done effectively.

8. Avoid reading the politics news unless it is related to governance as:
a) Concept of freebies politics
b) Issue related to the Separation of Power
c) Logjam between Executive and Judiciary … overall the recognition of underline issue is must.

9. If one come across some particular incident as in article “The groundwater beneath their feet” http://www.thehindu.com/news/national/the-groundwater-beneath-their-feet/article17321183.ece, it is particularly related to the incident in Tamil Nadu but the issue can be “Impact of industrial pollution on water bodies”. The example can be quoted in Mains answer. So the case studies should be noted down.

10. For the prelims, the terms in news should be noted down carefully because questions are generally related to the terminologies. Apart from it the reports, committees, commissions and schemes etc. should also be noted down.

As stated earlier, everyone has different learning pattern so adopt this methodology for at least 10 days. If it works, try to incorporate it with flexibility according to your timings, number of newspaper, etc.

To conclude, the syllabus of Civil Services Examination is very vast and it is humanly not possible to cover all and be master in all. The important thing is that one should be able to differentiate between – “What to read and not to read”.

MAYURI KHANNA

Avatar

UPSC Prelims Answer Key 2018 with Explanation

  • by

Here, we are providing the UPSC Prelims Answer Key 2018 for GS Paper 1 with detailed explanations for the aspirants. This will help the candidates in tallying the answers to know their scores and start preparation for Mains freely.

46 questions of UPSC GS Paper 1 have been from our Target PT programme on our website and PT Test Series 2018. The links and questions have been provided with the respective UPSC questions.

 

 

We are uploading here the UPSC Prelims Answer Key: (Set – B)

1. Which of the following led to the introduction of English Education in India?
1. Charter Act of 1813
2. General Committee of Public Instruction, 1823
3. Orientalist and Anglicist Controversy
Select the correct answer using the code given below:
(a) 1 and 2 only
(b) 2 only
(c) 1 and 3 only
(d) 1, 2 and 3
Ans: d
Exp: • The East India Company began to adopt a dual policy in the sphere of education. It discouraged the prevalent system of oriental education and gave importance to western education and English language.
• The Charter Act of 1813 adopted a provision to spend one lakh rupees per annum for the spread of education in India.
• Although there was a prolonged debate pertaining to education during the course of a general discussion on the Act of 1813 in the British Parliament, yet the matter continued to generate debate for the next 20 years. Consequently, not even a single penny out of the allocated funds could be spent on education.
• The contemporary British scholars were divided into two groups on the issue of development of education in India. One group, called the Orientalists, advocated the promotion of oriental subjects through Indian languages. The other group, called the Anglicists, argued the cause of western sciences and literature in the medium of English language.
• In 1829, after assuming the office of the Governor-General of India, Lord William Bentinck, emphasized on the medium of English language in Indian education.
• In the beginning of 1835, the 10 members of the General Committee of Public Instruction were clearly divided into two equal groups.
• Five members including the Chairman of the committee Lord Macaulay were in favour of adopting English as medium of public instruction whereas the other five were in favour of oriental languages.
• The stalemate continued till 2 February 1835 when the Chairman of the committee, Lord Macaulay announced his famous Minute advocating the Anglicist point of view.
• Consequently, despite fierce opposition from all quarters, Bentinck got the resolution passed on 7 March 1835 which declared that henceforth, government funds would be utilized for the promotion of western literature and science through the medium of English language.

Source: http://www.iasscore.in/upsc-prelims/1857-revoltevolution-of-education-of-press-administration-before-1857

 

2. Which one of the following is an artificial lake?
(a) Kodaikanal (Tamil Nadu)
(b) Kolleru (Andhra Pradesh)
(c) Nainital (Utrarakhand)
(d) Renuka (Himachal Pradesh)
Ans-a
Exp-Kodaikanal Lake, also known as Kodai Lake is a manmade lake located in the Kodaikanal city in Dindigul district in Tamil Nadu, India. Sir Vere Henry Levinge, the then Collector of Madurai, was instrumental in creating the lake in 1863, amidst the Kodaikanal town which was developed by the British and early missionaries from USA. The lake is said to be Kodaikanal’s most popular geographic landmark and tourist attraction.

 

3. With reference to Pradhan Mantri Kaushal Vikas Yojana, consider the following statements:
1. It is the flagship scheme of the Ministry of Labour and Employment.
2. It, among other things, will also impart training in soft skills, entrepreneurship, financial and digital literacy.
3. It aims to align the competencies of the unregulated workforce of the country to the National Skill Qualification Framework.
Which of the statements given above is/are correct?
(a) 1 and 3 only
(b) 2 only
(c) 2 and 3 only
(d) 1, 2 and 3
Ans: c
Exp: Pradhan Mantri Kaushal Vikas Yojana (PMKVY) is the flagship scheme of the Ministry of Skill Development & Entrepreneurship (MSDE). The objective of this Skill Certification Scheme is to enable a large number of Indian youth to take up industry-relevant skill training that will help them in securing a better livelihood. Individuals with prior learning experience or skills will also be assessed and certified under Recognition of Prior Learning (RPL). Under this Scheme, Training and Assessment fees are completely paid by the Government.
Pradhan Mantri Kaushal Vikas Yojana (PMKVY)
Approved for another four years (2016-2020) to benefit 10 million youth. Pradhan Mantri Kaushal Vikas Yojana (PMKVY) is the flagship scheme of the Ministry of Skill Development & Entrepreneurship (MSDE).
The objective of this Skill Certification Scheme is to enable a large number of Indian youth to take up industry-relevant skill training that will help them in securing a better livelihood. Individuals with prior learning experience or skills will also be assessed and certified under Recognition of Prior Learning (RPL). Under this Scheme, Training and Assessment fees are completely paid by the Government.
Key Components of the Scheme:
1. Short Term Training
The Short Term Training imparted at PMKVY Training Centres (TCs) is expected to benefit candidates of Indian nationality who are either school/college dropouts or unemployed. Apart from providing training according to the National Skills Qualification Framework (NSQF), TCs shall also impart training in Soft Skills, Entrepreneurship, Financial and Digital Literacy. Duration of the training varies per job role, ranging between 150 and 300 hours. Upon successful completion of their assessment, candidates shall be provided placement assistance by Training Partners (TPs). Under PMKVY, the entire training and assessment fees are paid by the Government. Payouts shall be provided to the TPs in alignment with the Common Norms. Trainings imparted under the Short Term Training component of the Scheme shall be NSQF Level 5 and below.
2. Recognition of Prior Learning
Individuals with prior learning experience or skills shall be assessed and certified under the Recognition of Prior Learning (RPL) component of the Scheme. RPL aims to align the competencies of the unregulated workforce of the country to the NSQF. Project Implementing Agencies (PIAs), such as Sector Skill Councils (SSCs) or any other agencies designated by MSDE/NSDC, shall be incentivized to implement RPL projects in any of the three Project Types (RPL Camps, RPL at Employers Premises and RPL centres). To address knowledge gaps, PIAs may offer Bridge Courses to RPL candidates.

Source: http://iasscore.in/ias-prelims/skill-development-schemes

 

4. In 1920, which of the following changed its name to “Swarajya Sabha”?
(a) All India Home Rule League
(b) Hindu Mahasabha
(c) South Indian Liberal Federation
(d) The Servants of India Society
Ans: a

 

5. Which among the following events happened earliest?
(a) Swami Dayanand established Arya Samaj.
(b) Dinabandhu Mitra wrote Neeldarpan.
(c) Bankim Chandra Chattopadhyay wrote Anandmath.
(d) Satyendranath Tagore became the first India to succeed in the Indian Civil Services Examination.
Ans: b
Exp: Arya Samaj is an Indian Hindu reform movement that promotes values and practices based on the belief in the infallible authority of the Vedas. The samaj was founded by the sannyasi Dayananda Saraswati on 7 April 1875
Nil Darpan is a Bengali play written by Dinabandhu Mitra in 1858–1859. The play was published from Dhaka in 1860
Anandamath is a Bengali fiction, written by Bankim Chandra Chattopadhyay and published in 1882.
Satyendranath tagore(first to clear ICS) was selected for the Indian Civil Servicein June, 1863.

 

6. Which of the following is/are the possible consequence/s of heavy sand mining in riverbeds?
1. Decreased salinity in the river
2. Pollution of groundwater
3. Lowering of the water-table
Select the correct answer using the code given below:
(a) 1 only
(b) 2 and 3 only
(c) 1 and 3 only
(d) 1, 2 and 3
Ans: b
Exp: Sand Mining and impact on environment

Sand Mining is an activity referring to the process of the actual removal of sand from the foreshore including rivers, streams and lakes. Sand is mined from beaches and inland dunes and dredged from ocean beds and river beds. A related process is the mining of mineral sands, such as mineral deposits like diamond, gold and silver. These minerals typically occur combined with ordinary sand. The sand is dug up, the valuable minerals are separated in water by using their different density, and the remaining ordinary sand is re-deposited.

Excessive in-stream sand-and-gravel mining causes the degradation of rivers. In-stream mining lowers the stream bottom, which may lead to bank erosion. Depletion of sand in the streambed and along coastal areas causes the deepening of rivers and estuaries, and the enlargement of river mouths and coastal inlets. It may also lead to saline-water intrusion from the nearby sea. The effect of mining is compounded by the effect of sea level rise. Any volume of sand exported from streambeds and coastal areas is a loss to the system. It is also a threat to bridges, river banks and nearby structures. Sand mining also affects the adjoining groundwater system and the uses that local people make of the river.

In-stream sand mining results in the destruction of aquatic and riparian habitat through large changes in the channel morphology. Impacts include bed degradation, bed coarsening, lowered water tables near the streambed, and channel instability. These physical impacts cause degradation of riparian and aquatic biota and may lead to the undermining of bridges and other structures. Continued extraction may also cause the entire streambed to degrade to the depth of excavation. Sand mining generates extra vehicle traffic, which negatively impairs the environment. Where access roads cross riparian areas, the local environment may be impacted.

In-stream mining can have other costly effects such as many hectares of fertile streamside land are lost annually, as well as valuable timber resources and wildlife habitats in the riparian areas. Degraded stream habitats result in loss of fisheries productivity, biodiversity, and recreational potential. Severely degraded channels may lower land and aesthetic values.

Further all species require specific habitat conditions to ensure long-term survival. Native species in streams are uniquely adapted to the habitat conditions that existed before humans began large-scale alterations. These have caused major habitat disruptions that favored some species over others and caused overall declines in biological diversity and productivity. In most streams and rivers, habitat quality is strongly linked to the stability of channel bed and banks. Unstable stream channels are inhospitable to most aquatic species.

Factors that increase or decrease sediment supplies often destabilize bed and banks and result in dramatic channel readjustments. For example, human activities that accelerate stream bank erosion, such as riparian forest clearing or in-stream mining, cause stream banks to become net sources of sediment that often have severe consequences for aquatic species. Anthropogenic activities that artificially lower stream bed elevation cause bed instabilities that result in a net release of sediment in the local vicinity. Unstable sediments simplify and, therefore, degrade stream habitats for many aquatic species. Few species benefit from these effects.

The most important effects of in-stream sand mining on aquatic habitats are bed degradation and sedimentation, which can have substantial negative effects on aquatic life. The stability of sand-bed and gravel-bed streams depends on a delicate balance between stream-flow, sediment supplied from the watershed, and channel form. Mining-induced changes in sediment supply and channel form disrupt channel and habitat development processes. Furthermore, movement of unstable substrates results in downstream sedimentation of habitats. The affected distance depends on the intensity of mining, particles sizes, stream flows, and channel morphology.

The complete removal of vegetation and destruction of the soil profile destroys habitat both above and below the ground as well as within the aquatic ecosystem, resulting in the reduction in faunal populations.

Also, Channel widening shallows the streambed, producing braided flow or subsurface inter-gravel flow in riffle areas, hindering movement of fishes between pools. Channel reaches become more uniformly shallow as deep pools fill with gravel and other sediments, reducing habitat complexity, riffle-pool structure, and numbers of large predatory fishes.

Apart from it, sand mining transforms the riverbeds into large and deep pits; as a result, the groundwater table drops leaving the drinking water wells on the embankments of these rivers dry. Bed degradation from in-stream mining lowers the elevation of stream flow and the floodplain water table which in turn can eliminate water table-dependent woody vegetation in riparian areas, and decrease wetted periods in riparian wetlands. For locations close to the sea, saline water may intrude into the fresh water body.

In-stream sand mining activities will have an impact upon the river’s water quality. Impacts include increased short-term turbidity at the mining site due to resuspension of sediment, sedimentation due to stockpiling and dumping of excess mining materials and organic particulate matter, and oil spills or leakage from excavation machinery and transportation vehicles.

Increased riverbed and bank erosion increases suspended solids in the water at the excavation site and downstream.

Suspended solids may adversely affect water users and aquatic ecosystems. The impact is particularly significant if water users downstream of the site are abstracting water for domestic use. Suspended solids can significantly increase water treatment costs.

Source: http://iasscore.in/national-issues/sand-mining-and-its-impact

 

7. With reference to agricultural soils, consider the following statements:
1. A high content of organic matter in soil drastically reduces its water holding capacity.
2. Soil does not play any role in the sulphur cycle.
3. Irrigation over a period of time can contribute to the salinization of some agricultural lands.
Which of the statements given above is/are correct?
(a) 1 and 2 only
(b) 3 only
(c) 1 and 3 only
(d) 1, 2 and 3
Ans: b
Exp: “Each 1 percent increase in soil organic matter helps soil hold 20,000 gallons more water per acre.”
• Sulphur Cycle
Sulphur cycle, circulation of sulfur in various forms through nature. Sulphur is key to protein structure and is released to the atmosphere by the burning of fossil fuels. Sulphur occurs in all living matter as a component of certain amino acids. It is abundant in the soil in proteins and, through a series of microbial transformations, ends up as sulphates usable by plants.
Sulphur-containing proteins are degraded into their constituent amino acids by the action of a variety of soil organisms. The sulphur of the amino acids is converted to hydrogen sulphide (H2S) by another series of soil microbes. In the presence of oxygen, H2S is converted to sulfur and then to sulphate by sulfur bacteria. Eventually the sulfate becomes H2S.
Hydrogen sulphide rapidly oxidizes to gases that dissolve in water to form sulphurous and sulphuric acids. These compounds contribute in large part to the “acid rain” that can kill sensitive aquatic organisms and damage marble monuments and stone buildings.
Thus cycle can be divided as:
· Sulphur Cycle in Soils
Sulphur enters the trophic cycle in terrestrial plants via root adsorption in the form of inorganic sulphates (e.g., calcium sulphate, sodium sulphate) or by direct assimilation of amino acids released in the decomposition of dead or excreted organic matter. Bacterial and fungal (Aspergillus and Neurospora) mineralization of the organic sulphhydryl in amino acids followed by oxidation results in sulphate; this adds to the sulphate pool for root adsorption.
· Sulphur Cycle in Atmosphere
Sulphur in the atmosphere comes from several different sources: decomposition and/or combustion of organic matter, combustion of fossil fuels, and ocean surfaces and volcanic eruptions. The most prevalent form of sulphur entering the atmosphere is sulphur dioxide (SO2). It, along with other atmospheric forms such as elemental sulphur and hydrogen sulphide, is oxidized to sulphur trioxide (SO3), which combines with water to form sulphuric acid (H2SO4), leading to acid rain.
Atmospheric sulphur, largely in the form of sulphuric acid, is removed by two general processes: rainout, which includes all processes within clouds that result in removal; and washout, which is the removal by precipitation below the clouds. Depending on the amount of the various sulphur compounds available to form the sulphuric acid, the degree of acidity can be strong enough to ap-proximate that of battery acid. Atmospheric inputs of sulphuric acid provide the dominant source of both hydrogen ions (H+) for cation replacement.
· Sulphur in Sediments
The sedimentary aspect of the cycle involves the precipitation of sulphur in the presence of such cations as iron (Fe) and calcium (Ca) as highly insoluble ferrous sulphide (FeS) and ferric sulphide (Fe2S3, pyrite) or relatively insoluble calcium sulphate (CaSO4).
The oxidation of sulphides in marine sediments is a key process, though poorly understood.

Salinity also Caused due to excessive irrigation in dry conditions which promotes capillary action

Source: http://www.iasscore.in/upsc-prelims/nitrogen-cyclesulphur-cycle-hydrological-cycle

 

8. The Partnership for Action on Green Economy (PAGE), a UN mechanism to assist countries transition towards greener and more inclusive economies, emerged at
(a) The Earth Summit on Sustainable Development 2002, Johannesburg
(b) The United Nations Conference on Sustainable Development 2012, Rio de Janeiro
(c) The United Nations Framework Convention on Climate Change 2015, Paris
(d) The World Sustainable Development Summit 2016, New Delhi
Ans: b
Exp: Sustainable development has been the overarching goal of the international community since the UN Conference on Environment and Development (UNCED) in 1992. Amongst numerous commitments, the Conference called upon governments to develop national strategies for sustainable development, incorporating policy measures outlined in the Rio Declaration and Agenda 21. Despite the efforts of many governments around the world to implement such strategies as well as international cooperation to support national governments, there are continuing concerns over global economic and environmental developments in many countries. These have been intensified by recent prolonged global energy, food and financial crises, and underscored by continued warnings from global scientists that society is transgressing a number of planetary boundaries or ecological limits.
With governments today seeking effective ways to lead their nations out of these related crises whilst also taking into account these planetary boundaries, green economy (in its various forms) has been proposed as a means for catalysing renewed national policy development and international cooperation and support for sustainable development. The concept has received significant international attention over the past few years as a tool to address the 2008 financial crisis as well as one of two themes for the 2012 UN Conference on Sustainable Development (Rio+20). This has resulted in a rapidly expanding literature including new publications on green economy from a variety of influential international organisations, national governments, think tanks, experts, non-government organisations and others.
Governments agreed at Rio+20 to frame the green economy as an important tool for sustainable development; one that is inclusive and can drive economic growth, employment, and poverty eradication, whilst maintaining the healthy functioning of the Earth’s ecosystems. Importantly, the outcome document also recognises that capacity building, information exchange and experience sharing will be critical for implementing green economy policies.
Recent initiatives on green economy or green growth by the United Nations Environment Program (UNEP), the UN Department of Economic and Social Affairs (UNDESA), the United Nations Conference on Trade and Development (UNCTAD), the International Labour Organisation (ILO), the World Bank, the Organisation for Economic Cooperation and Development (OECD), the Global Green Growth Institute (GGGI), the Partnership for Action on Green Economy (PAGE), the Green Growth Knowledge Platform (GGKP) the Green Economy Coalition

 

9. “3D printing” has applications in which of the following?
1. Preparation of confectionery items
2. Manufacture of bionic ears
3. Automotive industry
4. Reconstructive surgeries
5. Data processing technologies
Select the correct answer using the code given below:
(a) 1, 3 and 4 only
(b) 2, 3 and 5 only
(c) 1 and 4 only
(d) 1, 2, 3, 4 and 5
Ans: d
Exp: 1 Manufacturing applications
1.1 Cloud-based additive manufacturing
1.2 Mass customization
1.3 Rapid manufacturing
1.4 Rapid prototyping
1.5 Research
1.6 Food
1.7 Agile tooling

2 Medical applications
2.1 Bio-printing
2.2 Medical devices
2.3 Pills

3 Industrial applications
3.1 Apparel
3.2 Industrial art and jewelry
3.3 Automotive industry
3.4 Construction
3.5 Firearms
3.6 Computers and robots
3.7 Soft sensors and actuators
3.8 Space
4 Sociocultural applications
4.1 Art and jewelry
4.2 3D selfies
4.3 Communication
4.4 Domestic use
4.5 Education and research
4.6 Environmental use
4.7 Cultural heritage
4.8 Specialty materials

Source: http://www.iasscore.in/upsc-prelims/3-d-printing

 

10. Consider the following statements:
1. The Barren Island volcano is an active volcano located in the Indian territory.
2. Barren Island lies about 140 km east of Great Nicobar.
3. The last time the Barren Island volcano erupted was in 1991 and it has remained inactive since then.
Which of the statements given above is/are correct?
(a) 1 only
(b) 2 and 3
(c) 3 only
(d) 1 and 3
Ans: a
Exp: According to scientists from Goa based National Institute of Oceanography (NIO), India’s only live volcano at Barren Island in the Andaman and Nicobar has become active again
After lying dormant for 150 years, Barren Island volcano had erupted in 1991 and since then it is showing sporadic activity. Now it is erupting in small episodes of five to 10 minutes.
The Barren Islands are located around 140 km northeast of the Andamans capital city Port Blair.

 

GSSCORE QUESTION

Which of the following statements are true related to major island groups in India?

1.   Andaman and Nicobar island group is an elevated portion of submarine mountains.

2.   Entire island group of Lakshadweep and Minicoy is built of coral deposits.

3.   Barren island is the only active volcano in India situated in the Nicobar Islands.

Codes:

(a)   1 & 3

(b)   2 & 3

(c)   1 & 2

(d)   All

Ans: C

Exp: It is located in Andaman Islands.

Source: http://iasscore.in/upsc-pt-practice-test-rank.php?test_id=134

 

11. Why is a called Prosopis julifloru often mentioned is news?
a) Its extract is widely used in cosmetics.
b) It tends to reduce the biodiversity in the area in which it grows.
c) Its extracts is used in the synthesis of pesticides.
d) None of the above
Ans: b
Exp: Prosopis juliflora (P juliflora), an exotic tree, is one of the top invaders in India. A native of South and Central America, it was introduced in India to meet the fuel wood requirement of the rural poor and to restore degraded lands. It tends to reduce the biodiversity in the area in which it grows.

 

GSSCORE QUESTION

Recently Madras High Court has ordered the Tamil Nadu government to enact a law on prohibition of seemaikaruvelam trees (prosopisjuliflora). What is the reason behind this?

(a)   These trees release toxic chemicals which are causing health problems in the state.

(b)   These trees are against the symbol and tenets of Vashnaivism sect in the state.

(c)   These trees suck lot of water by invading into the water bodies.

(d)   These trees are the home to various pathogens and insects.

Ans: C

(PT Current Affairs Test 1)

 

12. Consider the following statements
1. Most of the world’s coral reefs are in tropical waters.
2. More than one third of the world’s coral reefs are located in the territories of Australia, Indonesia and Philippines.
3. Coral reefs host far more number of animal phyla than those hosted by tropical rainforests.
Which of the statements given above is/are correct?
a) 1 and 2 only
b) 3 only
c) 1 and 3 only
d) 1,2 and 3
Ans: d
Exp: Coral reefs are found in circum-tropical shallow tropical waters along the shores of islands and continents. The reef substrate is mainly composed of calcium carbonate from living and dead corals. Many other invertebrates, vertebrates, and plants live in close association to the corals, with tight resource coupling and recycling, allowing coral reefs to have extremely high productivity and biodiversity, such that they are referred to as ‘the Tropical Rainforests of the Oceans’.
Coral reefs are believed by many to have the highest biodiversity of any ecosystem on the planet—even more than a tropical rainforest.
Major Regions of Coral Reef Development
Globally, three major regions of coral reef development are generally recognized, each with a somewhat distinctive biota. These are:
(1) The Indo-Pacific – Includes most of the Indian Ocean (excluding the Red Sea), and the western Pacific.
(2) The Wider Caribbean(tropical western Atlantic) – Includes Florida, The Bahamas, Caribbean Sea proper, and coastal waters off northeastern S. America.
(3) The Red Sea
Thus Australia, Indonesia and Philippines cover one-third area.
Source: http://www.iasscore.in/upsc-prelims/estuaries-mangroves-coral-polyps

 

13. “momentum for change : climate neutral now” is an initiative launched by
a) The intergovernmental panel on climate change
b) The UNEP secretariat
c) The UNFCCC secretariat
d) The world meteorological organization
Ans: c
Exp: Momentum for Change: Climate Neutral Now brings together two of the secretariat’s flagship activities that recognize leadership in tackling climate change by non-Party stakeholders.
It has been launched by the (UNFCCC) United Nations Climate Change secretariat has launched a new initiative that will showcase efforts by individuals, companies and governments that are achieving real results in transitioning to climate neutrality.

 

14. With reference to educational institutions during colonial rule in India, consider the following pairs:
Institution Founder
1. Sanskrit college at Banaras William Jones
2. Calcutta Madarsa Warren Hastings
3. Fort William college Arthur Wellesley
Which of the pairs given above is/are correct?
a) 1 and 2
b) 2 only
c) 1 and 3
d) 3 only
Ans: b
Exp: Sanskrit college: Jonathan Duncan
Calcutta madarsa: Warren Hastings
Fort William College: Richard Wellesly

Source: http://www.iasscore.in/upsc-prelims/1857-revoltevolution-of-education-of-press-administration-before-1857

 

15.Consider the following pairs :
Region sometimes mentioned in news country
1. Catalonia – Spain
2. Crimea – Hungary
3. Mindanao – Philippines
4. Oromia – Nigeria
Which of the pairs given above are correctly matched?
a) 1, 2 and 3
b) 3 and 4 only
c) 1 and 3 only
d) 2 and 4 only
Ans: c
Exp: Catalonia: Spain
Crimea: Ukraine
Mindanao: Philippines
Oromia: Ethiopia

 

16. Which one of the following statements correctly describes the meaning of legal tender money?
a) The money which is tendered in courts of law to defray the fee of legal cases.
b) The money which a creditors is under compulsion to accept in settlement of his claims.
c) The bank money in the forms of cheque, drafts, bills of exchange, etc.
d) The metallic money in circulation in a country.
Ans: b
Exp: There is no complete definition in the options, so we need to choose most suitable, which is b, as legal tender is acceptable by law, which no one can refuse to accept.

Source: http://www.iasscore.in/upsc-prelims/money-inflation

 

17. if a commodity is provided free to the public by the government, then
a) The opportunity cost is zero.
b) The opportunity cost is ignored.
c) The opportunity cost is transferred from the consumers of the product to the tax-paying public.
d) The opportunity cost is transferred from the consumers of the product to the government.
Ans: d *
Exp: Opportunity cost is the cost which could have been earned from second best investment option. For free goods, the opportunity cost is zero for the person consuming it, however, it is not so for the provider of that good. The choice of spending on various alternatives is available with government and not tax payers. Thus, it is transferred to government.

 

18. Increase in absolute and per capital real GNP do not connote a higher level of economics development, if
a) Industrial output fails to keep pace with agriculture output.
b) Agriculture output fails to keep pace with industrial output.
c) Poverty and unemployment increase.
d) Imports grow faster than exports.
Ans: c
Exp: If gains of increase in per capita income are grabbed by a small section of society, then economic growth will not lead to economic development. For example – Libya

 

GSSCORE QUESTION

Choose the correct statement(s) with regard to GNP per capita measurement:

1.   GNP per capita is used for inter-country comparison of overall development.

2.   It reflects the pattern of distribution of income within the country hence, can be used as a measure of poverty.

Which of the above statements is/are correct?

(a)   Only 1

(b)   Only 2

(c)   Both

(d)   None

Ans. A

Exp: It gives average income and does not reflect the pattern of income within the country hence not a satisfactory measure of poverty.

(PT crash course Economy test 1, question 26)

 

19. Consider the following statements.
Human capital formation as a concept is better explained in terms of a process which enables.
a) Individuals of a country to accumulate more capital.
b) Increasing the knowledge, skill levels and capacities of the people of the country.
c) Accumulation of tangible wealth.
d) Accumulation of intangible wealth.
Which of the statements given above is/are correct?
1) 1 and 2
2) 2 only
3) 2 and 4
4) 1,3 and 4
Ans: c
Exp: While 2 is very meaning of human capital accumulation, 4 also increases due to 2 only, for example, patents, copyrights, etc.

 

20.Despite being a high saving economy, capital formation may not result in significant increase in output due to
a) Weak administrative machinery
b) Illiteracy
c) High population density
d) High capital output ratio
Ans: d
Exp: If a country has poor technology and low efficiency, even high savings, will lead to low economic growth.

 

21. After the Santhal uprising subsided, what was/were the measure/measures taken by the colonial government?
1) The territories called ‘santhal paraganas were created.
2) It became illegal for a santhal to transfer land to a non-santhal.
Select the correct answer using the code given below:
a) 1 only
b) 2 only
c) Both 1 and 2
d) Neither 1 nor 2
Ans: C
Exp:. Santhal Pargana District was created in 1885 after partition of Bhagalpur and Birbhum. An important reason behind the creation of Santhal Pargana was Santhal Mutiny in 1854-55.

 

22.Economically, one of the results of the British rule in India on the 19th century was the
a) Increase in the export of Indian handicrafts
b) Growth in the number of Indian owned factories.
c) Commercialization of Indian agriculture.
d) Rapid increase in the urban population
Ans: c
Exp: Commercialization of Indian agriculture started post 1813 when the industrial revolution in England gained pace.

Source: http://iasscore.in/upsc-prelims/administration-before-1857-economic-policies

 

23. If the president of India exercise his power as provided under article 356 of the constitutional in respect of a particular state, then
a) The assemble of the state is automatically dissolved.
b) The powers of the legislature of that state shall be exercisable by or under the authority of the parliament.
c) Article 19 is suspended in that state.
d) The president can make laws relating to that state.
Ans: d
Exp: When the President’s Rule is imposed in a State the Parliament can delegate the power to make laws for the state to the President or to any other authority specified by him in this regard.

Source: http://iasscore.in/upsc-prelims/emergency

 

24.Consider the following pairs :
Crafts Heritage of
1. Puthukkuli shawls Tamil Nadu
2. Sujni embroidery Maharashtra
3. Uppada jamdani saris Karnataka
Which of the pairs given above is/are correct?
a) 1 only
b) 1 and 2
c) 3 only
d) 2 and 3
Ans: a
Exp: Sujni Embroidery is of Bihar
Uppada Jamdani Saris is of Andhra Pradesh

 

25. In which of the following areas can GPS technology be used?
1) Mobile phone operations
2) Banking operations
3) Controlling the power grids
Select of the correct answer using the code below :
a) 1 only
b) 2 and 3 only
c) 1 and 3 only
d) 1,2 and 3
Ans: d
Exp: It is a global navigation satellite system that provides geolocationand time information to a GPS receiver anywhere on or near the Earth where there is an unobstructed line of sight to four or more GPS satellites.
Many civilian applications use one or more of GPS’s three basic components: absolute location, relative movement, and time transfer.
• Agriculture
• Astronomy
• Automated vehicle
• Cartography
• Cellular telephony.
• Clock synchronization
• Disaster relief/emergency services
• Radio occultation for weather and atmospheric science applications.
• Fleet tracking: used to identify, locate and maintain contact reports with one or more fleet vehicles in real-time.
• Geofencing
• Geotagging
• GPS aircraft tracking
• GPS for mining
• GPS data mining.
• Navigation: navigators value digitally precise velocity and orientation measurements.
• Surveying: surveyors use absolute locations to make maps and determine property boundaries.

Source: http://www.iasscore.in/upsc-prelims/spatial-technology

 

26. Consider the following statements:
1. The Reserve Bank of India manages and services Government of India Securities but not any State Government Securities.
2. Treasury bills are issued by the Government of India and there are no treasury bills issued by the State Governments.
3. Treasury bills offer are issued at a discount from the par value.
Which of the statements given above is/are correct?
(a) 1 and 2 only
(b) 3 only
(c) 2 and 3 only
(d) 1, 2 and 3
Ans: c
Exp: Treasury Bills are issued only by the central government in India. The State governments do not issue any treasury bills. Interest on the treasury bills is determined by market forces.
Treasury bills, or T-bills, are short-term debt instruments issued by the U.S Treasury. T-bills are issued for a term of one year of less. T-bills are considered the world’s safest debt as they are backed by the full faith and credit of the United States government.
Treasury bills offer are issued at a discount from the par value.

Source: http://www.iasscore.in/upsc-prelims/regulatory-bodies
http://iasscore.in/upsc-prelims/money-market-capital-marketinsurance

 

27. Consider the following statements:
1. The Earth’s magnetic field has reversed every few hundred thousand years.
2. When the Earth was created more than 4000 million years ago, there was 54% oxygen and no carbon dioxide.
3. When living organisms originated, they modified the early atmosphere of the Earth.
Which of the statements given above is/are correct?
(a) 1 only
(b) 2 and 3 only
(c) 1 and 3 only
(d) 1, 2 and 3

Ans: c
Exp: The Earth’s Magnetic field has reversed every few hundred thousand years. This has been proved through Sea-Floor spreading.
When Earth was created there was no oxygen in the atmosphere. Oxygen makes up about one-fifth the volume of Earth’¬s atmosphere today and is a central element of life as we know it. But that wasn’¬t always the case. Oxygen, although always present in compounds in Earth’¬s interior, atmosphere, and oceans, did not begin to accumulate in the atmosphere as oxygen gas (O2) until well into the planet’¬s history.
Carbon dioxide, water vapor, and methane played an important role in Earth’¬s subsequent development.
By 2.7 billion years ago, a new kind of life had established itself: photosynthetic microbes called cyanobacteria, which were capable of using the Sun’¬s energy to convert carbon dioxide and water into food with oxygen gas as a waste product. They lived in shallow seas, protected from full exposure to the Sun’¬s harmful radiation.
These organisms became so abundant that by 2.4 billion years ago the free oxygen they produced began to accumulate in the atmosphere.

Source: http://www.iasscore.in/upsc-prelims/other-geographical-facts
http://www.iasscore.in/upsc-prelims/basic-theoriesinterior-of-earthrocks

 

28. The terms ‘WannaCry, Petya and EternalBlue’ sometimes mentioned in the news recently are related to
(a) Exoplanets
(b) Cryptocurrency
(c) Cyber attacks
(d) Mini satellites

Ans: c
Exp: Wannacry, Petya and EternalBlue are relsted to cyber attacks. These are form of Ransonware.

Source: http://iasscore.in/ias-prelims/wannacry-malware

 

29. With reference to the circumstances in India agriculture, the concept of “Conservation Agriculture” assumes significance. Which of the following fall under the Conservation Agriculture?
1. Avoiding the monoculture practices
2. Adopting minimum tillage
3. Avoiding the cultivation of plantation crops
4. Using crop residues to cover soil surface
5. Adopting spatial and temporal crop sequencing/crop rotations
Select the correct answer using the code given below:
(a) 1, 3 and 4
(b) 2, 3, 4 and 5
(c) 2, 4 and 5
(d) 1, 2, 3 and 5
Ans: c
Exp: Conservation Agriculture is a set of soil management practices that minimize the disruption of the soil’s structure, composition and natural biodiversity. Despite high variability in the types of crops grown and specific management regimes, all forms of conservation agriculture share three core principles. These include:
o maintenance of permanent or semi-permanent soil cover (using either a previous crop residue or specifically growing a cover crop for this purpose);
o minimum soil disturbance through tillage (just enough to get the seed into the ground) ;
o regular crop rotations to help combat the various biotic constraints;
Conservation Agriculture also uses or promotes where possible or needed various management practices listed below:
o utilization of green manures/cover crops (GMCC’s) to produce the residue cover;
o no burning of crop residues;
o integrated disease and pest management;
o controlled/limited human and mechanical traffic over agricultural soils.

30. The term “sixth mass extinction/sixth extinction” is often mentioned in the news in the context of the discussion of
(a) Widespread monoculture practices in agriculture and large-scale commercial farming with indiscriminate use of chemicals in many parts of the world that may result in the loss of good native ecosystems.
(b) Fears of a possible collision of a meteorite with the Earth in the near future in the manner it happened 65 million years ago that caused the mass extinction of many species including those of dinosaurs.
(c) Large scale cultivation of genetically modified crops in many parts of the world and promoting their cultivation in other parts of the world which may cause the disappearance of good native crop plants and the loss of food biodiversity.
(d) Mankind’s over-exploitation/misuse of natural resources, fragmentation/loss of natural habitats, destruction of ecosystems, pollution and global climate change.
Ans: d
Exp: Earth is currently in the midst of what is being considered the 6th great mass extinction, or the Holocene extinction, or sometimes the Anthropocene extinction.
An increasing number of species is disappearing from the face of the earth due to the human activities. This man-made mass extinction represents a very severe depletion of biodiversity, particularly because it is occurring within a short period of time.

 

GSSCORE QUESTION

Consider the following statements with respect to the biodiversity extinction event:

1.   An extinction event is widespread and rapid decrease in biodiversity of the earth.

2.   Speciation is the rate which new species are formed, which contributes to the biodiversity.

3.   Extinction events are recorded through mass disappearance of fossil records, especially for marine organisms

4.   The cretaceous- tertiary mass extinction is famed for death of the dinosaurs.

Which of the above statements are correct?

(a)   1 and 3

(b)   1, 3 and 4

(c)   1 and 4

(d)   All

Ans. D

Exp: Extinction is every day. Mass extinction is not. An extinction event is widespread and rapid decrease in biodiversity of the earth. It occurs when prevailing rate of extinction far exceeds the background or natural rate of extinction .this rate is measured normally in number of species going extinct over a given period of time.

The opposite of extinction is speciation – the rate at which new species are formed, which contributes to the biodiversity.

Extinction events are recorded through mass disappearance of fossil records, especially for marine organisms, whose fossils are better preserved.

Scientists have recognized several mass extinction events in past 500 million years , out of which five are considered to major ones, these are:

•   End Ordovician – Silurian E E ( 450-440 million years )

•   Late Devonian E E( 375 MY)

•   Permian-Triassic E E( 250 MY)

•   Triassic-Jurassic E E ( 200 MY )

•   Cretaceous- tertiary E E( 66 MY) : It is famed for the death of the dinosaurs.

(Prelims 2018 PT test 13, question 27)

Source: http://www.iasscore.in/upsc-prelims/types-of-species

 

31. Consider the following events:
1. The first democratically elected communist party government formed in a State in India.
2. India’s then largest bank, ‘Imperial Bank of India’, was renamed ‘State Bank of India’.
3. Air India was nationalised and became the national carrier.
4. Goa became a part of independent India.
Which of the following is the correct chronological sequence of the above events?
(a) 4 – 1 – 2 – 3
(b) 3 – 2 – 1 – 4
(c) 4 – 2 – 1 – 3
(d) 3 – 1 – 2 – 4
Ans: b
Exp: The Kerala Legislative Assembly election of 1957 was the first assembly election in the Indian state of Kerala. The Communist Party of India won the election with 60 seats. The election led to the formation of first democratically elected communist government in India
The Government of India took control of the Imperial Bank of India in 1955, with Reserve Bank of India (India’s central bank) taking a 60% stake, renaming it the State Bank of India.
In 1953-Air India was nationalized
On December 19, 1961, Goa officially became part of India

 

32. Right to Privacy is protected as an intrinsic part of Right to Life and Personal Liberty. Which of the following in the Constitution of India correctly and appropriately imply the above statements?
(a) Article 14 and the provisions under the 42nd Amendment to the Constitution
(b) Article 17 and the Directive Principles of State Policy in Part IV
(c) Article 21 and the freedoms guaranteed in Part III
(d) Article 24 and the provisions under the 44th Amendment to the Constitution
Ans: c
Exp: Article 21- Protection of life and personal liberty No person shall be deprived of his life or personal liberty except according to procedure established by law.

 

GSSCORE QUESTION

Which of the following fundamental rights and their derivation from Constitutional Article is/are correctly matched?

1.   Right to timely medical treatment in government hospital – Article 21.

2.   Freedom of commercial advertisements – Article 19.

3.   Right to peaceful protest – Article 16.

4.   Right to privacy – Article 21.

Select the correct answer using the codes below:

(a)   Only 4

(b)   1, 2 and 4

(c)   1, 2, 3 and 4

(d)   2, 3 and 4

Ans. B

Exp: Right to peaceful protest is a fundamental right under Article 19.

(PT 2018 test 3, question 9)

Source: http://www.iasscore.in/upsc-prelims/basic-fundamental-rights-for-citizens-and-aliens-military-laws

 

33. Consider the following:
1. Areca nut
2. Barley
3. Coffee
4. Finger millet
5. Groundnut
6. Sesamum
7. Turmetic
The Cabinet Committee on Economic Affairs has announced the Minimum Support Price for which of the above?
(a) 1, 2, 3 and 7 only
(b) 2, 4, 5 and 6 only
(c) 1, 3, 4, 5 and 6 only
(d) 1, 2, 3, 4, 5, 6 and 7
Ans: b
Exp:
The MSP is announced by the Government of India for 25 crops currently at the beginning of each season viz. Rabi and Kharif. Following are the 25 crops covered by MSP:
Kharif Crops Rabi Crops
1 Paddy 15 Wheat
2 Jowar 16 Barley
3 Bajra 17 Gram
4 Maize 18 Masur (Lentil)
5 Ragi 19 Rapeseed/Mustard
6 Arhar(Tur) 20 Safflower
7 Moong 21 Toria
8 Urad Other Crops
9 Cotton 22 Copra
10 Groundnut 23 De-Husked Coconut
11 Sunflower Seed 24 Jute
12 Soyabeen Black 25 Sugarcane
13 Sesamum
14 Nigerseed

 

34. In which one of the following States in Pakhui Wildlife Sanctuary located?
(a) Arunachal Pradesh
(b) Manipur
(c) Meghalaya
(d) Nagaland
Ans: a
Exp: Pakhui Wildlife Sanctuary (862 km2, 92°36′ – 93°09’E and 26°54 – 27°16’N) lies in the foothills of the Eastern Himalaya in the East Kameng District of Arunachal Pradesh.

 

35. With reference to India’s satellite launch vehicles, consider the following statements:
1. PSLVs launch the satellites useful for Earth resources monitoring whereas GSLVs are designed mainly to launch communication satellites.
2. Satellites launched by PSLV appear to remain permanently fixed in the same position in the sky, as viewed from a particular location on Earth.
3. GSLV Mk III is a four-staged launch vehicle with the first and third stages using solid rocket motors; and the second and fourth stages using liquid rocket engines.
Which of the statements given above is/are correct?
(a) 1 only
(b) 2 and 3
(c) 1 and 2
(d) 3 only
Ans: a
Exp: PSLVs launch the satellites useful for Earth resources monitoring whereas GSLVs are designed mainly to launch communication satellites.
Satellites launched by GSLV appear to remain permanently fixed in the same position in the sky, as viewed from a particular location on Earth.
GSLV Mk III is a three-stage heavy lift launch vehicle developed by ISRO. The vehicle has two solid strap-ons, a core liquid booster and a cryogenic upper stage.
GSLV Mk III is designed to carry 4 ton class of satellites into Geosynchronous Transfer Orbit (GTO) or about 10 tons to Low Earth Orbit (LEO), which is about twice the capability of GSLV Mk II.
The two strap-on motors of GSLV Mk III are located on either side of its core liquid booster. Designated as ‘S200’, each carries 205 tons of composite solid propellant and their ignition results in vehicle lift -off . S200s function for 140 seconds. During strap-ons functioning phase, the two clustered Vikas liquid Engines of L110 liquid core booster will ignite 114 sec after lift -off to further augment the thrust of the vehicle. These two engines continue to function after the separation of the strap-ons at about 140 seconds after lift -off.

 

GSSCORE QUESTION

Consider the following statements about the recently launched GSLV Mark-III X:

1.   It uses cryogenic technology.

2.   It is only fuelled by solid propellant in two stages.

3.   It will help India become self-reliant in launching communication satellites.

Which of the above statements are correct?

(a)   1 and 2

(b)   1 and 3

(c)   2 and 3

(d)   All

Ans. B

Exp: Indian Space Research Organisation (ISRO’s) first sub-orbital flight and India’s latest generation launch vehicle- GSLV Mark-III X was successfully lifted off from SatishDhawan Space Centre, Sriharikota,Andhra Pradesh.

About GSLV-Mark III

•   The GSLV-Mark III is a three stage/engine launch vehicle.

•   Its first stage comprises two identical S-200 large solid boosters with 200 tonne solid propellant that are strapped on to the second stage, the L110 re-startable liquid stage.

•   The third stage/engine is the cryogenic which is more efficient as it provides more thrust for every kilogram of propellant burnt.

Thus, this successful launch will help India in perfecting the cryogenic engine technology and help to become self-reliant in launching communication satellites.

(PT 2018 test 9, question 32)

Source: http://iasscore.in/upsc-prelims/types-of-satellites-indian-space-programme

 

36. With reference to the governance of public sector banking in India, consider the following statements:
1. Capital infusion into public sector banks by the Government of India has steadily increased in the last decade.
2. To put the public sector banks in order, the merger of associate banks with the parent State Bank of India has been affected.
Which of the statements given above is/are correct?
(a) 1 only
(b) 2 only
(c) Both 1 and 2
(d) Neither 1 nor 2
Ans: b
Exp: After the Securitisation and Reconstruction of Financial Assets and Enforcement of Securities Interest Act, 2002 (also known as the SARFAESI Act) ,the government capital infusion steadily decreased until NPA problem introduced in 2015.

 

37. Consider the following items:
1. Cereal grains hulled
2. Chicken eggs cooked
3. Fish processed and canned
4. Newspapers containing advertising material
Which of the above items is/are exempted under GST (Goods and Services Tax)?
(a) 1 only
(b) 2 and 3 only
(c) 1, 2 and 4 only
(d) 1, 2, 3 and 4
Ans: c

 

38. Consider the following statements:
1. The definition of “Critical Wildlife Habitat” is incorporated in the Forest Rights Act, 2006.
2. For the first time in India, Baigas have been given Habitat Rights.
3. Union Ministry of Environment, Forest and Climate Change officially decides and declares Habitat Rights for Primitive and Vulnerable Tribal Groups in any part of India.
Which of the statements given above is/are correct?
(a) 1 and 2 only
(b) 2 and 3 only
(c) 3 only
(d) 1, 2 and 3
Ans: a
Exp: The Critical Wildlife Habitats have been envisaged in Scheduled Tribes and Other Traditional Forest Dwellers (Recognition of Forest Rights) Act, 2006. In a bid to undo historical injustice meted out to primitive tribal communities living in central India, the government of Madhya Pradesh has for the first time recognised the habitat rights of seven villages in Dindori district, mostly inhabited by the Baiga.Gram panchayats decide Habitat Rights for Primitive and Vulnerable.

 

39. Consider the following:
1. Birds
2. Dust blowing
3. Rain
4. Wind blowing
Which of the above spread plant diseases?
(a) 1 and 3 only
(b) 3 and 4 only
(c) 1, 2 and 4 only
(d) 1, 2, 3 and 4
Ans: d
Exp: Plant diseases can be infectious (transmitted from plant to plant) or noninfectious. Common plant disorders are caused by deficiencies in plant nutrients, by waterlogged or polluted soil, and by polluted air. Too little (or too much) water or improper nutrition can cause plants to grow poorly. Plants can also be stressed by weather that is too hot or too cold, by too little or too much light, and by heavy winds. Pollution from automobiles and industry, and the excessive application of herbicides (for weed control) can also cause noninfectious plant disorders.

Source: http://www.iasscore.in/upsc-prelims/general-science-part-2

 

40. With reference to organic farming in India, consider the following statements:
1. ‘The National Programme for Organic Production’ (NPOP) is operated under the guidelines and directions of the Union Ministry of Rural Development.
2. ‘The Agricultural and Processed Food Products Export Development Authority’ (APEDA) functions as the Secretariat for the implementation of NPOP.
3. Sikkim has become India’s first fully organic State.
Which of the statements given above is/are correct?
(a) 1 and 2 only
(b) 2 and 3 only
(c) 3 only
(d) 1, 2 and 3
Ans: b
Exp: Ministry of Commerce has implemented the National Programme for Organic Production (NPOP) since 2001. The testing is to be carried out in ISO 17025 accredited and preferably APEDA approved laboratories. Sikkim has become India’s first fully organic state by implementing organic practices on around 75,000 hectares of agricultural land.

 

41. Consider the following statements:
1. In the first Lok Sabha, the single largest party in the opposition was the Swatantra Party.
2. In the Lok Sabha, a “Leader of the Opposition” was recognised for the first time in 1969.
3. In the Lok Sabha, if a party does not have a minimum of 75 members, its leader cannot be recognized as the Leader of the Opposition.
Which of the statements given above is/are correct?
(a) 1 and 3 only
(b) 2 only
(c) 2 and 3 only
(d) 1, 2 and 3
Ans: b
Exp: 1952 elections The Congress had won 364 out of 489 Lok Sabha seats in the first-ever general election held in India. Its vote share was 45 per cent. The CPI was the second party with just 16 seats.
The Swatantra Party was an Indian liberal-conservative political party that existed from 1959 to 1974
Leader of opposition was recognized for the first time in 1969.
To become leader of opposition, a political party needs atleast 10% strength of the house. House total 10% seat means
Rajya Sabha 245 25 (10%)
Lok Sabha 543 55(10%)

 

GSSCORE QUESTION

Which of the following statements is/are true regarding post of Leader of Opposition?

1.   Post of Leader of Opposition is given to party leader and not to alliance leader.

2.   Original constitution provided statutory recognition to the leader of opposition.

3.   He is entitled to salary, allowances and other facilities equivalent to that of a cabinet minister.

Codes:

(a)   1 and 3

(b)   2 and 3

(c)   Only 3

(d)   All

Ans. A

Exp: Leader of the Opposition

•   In each House of Parliament, there is the ‘Leader of the Opposition’. The leader of the largest Opposition party having not less than one-tenth seats of the total strength of the House is recognised as the leader of the Opposition in that House.

•   The leader of Opposition in the LokSabha and the RajyaSabha were accorded statutory recognition in 1977.

•   They are also entitled to the salary, allowances and other facilities equivalent to that of a cabinet minister.

(PT 2018 test 10, question 43)

 

42. Which of the following leaf modifications occur(s) in the desert areas to inhibit water loss?
1. Hard and waxy leaves
2. Tiny leaves
3. Thorns instead of leaves
Select the correct answer using the code given below:
(a) 2 and 3 only
(b) 2 only
(c) 3 only
(d) 1, 2 and 3
Ans-d
Exp-“The vegetation of hot desert climate is xerophytic type which has special characteristics to withstand very high rate of evaporation. They have long roots, thick barks, waxy leaves, thorns and little leaves.”

 

GSSCORE QUESTION

Consider the following statements with respect to the adaptations made by animals in the Desert ecosystem:

1.   Desert plants show phenomenon of Allelopathy.

2.   Desert animals prevent water loss from their body by reducing surface area.

3.   Most of the desert plants store water in fleshy leaves, stems or roots and have deep root systems to get water supply.

Which of the above stated adaptations is/are correct?

(a)   1 and 2

(b)   Only 2

(c)   2 and 3

(d)   All

Ans: A

Exp: Desert plants show phenomenon of Allelopathyi,e, they secrete some chemical substance which inhibits the growth of plants growing in their near vicinity .

Succulent plants store water in fleshy leaves, stems or roots. All cacti are succulents, as are such non-cactus desert dwellers as agave, aloe, elephant trees, and many euphorbias. Several other adaptations are essential for the water storing habit to be effective.

Owl’s clover, California poppy and other drought avoidance plants die after channeling all their energy into producing seeds

A succulent must be able to absorb large quantities of water in short periods. Desert rains are often light and brief, and the soil dries rapidly under an intense sun. To cope with these conditions, nearly all succulents have extensive, shallow root systems. The roots of a saguaro extend horizontally about as far as the plant is tall but are rarely more than four inches (10 cm) deep. The water-absorbing roots are mostly within the upper half inch (1.3 cm). So that they can get water from atmosphere.

(PT 2018 Test 1, question 9)

Source: http://www.iasscore.in/upsc-prelims/adaptation-interaction-between-species

 

43. As per the NSSO 70th Round “Situation Assessment Survey of Agricultural Households”, consider the following statements:
1. Rajasthan has the highest percentage share of agricultural households among its rural households.
2. Out of the total agricultural households in the country, a little over 60 percent belong to OBCs.
3. In Kerala, a little over 60 percent of agricultural households reported to have received maximum income from sources other than agricultural activities.
Which of the statements given above is/are correct?
(a) 2 and 3 only
(b) 2 only
(c) 1 and 3 only
(d) 1, 2 and 3
Ans: c
Exp: Rajasthan has highest agricultural households (78.4) among rural households, in terms of proportion, in absolute numbers it is Uttar Pradesh
The share of OBC households among rural households is about 45 percent.
In Kerala, about 61 percent agricultural families have more income from agricultural sources.

 

44. How is the National Green Tribunal (NGT) different from the Central Pollution Control Board (CPCB)?
1. The NGT has been established by an Act whereas the CPCB has been created by an executive order of the Government.
2. The NGT provides environmental justice and helps reduce the burden of litigation in the higher courts whereas the CPCB promotes cleanliness of streams and wells, and aims to improve the quality of air in the country.
Which of the statements given above is/are correct?
(a) 1 only
(b) 2 only
(c) Both 1 and 2
(d) Neither 1 nor 2
Ans: b
Exp: The Central Pollution Control Board (CPCB), is statutory organisation, was constituted in September, 1974 under the Water (Prevention and Control of Pollution) Act, 1974. It is also the statutory body.

Source: http://iasscore.in/upsc-prelims/environmental-institutions-environmental-legislations

 

45. Consider the following statements:
1. The Parliament of India can place a particular law in the Ninth Schedule of the Constitutions of India.
2. The validity of a law placed in the Ninth Schedule cannot be examined by any court and no judgement can be made on it.
Which of the statements given above is/are correct?
(a) 1 only
(b) 2 only
(c) Both 1 and 2
(d) Neither 1 nor 2
Ans: a
Exp: In a landmark ruling on 11 January 2007, the Supreme Court of India ruled that all laws (including those in the Ninth Schedule) would be open to Judicial Review if they violated the basic structure of the constitution.

 

46. Which one of the following best describes the term “Merchant Discount Rate” sometimes seen in news?
(a) The incentive given by a bank to a merchant for accepting payments through debit cards pertaining to that bank.
(b) The amount paid back by banks to their customers when they use debit cards for financial transactions for purchasing goods or services.
(c) The charge to a merchant by a bank for accepting payments from his customers through the bank’s debit cards.
(d) The incentive given by the Government to merchants for promoting digital payments by their customers through Point of Sale (PoS) machines and debit cards.
Answer – c)

Exp: Merchant discount rate on debit cards is the amount that a merchant has to pay to its service providers when a consumer swipes her card on the merchant’s point-of-sales terminal. It is also applicable for online transactions and QR-based transactions.
The amount that the merchant pays for every transaction gets distributed among three stakeholders—the bank that enable the transaction, the vendor that installs the PoS machine and the card network provider such as Visa or MasterCard.

 

47. What is/are the consequence/consequences of a country becoming the member of the ‘Nuclear Suppliers Groups’?
1. It will have access to the latest and most efficient nuclear technologies.
2. It automatically becomes a member of “The Treaty on the Non-Proliferation of Nuclear Weapons (NPT)”.
Which of the statements given above is/are correct?
(a) 1 only
(b) 2 only
(c) Both 1 and 2
(d) Neither 1 nor 2
Ans: a
Exp: NSG membership also means India can begin to commercially produce nuclear power equipment, which it can then even sell to other countries. With access to state-of-the-art nuclear technologies, it can maximize its production benefits.
NSG member will not automatically becomes a member of “The Treaty on the Non-Proliferation of Nuclear Weapons (NPT)”.

Source: http://www.iasscore.in/topical-analysis/nuclear-suppliers-group-nsg-

 

48. With reference to India’s decision to levy an equalization tax of 6% on online advertisement services offered by non-resident entities, which of the following statements is/are correct?
1. It is introduced as a part of the Income Tax Act.
2. Non-resident entities that offer advertisement services in India can claim a tax credit in their home country under the “Double Taxation Avoidance Agreements”.
Select the correct answer using the code given below:
(a) 1 only
(b) 2 only
(c) Both 1 and 2
(d) Neither 1 nor 2
Ans: d
Exp: The levy was introduced in the Budget as part of the finance bill and not as a part of Income Tax Act. So, because of this the companies would not be able to take the benefit of tax treaties to avoid double taxation in their home countries.
It provides that income of the non resident from provision of the specified services to the assessee under chapter VIII of the FA is exempt from income tax in the hands of the non resident if the same is chargeable to equalization levy. However, it does not mean that the income of the non resident from the specified services would be charged to income tax if the same is not chargeable to equalization levy.

 

49. Consider the following statements:
1. The Fiscal Responsibility and Budget Management (FRBM) Review Committee Report has recommended a debt to GDP ratio of 60% for the general (combined) government by 2023, comprising 40% for the Central Government and 20% for the State Governments.
2. The Central Government has domestic liabilities of 21% of GDP as compared to that of 49% of GDP of the State Governments.
3. As per the Constitution of India, it is mandatory for a State to take the Central Government’s consent for raising any loan if the former owes any outstanding liabilities to the latter.
Which of the statements given above is/are correct?
(a) 1 only
(b) 2 and 3 only
(c) 1 and 3 only
(d) 1, 2 and 3

Ans: c
Exp: As per N.K. Singh committee report recommendations:
The combined debt-to-GDP ratio of the centre and states should be brought down to 60 per cent by 2023 (comprising of 40 per cent for the Centre and 20% for states) as against the existing 49.4 per cent, and 21 per cent respectively.
Statement 3 is correct as per article 293 of the constitution.

Source: http://www.thehindu.com/opinion/lead/learning-to-run-twice-as-fast/article18260719.ece

 

50. Consider the following statements:
1. The quantity of imported edible oils is more than the domestic production of edible oils in the last five years.
2. The government does not impose any customs duty on all the imported edible oils as a special case.
Which of the statements given above is/are correct?
(a) 1 only
(b) 2 only
(c) Both 1 and 2
(d) Neither 1 nor 2
Ans: a
Exp: The quantity of the imported oil is almost 60% of total oil consumption in India. Government imposes custom duty on several edible oil, such as palm oil

 

51. He wrote biographies of Mazzini, Garibaldi, Shivaji and Shrikrishna; stayed in America for some time; and was also elected to the Central Assembly. He was
(a) Aurobindo Ghosh
(b) Bipin Chandra Pal
(c) Lala Lajpat Rai
(d) Motilal Nehru
Ans: c

 

52. Consider the following statements:
1. Aadhaar card can be used as a proof of citizenship or domicile.
2. Once issued, Aadhaar number cannot be deactivated or omitted by the Issuing Authority.
Which of the statements given above is/are correct?
(a) 1 only
(b) 2 only
(c) Both 1 and 2
(d) Neither 1 nor 2
Ans: d
Exp: Calcutta High Court ruled that Aadhaar Card is not a proof of citizenship and stated that “Aadhaar Card by itself shall not confer any right of or be proof of citizenship or domicile in respect of the holder thereto” while rejecting a claim of citizenship of the accused who was issued the card in view of his long residence in the country.
The Aadhaar Act regulations, which are currently in Parliament, state that an individual’s Aadhaar number may be “omitted” permanently or deactivated temporarily by the Unique Identification Authority of India, the agency responsible for issuing the numbers and managing the database. The regulations give the Authority the power to deactivate Aadhaar numbers even in the absence of an effective grievance redressal procedure for those whose numbers have been suspended.

Source:http://iasscore.in/national-issues/the-aadhaar-targeted-delivery-of-financial-and-other-subsidies-benefits-and-service-bill-2016

 

53. Which of the following has/have shrunk immensely/dried up in the recent past due to human activities?
1. Aral Sea
2. Black Sea
3. Lake Baikal
Select the correct answer using the code given below:
(a) 1 only
(b) 2 and 3
(c) 2 only
(d) 1 and 3
Ans: d

 

54. “Rule of Law Index” is released by which of the following?
(a) Amnesty International
(b) International Court of Justice
(c) The office of UN Commissioner for Human Rights
(d) World Justice Project
Ans: d
Exp: The World Justice Project Rule of Law Index® measures rule of law adherence in 113 countries and jurisdictions worldwide based on more than 110,000 household and 3,000 expert surveys. Featuring primary data, the WJP Rule of Law Index measures countries’ rule of law performance across eight factors: Constraints on Government Powers, Absence of Corruption, Open Government, Fundamental Rights, Order and Security, Regulatory Enforcement, Civil Justice, and Criminal Justice.
55. Which one of the following links all the ATMs in India?
(a) Indian Banks’ Association
(b) National Securities Depository Limited
(c) National Payments Corporation of India
(d) Reserve Bank of India
Ans: c
Exp: National Financial Switch (NFS) is the largest network of shared automated teller machines (ATMs) in India. It was designed, developed and deployed by the Institute for Development and Research in Banking Technology (IDRBT) in 2004, with the goal of inter-connecting the ATMs in the country and facilitating convenience banking. It is run by the National Payments Corporation of India(NPCI).

 

56. Regarding Money Bill, which of the following statements in not correct?
(a) A bill shall be deemed to be a Money Bill if it contains only provisions relating to imposition, abolition, remission, alteration or regulation of any tax.
(b) A Money Bill has provisions for the custody of the consolidated fund of India or the contingency fund of India.
(c) A Money Bill is concerned with the appropriation of moneys out of the contingency fund of India.
(d) A Money Bill deals with the regulation of borrowing of money or giving any guarantee by the government of India.
Ans: c
Exp: Article 110 states that a Bill is deemed to be a money bill if it contains provisions dealing with all or any of the following matters:
(i) The imposition, abolition, remission, alteration or regulation of any tax
(ii) The regulation of the borrowing of money or the giving of any guarantee by the Government of India, or the amendment of the law with respect to any financial obligations undertaken or to be undertaken by the Government of India
(iii) The custody of the Consolidated Fund or the Contingency Fund of India, the payment of moneys into or the withdrawal of moneys from any such fund
(iv) The appropriation of moneys out of the Consolidated Fund of India
(v) The declaring of any expenditure to be expenditure charged on the consolidated Fund of India or the increasing of the amount of any such expenditure
(vi) The receipt of money on account of the Consolidated Fund of India or the Public Account of India or the custody or issue of such money or the audit of the accounts of the Union or of a State

Source: http://iasscore.in/upsc-prelims/parliamentary-functioning-passage-bill-budgeting

 

57. With reference to the election of the president of India, consider the following statements.
1. The value of the vote of each MLA varies from state to state.
2. The value of the vote of MPs of the Lok Sabha is more than the value of the vote of KPs of the Rajya Sabha.
Which of the statements given above is/are correct?
(a) 1 only
(b) 2 only
(c) Both 1 and 2
(d) Neither 1 nor 2
Ans: c
Exp: Value of a vote and securing parity
a) Art. 55 provides for uniformity in the scale of representation of different states at the election of the President.
b) Besides, there should also be parity between the states as a whole and the union.
c) For this purpose the value of votes of an MLA and an MP is counted under the following formula:

The number of MPs in Lok Sabha are higher so value of vote is larger than Rajya Sabha. Question has asked MPs not MP.

Source: http://iasscore.in/upsc-prelims/president-functions-governor-functions

 

58. In the context, what is the implication of ratifying the ‘Additional protocol’ with the international Atomic Energy Agency (IAEA)’?
(a) The civilian nuclear reactors come under IAEA safeguards.
(b) The military nuclear installations come under the inspection of IAEA.
(c) The country will have the privilege to but uranium from the Nuclear Suppliers Group (NSG).
(d) The country automatically becomes a member of the NSG
Ans: a
Exp: The Additional Protocol is not a stand-alone agreement, but rather a protocol to a safeguards agreement that provides additional tools for verification. In particular, it significantly increases the IAEA’s ability to verify the peaceful use of all nuclear material in States with comprehensive safeguards agreements.

 

59. Consider the following countries.
1. Australia
2. Canada
3. China
4. India
5. Japan
6. USA
Which of the above are among the ‘free-trade partners’ of ASEAN?
(a) 1, 2, 4 and 5
(b) 3, 4, 5, and 6
(c) 1, 3, 4 and 5
(d) 2, 3, 4 and 6
Ans: c
Exp: ASEAN has existing free trade agreements with Australia, China, India, Japan, South Korea and New Zealand.

Source:  http://www.iasscore.in/upsc-prelims/trade-agreements

 

60. With reference to the ‘Global Alliance for Climate-Smart Agriculture (GACSA)’, which of the following statements is/are correct?
1. GACSA is an outcome of the Climate Summit held in Paris in 2015
2. Membership of GACSA does not create any binding obligations.
3. India was instrumental in the creation of GACSA
Select the correct answer using the code given below:
(a) 1 and 3 only
(b) 2 only
(c) 2 and 3 only
(d) 1, 2 and 3
Ans: b *
Exp: The Global Alliance for Climate-Smart Agriculture (GACSA), was launched on 23 September 2014 at the UN Climate Summit. GACSA is an independent alliance, governed by its members through a Strategic Committee and its co-Chairs. Since the 1 January FAO is hosting the Facilitation Unit of GACSA with is supported through a multi donor trust fund.
GACSA is a voluntary alliance of partners, dedicated to addressing the challenges facing food security and agriculture under a changing climate. In particular the alliance has the objective of up scaling the climate smart agriculture approach, a concept which was originally developed by FAO.
GACSA’s ambitious future is based upon supporting and inspiring action. Farmers, fishers, foresters, and ranchers are at the center of this action, and therefore GACSA is devoting its Annual Forum to showcasing climate-smart agriculture in action.
The Forum will be a dynamic gathering, where the participants share solutions, discuss challenges, and build partnerships on climate-smart agriculture. We are seeking out action-oriented stakeholders to come to both share and learn at the GACSA Annual Forum.

 

61. Which of the following is/are the aim/aims of “Digital India” plan of the Government of India?
1. Formation of India’s own Internet companies like china did.
2. Established a policy framework to encourage overseas multinational corporations that collect big data to build their large data centers within our national geographical boundaries.
3. Connect many of our villages to the internet and bring Wi-Fi to many of our schools, public places and major tourist centers.
Select the correct answer using the code given below:
(a) 1 and 2 only
(b) 3 only
(c) 2 and 3 only
(d) 1, 2 and 3
Ans: b
Exp: Digital Infrastructure as a Utility to Every Citizen
• Availability of high speed internet as a core utility for delivery of services to citizens
• Cradle to grave digital identity that is unique, lifelong, online and authenticable to every citizen
• Mobile phone & bank account enabling citizen participation in digital & financial space
• Easy access to a Common Service Centre
• Shareable private space on a public cloud
• Safe and secure cyber-space

Digital Empowerment of Citizens
• Universal digital literacy
• Universally accessible digital resources
• Availability of digital resources / services in Indian languages
• Collaborative digital platforms for participative governance
• Citizens not required to physically submit Govt. documents / certificates

Source: http://iasscore.in/national-issues/digital-india-programme-importance-and-impact

 

62. Consider the following pairs:
Towns sometimes mentioned in news Country
1. Aleppo – Syria
2. Kirkuk – Yemen
3. Mosul – Palestine
4. Mazar-i-sharif – Afghanistan
Which of the pairs given above are correctly matched?
(a) 1 and 2
(b) 1 and 4
(c) 2 and 3
(d) 3 and 4

Ans: b

63. In the Federation established by The Government of India Act of 1935, residuary powers were given to the
(a) Federal Legislative
(b) Governor General
(c) Provincial Legislature
(d) Provincial Governors
Ans: b
Exp: The residuary power, however, were not allocated either to the Federation or to the Provinces but was under Section 105 of the Government of India, 1935, reserved to be allocated by the Governor-General in his discretion to the Federation or to the Provinces.

Source: http://iasscore.in/upsc-prelims/constitution-and-its-typesconstitutional-development

 

64. Consider the following statements:
1. The Speakers of the Legislative Assembly shall vacate his/her office if he/she ceases to be a member of the Assembly
2. Whenever the legislative assembly is dissolved, the speaker shall vacate his/her office immediately.
Which of the statements given above is/are correct?
(a) 1 only
(b) 2 only
(c) Both 1 and 2
(d) Neither 1 nor 2
Ans: a
Exp: Speaker of Assembly vacates his office earlier in any of the following three cases:
1. if he ceases to be a member of the assembly;
2. if he resigns by writing to the deputy speaker; and
3. if he is removed by a resolution passed by a majority of all the then members of the assembly.

Such a resolution can be moved only after giving 14 days advance notice.

Whenever the Assembly is dissolved, the Speaker does not vacate his office and continues till the newly- elected member meets.

Source: http://www.iasscore.in/upsc-prelims/parliamentary-functioning-passage-bill-budgeting

 

65. Which one of the following reflects the most appropriate relationship between law and liberty?
(a) It there are more laws, there is less liberty.
(b) If there are no laws, there is no liberty.
(c) If there is liberty, laws have to be made by the people.
(d) If laws are changed too often, liberty is in danger
Ans: b
Exp: It was a statement by John Locke and also endorsed by Leviathan theory Thomas Hobbes.

 

66. Consider the following statements:
1. No criminal proceedings shall be instituted against the Governor of a State in any court during his term of office.
2. The emoluments and allowance of the Governor of a State shall not be diminished during his term of office.
Which of the statements given above is/are correct?
(a) 1 only
(b) 2 only
(c) Both 1 and 2
(d) Neither 1 nor 2
Ans: c
Exp: Like the President, the governor is also entitled to a number of privileges and immunities. He enjoys personal immunity from legal liability for his official acts. During his term of office, he is immune from any criminal proceedings, even in respect of his personal acts.

The consequences of the proclamation of a Financial Emergency are as follows:
1. The executive authority of the Centre extends (a) to directing any state to observe such canons of financial propriety as are specified by it; and (b) to directions as the President may deem necessary and adequate for the purpose.
2. Any such direction may include a provision requiring (a) the reduction of salaries and allowances of all or any class of persons serving in the state; and (b) the reservation of all money bills or other financial bills for the consideration of the President after they are passed by the legislature of the state.
3. The President may issue directions for the reduction of salaries and allowances of (a) all or any class of persons serving the Union; and (b) the judges of the Supreme Court and the high court.

 

GSSCORE QUESTION

Consider the following statements with reference to the Governor in the State:

1.   Same person can be appointed as a Governor of State and adjacent Union Territory.

2.   Governor remains in the post during the pleasure of the President.

3.   Governor cannot be imprisoned by State police for his criminal act.

Which of the above statements is/are incorrect?

(a)   1 and 3

(b)   Only 1

(c)   1 and 2

(d)   2 and 3

Ans. B

Exp: Governor

•   Usually, there is a governor for each state, but the 7th Constitutional Amendment Act of 1956 facilitated the appointment of the same person as a governor for two or more states. But union territories have either Lieutenant Governor or Administrator; governor has nothing to do with Union territory.

•   House of residence of Governor is called Raj Bhavan. The Governor shall hold office during the pleasure of the President, after his removal, he must vacate the Raj Bhavan.

•   Governor enjoys personal immunity from legal liability for his Official acts. During his term of office, he is immune from any criminal Proceedings, even in respect of his personal acts. He cannot be arrested or imprisoned. However, after giving two months’ notice, civil proceedings can be instituted against him during his term of office in respect of his personalacts.

(PT 2018 Test code 31, question 32)

 

67. The well-known painting “Bani Thani” belongs to the
(a) Bundi schools
(b) Jaipur school
(c) Kangra school
(d) Kishangarh school
Ans: d
Exp: Bani Thani is an India miniature painting painted by Nihâl Chand from the Marwar school of Kishangarh. It portrays a woman who is elegant and graceful. The painting’s subject, Bani Thani, was a singer and poet in Kishangarh in the time of king Sawant Singh (1748–1764). She has been compared to the Mona Lisa.

 

 

68. What is “Terminal High Altitude Area Defense (THAAD)” sometimes seen in the news?
(a) An Israeli radar system
(b) India’s indigenous anti missile Programme.
(c) An American anti missile system
(d) A defense collaborations between Japan and South Korea
Ans: c
Exp: Terminal High Altitude Area Defense (THAAD), formerly Theater High Altitude Area Defense, is an American anti-ballistic missile defense system designed to shoot down short-, medium-, and intermediate-range ballistic missiles in their terminal phase (descent or reentry) by intercepting with a hit-to-kill approach.

 

 

69. With references to cultural history of India, consider the following statements:
1. Most of the Tyagaraja kritis are devotional songs in praise of lord Krishna.
2. Tyagaraja created several new ragas.
3. Annamacharya and Tyagaraj are contemporaries
4. Annamacharya kirtanas are devotional songs in praise of Lord Venkateshwara
Which of the statements given above are correct?
(a) 1 and 3 only
(b) 2 and 4 only
(c) 1, 2 and 3
(d) 2, 3 and 4
Ans: b
Exp: Tyagaraja composed thousands of devotional compositions, most in Telugu and in praise of Lord Rama many of which remain popular today. [2] Of special mention are five of his compositions called the Pancharatna Kritis which are often sung in programs in his honour. Annamayya (22 May 1408 – 4 April 1503) was a 15th-century Hindu saint and is the earliest known Indian musician to compose songs called sankirtanas in praise of the god Venkateswara, a form of Vishnu.

 

 

70. Which of the following are regarded as the main features of the “Rule of Law”?
1. Limitation of powers
2. Equality before law
3. People’s responsibility to the government
4. Liberty and civil rights
Select the correct answer using the code given below:
(a) 1 and 3 only
(b) 2 and 4 only
(c) 1,2 and 4 only
(d) 1, 2, 3 and 4
Ans: c
Exp: The Rule of Law identified eight principles to define it.
1. The law must be accessible and predictable.
2. Questions of legal rights should be resolved by the law and not the exercise of discretion.
3. The law should apply equally to all, except where objective differences justify differentiation.
4. Ministers must act within their powers and not exceed their limits.
5. The law must afford adequate protection of fundamental human rights.

Source: http://iasscore.in/national-issues/concept-of-rule-of-law

 

71. With reference to the Indian Regional Navigation Satellite System (IRNSS), consider the following statements.
1. IRNSS has three satellites in geostationary and four satellites in geosynchronous orbits.
2. IRNSS covers entire India and about 5500 sq. km beyond its borders.
3. India will have its own satellite navigation system with full global coverage by the middle of 2019.
Which of the statements given above is/are correct?
(a) 1 only
(b) 1 and 2 only
(c) 2 and 3 only
(d) None
Ans: a
Exp: ISRO opted for seven satellites — three in geostationary and four (as two pairs) in geosynchronous orbits — to provide the best navigation services.
India has an independent regional navigation satellite capability that covers the entire country and an area extending about 1,500 sq. km beyond its border, with a position accuracy better than 20m in all weather conditions.
The satellite is also working.

Source: http://www.iasscore.in/upsc-prelims/recent-and-future-space-missions

 

72. Consider the following phenomena:
1. Light is affected by gravity.
2. The universe is constantly expanding.
3. Matter warps its surrounding space-time.
Which of the above is/are the prediction/predictions of Albert Einstein’s General Theory of Relativity, often discussed in media?
(a) 1 and 2 only
(b) 3 only
(c) 1 and 3 only
(d) 1, 2 and 3
Ans: c
Exp: Einstein’s general theory of relativity is based on how we think gravity governs the behaviour of the Universe. We know that matter in the Universe warps the surrounding fabric of spacetime, and this warping effect is what we refer to as gravity.

According to Einstein’s theory, light, just like any other form of matter, is affected by gravity. That is, light also “falls” in a gravitational field. Just as a comet’s trajectory is deflected by the sun when it passes nearby, a ray of starlight grazing the sun would also be deflected or bent. When we (or our camera) look at the star, we presume that the path taken by the light is straight. The effect of the bending of the light, then, is to make the star appear to be located in a slightly different position in the sky.
Source: http://www.iasscore.in/topical-analysis/nuclear-suppliers-group-nsg-

 

73. With reference to the Genetically Modified mustard (GM mustard) developed in India, consider the following statements:
1. GM mustard has the genes of a soil bacterium that give the plant the property of pest-resistance to a wide variety of pests.
2. GM mustard has the genes that allow the plant cross-pollination and hybridization.
3. GM mustard has been developed jointly by the IARI and Punjab Agricultural University.
Which of the statements give above is/are correct?
(a) 1 and 3 only
(b) 2 only
(c) 2 and 3 only
(d) 1, 2 and 3
Ans: b
Exp: DMH-11 is a Genetically Modified (GM) mustard hybrid. Hybrids are normally obtained by crossing 2 genetically diverse plants from the same species. The 1st-generation offspring resulting from it has higher yields than what either of the parents is individually capable of giving. But there is no natural hybridization system in mustard, unlike in, say, cotton, maize or tomato. This is because its flowers contain both the female (pistil) and male (stamen) reproductive organs, making the plant naturally self-pollinating.

Source: http://iasscore.in/national-issues/-gm-mustard-issue

 

74. Consider the following pairs:
Terms sometimes Context/Topic
1. Belle II experiment ___ Artificial Intelligence
2. Blockchain technology ___ Digital/Cryptocurrency
3. CRISPR – Cas9 ___ Particle Physics
Which of the pairs given above is/are correctly matched?
(a) 1 and 3 only
(b) 2 only
(c) 2 and 3 only
(d) 1, 2 and 3
Ans: b
Exp: The Belle II experiment is a particle physics experiment designed to study the properties of B mesons (heavy particles containing a bottom quark).
CRISPR – Cas9 is related to biotechnology.

 

GSSCORE QUESTION

With reference to the CRISPR-Cas9, consider the following statements:

1.   It could be used to modify disease causing Genes and removing the faulty script from the genetic code.

2.   It may become a tool for selecting desired characteristics such as Intelligence and Attractiveness.

Which of the above statements is/are correct?

(a)   Only 1

(b)   Only 2

(c)   Both

(d)   None

Ans. C

Exp: Genome editing (also called gene editing) is a group of technologies that give scientists the ability to change an organism’s DNA. These technologies allow genetic material to be added, removed, or altered at particular locations in the genome.

CRISPR could be used to modify disease-causing genes in embryos brought to term, removing the faulty script from the genetic code of that person’s future descendants as well. Genome editing (Gene editing) could potentially decrease, or even eliminate, the incidence of many serious genetic diseases, reducing human suffering worldwide.

It might also be possible to install genes that offer lifelong protection against infection.

Making irreversible changes to every cell in the bodies of future children and all their descendants would constitute extraordinarily risky human experimentation.

It will become a tool for selecting desired characteristics such as intelligence and attractiveness.

(PT test 2018 science sectional test, question 17)

 

75. Which of the following statement best describes “carbon fertilization?
(a) Increased plant growth due to increased concentration of carbon dioxide in the atmosphere
(b) Increased temperature of Earth due to increased concentration of carbon dioxide in the atmosphere
(c) Increased acidity of oceans as the result of increased concentration of carbon dioxide in the atmosphere
(d) Adaptation of all living beings on Earth to the climate change brought about by the increased concentration of carbon dioxide in the atmosphere
Ans: a
Exp: The CO2 fertilization effect or carbon fertilization effect suggests that the increase of carbon dioxide in the atmosphere increases the rate of photosynthesis in plants. The effect varies depending on the plant species, the temperature, and the availability of water and nutrients

Source: http://www.iasscore.in/upsc-prelims/government-initiatives-in-it-sector

 

76. When the alarm of your smartphone rings in the morning, you wake up and tap it to stop the alarm which causes your geyser to be switched on automatically. The smart mirror in your bathroom shows the day’s weather and also indicates the level of water in your overhead tank. After you take some groceries from your refrigerator for making breakfast, it recognizes the shortage of stock in it and places an order for the supply of fresh grocery items. When you step out of your house and lock the door, all lights, fans, geysers and AC machines get switched off automatically. On your way to office, your car warns you about traffic congestion ahead and suggests an alternative route, and if yourare late for a meeting, it sends a message to your office accordingly.
In the context of emerging communication technologies, which one of the following terms best applies to the above scenario?
(a) Border Gateway Protocol
(b) Internet of Things
(c) Internet Protocol
(d) Virtual Private Network
Ans: b

 

77. Which reference to solar power production in India, consider the following statements:
1. India is the third largest in the world in the manufacture of silicon waters used in photovoltaic units.
2. The solar power tariffs are determined by the Solar Energy Corporation of India
Which of the statements given above is/are correct?
(a) 1 only
(b) 2only
(c) Both 1 and 2
(d) Neither 1 nor 2
Ans: D *

 

78. The staple commodities of export by the English East India Company from Bengal in the middle of the 18th century were
(a) Raw cotton, oil-seeds and opium
(b) Sugar, salt, zinc and lead
(c) Copper, silver, gold, spices and tea
(d) Cotton, silk, saltpeter and opium
Ans: d
Exp: British-Indian territory was developed as a source of food stuff and raw material for Britain, which fueled rapid growth in its manufacturing sector, crucial to the emergence of a powerful capitalist economy. (Indian exports consisted of raw cotton, jute, silk, oilseeds, wheat, indigo and tea.)

Source: http://iasscore.in/upsc-prelims/administration-before-1857-economic-policies

 

79. Which one of the following is a very significant aspect of the Champaran Satyagraha?
(a) Active all-India participation of lawyers, students and women in the National Movement
(b) Active involvement of Dalit and Tribal communities of India in the National Movement
(C) Joining of peasant unrest to India’s National Movement
(d) Drastic decrease in the cultivation of plantation crops and commercial crops
Ans: c

Source: http://iasscore.in/upsc-prelims/middle-phase-of-the-freedom-struggle-1915-1930

 

80. Who among the following were the founders of the “Hind Mazdoor Sabha” established in 1948?
(a) B. Krishna Pillai, E.M.S. Namboodiripad and K.C. George
(b) Jayaprakash Narayan, Deen Dayal Upadhyay and M.N. Roy
(c) C.P. Ramaswamy Iyer, K. Kamaraj and Veeresalingam Pantulu
(d) Ashok Mehta, T.S. Ramanujam and G.G. Mehta
Ans: d
Exp: The HMS was founded in Howrah in west bengal on 29 December 1948, by socialists, Forward Bloc followers and independent unionists. Its founders included Basawon Singh (Sinha), Ashok Mehta, R.S. Ruikar, Maniben Kara, Shibnath Banerjee, R.A. Khedgikar, T.S. Ramanujam, V.S. Mathur, G.G. Mehta. R.S. Ruikar was elected president and Ashok Mehta general secretary. HMS absorbed the Royist Indian Federation of Labour and the Hind Mazdoor Panchayat, which was formed in 1948 by socialists leaving the increasingly communist dominated AITUC

 

81. With reference to the religious practices in India, the “Sthanakvasi” sect belongs to
(a) Buddhism
(b) Jainism
(c) Vaishnavism
(d) Shaivism
Ans: b
Exp: After Bhadrabahu, the Jainism split into Digambara and Svetambara. The Digambara belong to the lineage of Acharya Vishakha and Shvetambar follow the tradition of Acharya Sthulabhadra. In around 1500 AD, the Swetambar sect divided into three sub-sects known as Swetambar Murtipujak, Sthanakvasi, and Terapanthi.

 

82. With reference to the cultural History of India, consider the following statements:
1. White marble was used in making Buland Darwaza and Khankah at Fatehpur Sikr.
2. Red sandstone and marble were used in making Bara Imambara and Rumi Darwaza at Lucknow.
Which of the statements given above is/are correct?
(a) 1 only
(b) 2 only
(c) Both 1 and 2
(d) Neither 1 nor 2
Ans: a
Exp: The Buland Darwaza at Fatehpur Sikri is “the most iconic architectural accomplishment of Akbar’s reign. It incorporates almost all the essential features of Akbar’s architectural traditions: red sandstone, stone carvings, relief by inserting white marble
Whereas Imambara instead of stones and marble, brick and lime were used. Stucco ornamentation (gajkari) was used to decorate the monuments, giving it a deep relief effect even on flat walls.

 

83. Which one of the following foreign travelers elaborately discussed about diamonds and diamond mines on India
(a) Francois Bernier
(b) Jean Baptiste Tavernier
(c) Jean de Thevenot
(d) Abbe Barthelemy Carre
Ans: b
Exp: Jean-Baptiste Tavernier (1605 – 1689)was a 17th-century French gem merchant and traveler.Tavernier, a private individual and merchant traveling at his own expense, covered, by his own account, 60,000 leagues (120,000 miles[clarification needed]) in making six voyages to Persia and India between the years 1630 and 1668.

 

84. With reference to India history, who among the following is future Buddha, yet to come to save the world?
(a) Avalokiteshvara
(b) Lokesvara
(c) Maitreya
(d) Padmapani
Ans: c
Exp: Maitreya Buddha
In the world of Buddhist eschatology, Maitreya literally means the future Buddha. Maitreya Buddha is considered as the 5th Buddha that is believed to appear in this Kalpa or era. Thus, Maitreya Buddha is considered as the Future Buddha that is yet to appear in this age. In various Buddhist sutra such as Amitabha Sutra, as well as Lotus Sutra, Maitreya Buddha is believed to be called as Ajita.
According to Buddhist history and tradition, Maitreya Buddha is believed to be Bodhisattva who will appear in the Earth in the future, will achieve Nirvana and will teach the people of Earth the pure Dharma just like Shakyamuni Buddha did. According to the Buddhist texts as well scriptures, Maitreya Buddha will be considered as the successor of the living Buddha i.e. Gautama Buddha. The Prophecy of Maitreya Buddha coming back to the terrestrial world is written in most of the major Schools of Buddhism in many Buddhist countries.

 

85. Which one of the following statements does not apply to the system of Subsidiary Alliance introduced by Lord Wellesley?
(a) To maintain a large standing army at other’ expense
(b) To keep India safe from Napoleonic danger
(c) To secure a fixed income for the Company
(d) To establish British paramountcy over the Indian States
Ans: c
Exp: (i) Those native princes or rulers who would enter into Subsidiary Alliance would not be free to declare war against any other power, nor enter into negotiations with, any power, native or otherwise without the con­sent of the English,
(ii) The princes who were comparatively strong and powerful would be permitted to retain their armies, but their armies must be placed under British generals,
(iii) The security of the dominion of a prince who would enter into Subsidiary Alliance would be the responsibility of the English but the expenses of forces that the latter would maintain for the purpose had to be borne by the prince for which adequate area of his dominion had to be surren­dered to the English.

 

GSSCORE QUESTION

Consider the following statements related to the work done by Governors:

1.   Warren Hastings abolished the system of dastaks, or free passes and regulated the internal trade.

2.   Lord Cornwallis inaugurated the policy of making appointments mainly on the basis of merit in civil services.

3.   Lord Hastings initiated the policy of Subsidiary Alliance under which the British had to maintain a contingent of British troops in territory of Indian ruler.

Which of the above stated works is/are correctly matched?

(a)   1 and 3

(b)   1 and 2

(c)   Only 2

(d)   All

Ans: B

Exp: Lord Wellesley initiated the policy of Subsidiary Alliance under which the British had to maintain a contingent of British troops in territory of Indian ruler.

The main principles of a subsidiary alliance were:

•   An Indian ruler entering into a subsidiary alliance with the British had to accept British forces in his  territory and also agreed to pay for their maintenance.

•   The ruler would accept a British people in his state.

•   Such a ruler would not form an alliance with any  other power, nor would he declare war against any power without the permission of the British.

•   The ruler would not employ any Europeans other than the British, and if he were already doing so, he would dismiss them.

•   In case of a conflict with any other state, the British would decide what to do, and he would accept their solution.

(PT crash course modern history test 3)

 

 


86. Consider the following statements:

1. Capital Adequacy Ratio (CAR) is the amount that banks have to maintain in the form of their own funds to offset any loss that banks incur if the account-holders fail to repay dues.
2. CAR is decided by each individual bank.
Which of the statements given above is/are correct?
(a) 1 only
(b) 2 only
(c) Both 1 and 2
(d) Neither 1 nor 2
Ans: a
Exp: CAR is not decided by each individual bank but by RBI on the basis of Basel Committee recommendations.

 

GSSCORE QUESTION

Why maintaining Capital Adequacy Ratio (CAR) is/are must in banks?

1.   Bank capital helps to prevent bank failure.

2.   It acts as a cushion to lessen the chance of the bank turning insolvent.

Which of the above stated reasons is/are correct?

(a)   Only 1

(b)   Only 2

(c)   Both

(d)   None

Ans: C

Exp: Why To Maintain Capital Adequacy Ratio (CAR):

•   Bank capital helps to prevent bank failure, which arises in case the bank cannot satisfy itsobligations to pay the depositors and other creditors.

•   The low capital bank has a negative net worth after the loss in its business. In other words, it turns into insolvent capital, therefore, acts as a cushion to lessen the chance of the bank turning insolvent.

•   The amount of capital affects returns for the owners (equity holders) of the bank.

(PT crash course Economy test 3)

87. The identity platform ‘Aadhaar’ provides open “Application Programming Interfaces (APIs)”. What does it imply?
1. It can be integrated into any electronic device.
2. Online authentication using iris is possible.
Which of the statements given above is/are correct?
(a) 1 only
(b) 2 only
(c) Both 1 and 2
(d) Neither 1 nor 2
Ans: C *

 

88. Very recently, in which of the following countries have lakhs of people either suffered from severe famine/acute malnutrition or died due to starvation caused by war/ethnic conflicts?
(a) Angola and Zambia
(b) Morocco and Tunisia
(c) Venezuela and Colombia
(d) Yemen and South Sudan
Ans: d

 

89. Regarding Wood’s Dispatch, which of the following statements are true?
1. Grants-in-Aid system was introduced
2. Establishment of universities was recommended.
3. English as a medium of instruction at all levels of education was recommended.
Select the correct answer using the code given below:
(a) 1 and 2 only
(b) 2 and 3 only
(c) 1 and 3 only
(d) 1, 2 and 3
Ans: a
Exp: Major Recommendations of Wood’s Dispatch
• An education department was to be established in every province.
• Universities on the model of the London University are established in big cities such as Bombay, Calcutta and Madras.
• At least one Government school is opened in every district.
• Affiliated private schools should be given grant-in-aid.
• The Indian natives should be given training in their mother-tongue also.
• In accordance with the Wood’s despatch, Education Departments were established in every province and universities were opened at Calcutta, Bombay and Madras in 1857 A.D.-and in Punjab in 1882 A.D. and at Allahabad in 1887 A.D.
• The medium of instruction at the primary level was to be vernacular while at the higher levels it would be English.

 

GSSCORE QUESTION

Which of the following statements is/are true about Wood’s Despatch?

1.   It contained the recommendations regarding the education reforms sent to Lord Canning, the then Governor General.

2.   Wood’s Dispatch strongly recommended the promotion of both technical and women’s education in India.

3.   In accordance with Wood’s despatch universities of Calcutta, Madras and Bombay were set up.

Codes:

(a)   Only 1

(b)   2 and 3

(c)   Only 3

(d)   All

Ans: C

Exp: It was sent to Lord Dalhousie for education reforms. It neglected technical and women’s education.

(PT Crash course Modern History test 2)

Source: http://iasscore.in/upsc-prelims/1857-revoltevolution-of-education-of-press-administration-before-1857

 

90. With reference to the Parliament of India which of the following Parliamentary Committees scrutinizes and reports to the House whether the powers to make regulations, rules, sub-rules, by-laws, etc. conferred by the Constitution or delegated by the Parliament are being properly exercised by the Executive within the scope of such delegation?
(a) Committee on Government Assurances
(b) Committee on Subordinate Legislation
(c) Rules Committee
(d) Business Advisory Committee
Ans: b
Exp: Every house of the parliament has a Committee on subordinate legislation whose main function is to examine the rules and regulations enacted by the executive to fill the gaps in the laws enacted by the parliament and report how far these rules are within limits prescribed in the main law. This committee has 15 members each in Rajya Sabha and Lok Sabha.

 

91. Consider the following statements:
1. As per the Right to Education (RTE) Act, to be eligible for appointment as a teacher in a State, a person would be required to posses the minimum qualification laid down by the concerned State council of Teacher Education.
2. As per the RTE Act, for teaching primary classes, a candidate is required to pass a Teacher Eligibility Test conducted in accordance with the National Council of Teacher Education Guidelines.
3. In India, more than 90% of teacher education institutions are directly under the State Governments.
Which of the statements given above is/are correct?
(a) 1 and 2
(b) 2 only
(c) 1 and 3
(d) 3 only
Ans: b
Exp: Eligibility conditions of teachers are determined by the National Council for Teacher Education.

 

92. Consider the following pairs
Tradition State
1. Chapchar Kut festival ___ Mizoram
2. Khongjom Parba ballad ___ Manipur
3. Thang-Ta dance ___ Sikkim
Which of the pairs given above is/are correct?
(a) 1 only
(b) 1 and 2
(c) 3 only
(d) 2 and 3
Ans: b
Exp: Thang Ta – is an ancient Manipuri Martial Art developed from the war environment of Manipur created by the Meitei. Thang Ta was also known as Huyen Lallong which means the art of sword and the spear.

 

93. Consider the following statements:
1. The food Safety and Standard Act, 2006 replaced the Prevention of Food Adulteration Act, 1954.
2. The Food Safety and Standards Authority of India (FSSAI) is under the charge of Director General of Health Services in the Union Ministry of Health and Family Welfare.
Which of the statements given above is/are correct?
(a) 1 only
(b) 2 only
(c) Both 1 and 2
(d) Neither 1 nor 2
Ans: a
Exp: The FSS Act is a bucket for all the older laws, rules and regulations for food safety. The FSS Act took 8 older acts into one umbrella.
1. Prevention of Food Adulteration Act, 1954
2. Fruit Products Order, 1955
3. Meat Food Products Order, 1973
4. Vegetable Oil Products (Control) Order, 1947
5. Edible Oils Packaging (Regulation) Order 1988
6. Solvent Extracted Oil, De- Oiled Meal and Edible Flour (Control) Order, 1967
7. Milk and Milk Products Order, 1992
Food Safety and Standards Authority of India (FSSAI) is an autonomous body established under the Ministry of Health, Family Welfare, Government of India. The FSSAI has been established under the Food Safety and Standards Act, 2006 which is a consolidating statute related to food safety and regulation in India. FSSAI is responsible for protecting and promoting public health through the regulation and supervision of food safety.

 

94. The term “two state solution” is sometimes mentioned in the news in the context of the affairs of
(a) China
(b) Israel
(c) Iraq
(d) Yemen
Ans: b
Exp: Two state solution is the preferred solution adopted for the Arab-Israel conflict by creation of states of Israel and Palestine.

 

95. With reference to the provisions made under the National Food Security Act, 2013, consider the following statements:
1. The families coming under the category of ‘below poverty line (BPL)’ only are eligible to receive subsidised food grains
2. The eldest woman in a household, of age 18 years or above, shall be the head of the Household for the purpose of issuance of a ratio card.
3. Pregnant women and lactating mothers are entitled to a ‘take-home ration’ of 1600 calories per day during pregnancy and for six months thereafter.
Which of the statements given above is/are correct?
(a) 1 and 2
(b) 2 only
(c) 1 and 3
(d) 3 only
Ans: b
Exp: The provisions are:
• Households covered under the Targeted Public Distribution System (TPDS) are entitled to five kilograms of foodgrains per person per month at subsidised prices.
• Households covered under Antyodaya Anna Yojana (AAY) scheme continue to be entitled to 35 kilograms per household per month, as AAY households constitute the poorest of the poor.
• Foodgrains that come under the TPDS will be available at the subsidised price of Rs. 3 per kilogram for rice, Rs. 2 per kilogram for wheat, and Re. 1 per kilogram for coarse grains or at a price fixed by the central government three years after the Act.
• Each state will identify eligible households for coverage under the TPDS. The list of eligible households will be placed in the public domain and displayed prominently.
• Pregnant women and young mothers are entitled to receive meals free of charge through the local childcare centre set up under the Integrated Child Development Services (ICDS) scheme. They are also entitled to receive maternity benefits of not less than Rs. 6,000 in instalments prescribed by the central government.
• Children aged 6 months to 6 years are entitled to age-appropriate meals free of charge through the local childcare centre set up under the ICDS.
• Children aged 6 to 14 years are entitled to one free midday meal everyday (except holidays) in government-aided schools and those run by local bodies.
• If foodgrains and meals are not supplied, entitled persons will receive a food security allowance from their state government.

• The oldest woman of the family (who must be at least 18 years of age) will be considered the head of the household when ration cards are being issued.
• Every state government must set up an internal grievance redress mechanism, which may include call centres, helplines and the designation of nodal officers.
• State governments must also appoint a District Grievance Redressal Officer for each district. This officer must enforce the Act and address the grievances of those who have been denied entitlements under the law.
• If any public sector employee or authority fails to provide the relief recommended by the District Grievance Redressal Officer, he or she will be fined a penalty of not more Rs. 5,000.

Source: http://iasscore.in/ias-prelims/public-distribution-system-in-india

 

96. India enacted The Geographical indicates of Goods (Registration and Protection) Act, 1999 in order to comply with the obligations to
(a) ILO
(b) IMF
(c) UNCTAD
(d) WTO
Ans: d
Exp: under the Uruguay and Singapore round agreement, the creation of TRIPS agreement and its expansion has brought the following under WTO rulings.

1. Copyright and Related Rights
2. Trademarks
3. Geographical Indications
4. Industrial Designs
5. Patents
6. Layout-Designs (Topographies) of Integrated Circuits
7. Protection of Undisclosed Information
8. Control of Anti-Competitive Practices in Contractual Licences

Thus the 1999 act for protection of GI was formed.

Source: http://www.iasscore.in/upsc-prelims/geographical-indication

 

97. Consider the following statements:
1. In India, State Governments do not have the power to auction non-coal mines.
2. Andhra Pradesh and Jharkhand do not have gold mines.
3. Rajasthan has iron ore mines.
Which of the statements given above is/are correct?
(a) 1 and 2
(b) 2 only
(c) 1 and 3
(d) 3 only
Ans: d
Exp: State governments have the power to auction non-coal mines. Jharkhand and Andhra Pradesh have two of the three active gold mines. Bhilwara in Rajasthan has an iron ore mine.

 

98. With reference to digital payments, consider the following statements:
1. BHIM app allows the user to transfer money to anyone with a UPI enabled bank account.
2. While a chip-pin debit card has four factors of authentication, BHIM app has only two factors of authentication.
Which of the statements given above is/are correct?
(a) 1 only
(b) 2 only
(c) Both 1 and 2
(d) Neither 1 nor 2
Ans: a
Exp: The BHIM apps has three levels of authentication. For one, the app binds with a device’s ID and mobile number, second a user needs to sync whichever bank account (UPI or non-UPI enabled) in order to the conduct transaction. Third, when a user sets up the app they are asked to create a pin which is needed to log into the app. Further, the UPI pin, which a user creates with their bank account is needed to go through with the transaction.

Source: http://www.iasscore.in/upsc-prelims/government-initiatives-in-it-sector

 

99. Among the following cities, which one lies on a longitude closest to that of Delhi?
(a) Bengaluru
(b) Hyderabad
(c) Nagpur
(d) Pune
Ans: a
Exp: Both Delhi and Bangaluru fall on same longitude

 

100. International Labour Organization’s Conventions 138 and 182 are related to
(a) Child labour
(b) Adaptation of agricultural practices to global climate change
(c) Regulation of food prices and food security
(d) Gender parity at the workplace

Ans: a
Exp: The Union Cabinet has given its approval for ratification of the two fundamental conventions of the International Labour Organization namely, Minimum Age Convention (No 138) concerning minimum age for admission to employment and the Worst Forms of Child Labour Convention (No 182) concerning the prohibition and immediate action for elimination of the worst forms of Child Labour.
India is a founder member of the International Labour Organization (ILO), which came into existence in 1919. At present, the ILO has 187 members. The principal means of action in the ILO is the setting up of International standards in the form of Conventions, Recommendations and Protocol. India has so far ratified 45 Conventions, out of which 42 are in force. Out of these 4 are Fundamental or Core Conventions.

IAS Score

Negate the Negative Marking

  • by IAS Score

Before prelims examination many Do’s and Don’ts are usually told to the aspirants related to choice of topics, number of attempts and avoidance of negative marking. Yet during the examination atmosphere, to increase the number of attempts student move for guesswork. This brings them in the vicious cycle of Negative marking. More guessing, more chances of negative marking.

This occurs due to the following reasons:

a) The Multiple Choice questions examination allows wide coverage of curriculum and due to this the length and breadth of the topics increases tremendously. Thus it becomes difficult for the students to have knowledge about everything.

b) Further due to notification of CSAT as qualifying, the entire load for passing the examination moves to General Studies. To beat the competition, it is important to attempt atleast 70 questions in Paper 1. Thus to increase the chances of passing the examination, student get tempted towards guessing the options to mark the answers.

Before moving towards the negative analysis of Negative Marking, firstly let us understand…why the concept of negative marking came up in Civil Services?

In a multiple choice test, if there is no negative marking then if an answer is correct then also it is not sure that the candidate actually knows the answer or the outcome is due to a random guess. This blurs the different between the serious and non-serious candidates…That is why negative marks concept have come up to check the entry of non-serious candidates. It discourages random guesses, and therefore the scores are more reliable.

This would give an advantage to the candidate who has a thorough knowledge of the subject and would eliminate candidates, who have merely superficial knowledge and want to score by mere guess work.

In the civil services examination, the cutoff marks are increasing year by year. The toughness of paper is increasing due to change in pattern of questions, so a slight mistake can place the student out of the race.

The total numbers of questions are 100 and to be on safe side a student should attempt atleast 60-65 questions. The proposition of guesswork lies on the number of questions attempted as well as whether it is wild guess or educated guess.

For example if a student is sure in 55 questions and want to move for guesswork, then atmost he should move for 5-7 questions as the risk will be less. Totally wild guesses should be avoided.

As in the previous year examination the number of current affairs questions were high. In such type of questions the guesswork should not be done (as either you will know the answer or not, there cannot be educated guess), whereas in the conceptual questions if the basic foundation is strong, a student can go for redundant scheme and can reduce the options to two. This will be called the educated guess.

Apart from that sometimes even the questions are easy but options become quite tricky, in hurry student fails to differentiate between the minute terms mentioned in the statements to make them redundant.

The cut-off of 2016 prelims has been disclosed by the UPSC, it is 116 whereas last year it was 107.34. As cut-off marks are increasing yearly, student should go for negating the negative marking.  However, the cut-off usually depends on the nature of questions (ratio of general/current and conceptual questions)

To negate the negative marking, the student may follow the following points:

a) The attraction towards guesswork should be completely avoided. Mind makeup should be there that one should go for maximum 4-5 guesses only as the risk of decrement in marks due to negative marking will be less.

b) Student should not loose hold of their nerves in the examination hall due to ‘mounting pressure’ syndrome which is quite natural. He/she should read the question carefully, read the guidelines such as “choose correct or incorrect statement” properly. In hurry many a times student fail to differentiate between them.

c) It is also observed that despite knowing the right answers, wrong bubbles are blackened and it is presumed to have marked the right answer. This is due to ignorance and stress. Avoid that.

d) Many students mark the answer with pencil and then in the last minute blacken the bubbles. In this hurry many a time student fails to mark all the answers or mark it incorrectly. So in the questions in which student is sure, he should blacken that at first instance only to avoid the last minute race.

e) The student should attempt all the questions in which he is sure at one go and then flip the questions second time to move for that questions in which there are confusion between the two options.

f) Do not follow elimination process for choosing an answer it may be ruinous. If you are not sure for your answer, leave them unanswered as there is no penalty for unanswered questions. Student should move for calculative risk only.

A sensible planning can bring a vast improvement in the marks, and student can greatly improve one’s marks by not falling into the traps we have discussed earlier in the article.

IAS Score

Last minute revision strategy for Prelims 2017

  • by IAS Score

Prelims examination is meant to serve as a screening test only; the marks obtained in the Preliminary Examination by the candidates who are declared qualified for admission to the Main Examination will not be counted for determining their final order of merit.

Prelims 2017 is approaching within a month and fine tuning of the preparation to clear the exam is must.

Though the UPSC has mentioned the prelims syllabus but the topics mentioned are in one word due to which clarity is very less and the range of topics covered in the examination is infinite. However, the correct preparation strategy can help the aspirant.

If we analyse the past few year papers the division of questions subject-wise is as follows:

S. No. Subject

 

2014 2015 2016
1 History and culture 20 17 15
2 Geography 14 16 7
3 Polity 14 13 7
4 Economy 10 13 18
5 Environment 18 11 18
6 Science and technology 16 8 8
7 Current affairs 8 22 27

 

By analysing the division of subjects, the current affairs portion has increased tremendously. However, the UPSC pattern cannot be predicted thus balance should be maintained while preparing the conceptual as well as the current affairs.

If we further analyse the division of subjects, there are few topics which generally get more weightage in paper.

Hereby listing few topics which cannot be ignored while preparing for Prelims:

ANCIENT HISTORY

  1. Indus Valley Civilisation
  2. Vedic Literature and Culture
  3. Jainism and Buddhism

( for this refer NCERT and NIOS).

MEDIEVAL HISTORY

  1. Focus should be on the Administrative and social work done by different rulers
  2. Bhakti and Sufi Movement

( for this refer Tamil Nadu NCERT of class XIth)

MODERN HISTORY

  1. Social Movements (objective and personality associated with it)
  2. All the important events (in form of story so that you can connect the events accordingly)

INDIAN CULTURE

  1. Architecture terminologies
  2. Classical Dance salient features
  3. Miniature Paintings (forms and features)

(Refer NCERT Class XIth Fine Arts Book)

PHYSICAL GEOGRAPHY

  1. Erosional and depostional Landforms (in geomorphology) (Refer NIOS)
  2. Climatic and vegetation division of the world (Refer class 7th NCERT for few divisions)
  3. Ocean currents (significance and presence Ocean-wise) (refer class XIth NCERT)

INDIAN GEOGRAPHY

  1. Physical features
  2. Drainage system
  3. Monsoon
  4. Soil and vegetation division
  5. Agriculture (crops basic characteristics)
  6. Latest Agriculture schemes
  7. Minerals locations

(Refer NCERT and NIOS for the above stated topics)

HUMAN GEOGRAPHY

  1. Data related to Census 2011 and terms associated
  2. Types of human settlements

(Refer http://iasscore.in/upsc-prelims/human-settlement-population-terms-census-2011)

POLITY

  1. Provisions related to Fundamental Rights and DPSPs
  2. Provisions related to the Executive and Legislature
  3. Other provisions which are frequent in news as
  4. NJAC and collegiums
  5. Official language provisions
  6. AFSPA and emergency provisions
  7. Election Commission
  8. About 14th Finance Commission
  9. About 7th Pay commission
  10. About Law Commission (basic provisions)
  11. Judiciary terms as – Arbitration, PIL, Gram Nyalayas, Para-Legal Volunteers, Lok Adalat)
  12. Niti Aayog, etc

WELFARE AND SOCIAL ISSUES

  1. Initiatives by the Ministries in last one year (Refer http://pib.nic.in/newsite/docpage.aspx?docid=553). This will help in revising all the important schemes and governance steps initiated by the government.

ENVIRONMENT

  1. Biodiversity – species, red book data, types of interactions, etc. (Refer NIOS)
  2. National Parks and their locations
  3. Pollution (types and new rules)
  4. Climate change (all the terms associated with it and steps initiated by India)

(Refer http://envfor.nic.in/sites/default/files/Environment%20AR%20English%202016-2017.pdf)

SCIENCE AND TECHNOLOGY

  1. In biology focus on Diseases and Nutrition
  2. Recent updates in Defence, Information Technology, Space and Biotechnology (mostly the questions are from these areas)

ECONOMY

  1. Basic economic definitions (clarity should be there in terms so that as aspirant can choose the correct option properly. (refer http://iasscore.in/free-downloads/study-material-for-revision-notes-economic-terminologies)
  2. National Income (refer class XIIth NCERT)
  3. Terms in the Union Budget and Economic Survey
  4. Latest schemes
  5. Banking Terms in detail (bad banks concept, shadow banking, payment banks, small banks, etc)

Hope this will help in preparing for Prelims 2017.

MAYURI KHANNA

IAS Score

Prelims Answer Key 2016, IAS PT 2016 Answer Key with Explanation

  • by IAS Score

Prelims 2016 Answer Key Set A

Q. 1. Which of the following statements is/are correct?

1. A bill pending in the Lok Sabha lapses on its prorogation.
2. A bill pending in the Rajya Sabha, which has not been passed by the Lok Sabha, shall not lapse on dissolution of the Lok Sabha.

Select the correct answer using the code given below:

(a) 1 only
(b) 2 only
(c) Both 1 and 2
(d) Neither 1 nor 2

Ans: b

Exp: A bill pending in the Rajya Sabha but not passed by the Lok Sabha does not lapse. Similarly due to prorogation bills do not lapses. Prorogation terminates a session of the House only.

Q. 2. Which of the following is/are the indicator/indicators used by IFPRI to compute the Global Hunger Index Report?

1. Undernourishment
2. Child stunting
3. Child mortality

Select the correct answer using the code given below:

(a) 1 only
(b) 2 and 3 only
(c) 1, 2 and 3
(d) 1 and 3 only

Ans: c

Exp: The GHI combines 4 component indicators: 1) the proportion of the undernourished as a percentage of the population; 2) the proportion of children under the age of five suffering from wasting; 3) the proportion of children under the age of five suffering from stunting; 4) the mortality rate of children under the age of five.

Q. 3. There has been a persistent deficit budget year after year. Which action/actions of the following can be taken by the Government to reduce the deficit?

1. Reducing revenue expenditure
2. Introducing new welfare schemes
3. Rationalizing subsidies
4. Reducing import duty

Select the correct answer using the code given below.

(a) 1 only
(b) 2 and 3 only
(c) 1 and 3 only
(d) 1,2,3 and 4

Ans: c

Exp: Reducing revenue expenditure and rationalizing subsidies will reduce the expenditure of the government. Whereas introducing new welfare schemes and reducing import duty will add additional burden to the government resources.

Q. 4. The establishment of “Payment Banks’ is being allowed in India to promote Financial inclusion. Which of the following statements is/are correct in this context?

1. Mobile telephone companies and supermarket chains that are owned and controlled by residents are eligible to be promoters of Payment Banks.
2. Payment Banks can issue both credit cards and debit cards.
3. Payment Banks cannot undertake lending activities.

Select the correct answer using the code given below:

(a) 1 and 2 only
(b) 1 and 3 only
(c) 2 only
(d) 1, 2 and 3

Ans: b

Exp: Existing non-bank Pre-paid Payment Instrument (PPI) issuers; and other entities such as individuals / professionals; Non-Banking Finance Companies (NBFCs), corporate Business Correspondents(BCs), mobile telephone companies, super-market chains, companies, real sector cooperatives; that are owned and controlled by residents; and public sector entities may apply to set up payments banks.
Payment Banks can issue ATM/ debit cards but not credit cards. Payment Banks can NOT give loans.

Q. 5. With reference to ‘LiFi’, recently in the news, which of the following statements is/are correct?

1. It uses light as the medium for high-speed data transmission.
2. It is a wireless technology and is several time faster than ‘WiFi.

Select the correct answer using the code given below:

(a) 1 only
(b) 2 only
(c) Both 1 and 2
(d) Neither 1 nor 2

Ans: c

Exp: Light Fidelity (Li-Fi)—a revolutionary new technology that transmits high-speed data using light. It is touted as being nearly 100 times faster than the traditional Wi-Fi technology based on transmission of radio waves. It uses visible-light communication or infrared and near-ultraviolet instead of radio-frequency spectrum, part of optical wireless communications technology, which carries much more information

Q. 6. The term Intended Nationally Determined Contribution is sometimes seen in the news in the context of:

a) Pledge made by the European countries to rehabilitate refuges from the war-affected Middle East.
b) Plain of nation outlined by the countries of the world to combat climate changes.
c) Capital contributed by the member countries in the establishment of Asian Infrastructure Investment Bank.
d) Plain of action outlined by the countries of the regarding Sustainable Developments Goals.

Ans: b

Exp: INDCs are the primary means for governments to communicate internationally the steps they will take to address climate change in their own countries. INDCs will reflect each country’s ambition for reducing emissions, taking into account its domestic circumstances and capabilities for achieving sustainable development goals.

Q. 7. Which one of the following is a purpose of ‘UDAY’, a scheme of the Government?

a) Providing technical and financial assistance to start-up entrepreneurs in the field or renewable source of energy.
b) Providing electricity to every household in the country by 2018
c) Replacing the coal-based power plants with natural gas, nuclear, solar, wind and tidal power plants over a period of time.
d) Providing for financial turnaround and revival of power distribution companies

Ans: d

Exp: UDAY (Ujwal DISCOM Assurance Yojana) scheme to provide a permanent solution for financial turnaround and revival of Power Distribution companies (discoms).

Q. 8. With reference to ‘IFC Masala Bonds’, sometimes given below is/are correct?

1. The International Financed Corporation, with offers these bonds, is an arm of the World Bank.
2. They are the rupee-denominated bonds and are a source of debt financing for the public and private sector.

Select the correct answer using the code given below.

(a) 1 only
(b) 2 Only
(c) Both 1 and 2
(d) Neither 1 nor 2

Ans: a

Exp: International Finance Corporation, the private-sector financing arm of World Bank, has launched rupee-denominated offshore bonds for Rs 1,600 crore, or about $250 million, attracting first-time investors to overseas rupee markets.

Q. 9. Regarding the taxation system of Krishna Deva, the ruler of Vijayanagar, consider the following statements :

1. The tax rate on land was fixed depending on the quality of the land.
2. Private owners of workshops paid an industries tax.

Which of the statements given above is/are correct?

(a) 1 only
(b) 2 Only
(c) Both 1 and 2
(d) Neither 1 nor 2

Ans: c

Exp: Land revenue was the chief source of income. Land was divided into four categories for purposes of assessment, wet land, dry land, orchards and woods. Usually the share was one sixth of the produce. Land revenue could be paid in cash or kind. The rates varied according to the type of the crops, soil, method of irrigation, etc.

Besides land tax, many professional taxes were also imposed. There were on shopkeepers, farm servants, workmen, posters, shoemakers, musicians etc. There was also a tax on property. Grazing and house taxes were also imposed. Commercial taxes consisted of levies, duties and customs on manufactured articles of trade were also levied. Private owners of workshops paid an industries tax.

Q. 10. Which one of the following books of ancient India has the love story of the son of the founder of Sunga dynasty?

(a) Swapnavasavadatta
(b) Malavikagnimitra
(c) Meghadoota
(d) Ratnavali

Ans: b

Q. 11. In the context of which of the following do you sometimes find the terms ‘amber box, blue box and green box’ in the news?

(a) WTO affairs
(b)SAARC affairs
(c)UNFCCC affairs
(d) India-EU negotiations on FTA

Ans: a

Exp: These are WTO jargon for different kinds of subsidies given to agriculture. In a nutshell, amber box subsidies constitute all forms of domestic support deemed to be trade distorting, primarily by encouraging excessive production. Blue box subsidies are considered somewhat less trade distorting, because while they directly link production to subsidies, they also set limits on production by way of quotas, for instance. Green box subsidies were initially considered non-distorting in terms of production and trade, though it is increasingly being recognized that they are at best minimally trade distorting. Direct income support schemes unlinked to production would be typical examples of green box subsidies.

Q. 12. Which of the following is/are included in the capital budget of the Government of India?

1. Expenditure on acquisition of assets like roads, buildings, machinery, etc.
2. Loans received from foreign governments.
3. Loans and advances granted to the States and Union Territories.

Select the correct answer using the code given below.

(a) 1 only
(b) 2 and 3 only
(c) 1 and 3 only
(d) 1, 2 and 3

Ans: d

Exp: Capital Budget consists of capital receipts and payments. Capital receipts are loans raised by the government from the public (which are called market loans), borrowings by the government from the Reserve Bank and other parties through sale of treasury bills, loans received from foreign bodies and governments, and recoveries of loans granted by the Central government to state and Union Territory governments and other parties. Capital payments consist of capital expenditure on acquisition of assets like land, buildings, machinery, and equipment, as also investments in shares, loans and advances granted by the Central government to state and Union Territory governments, government companies, corporations and other parties.

Q. 13. What is/are the importance/importances of the ‘United Nations Convention to Combat Desertification?

1. It aims to promote effective action through innovative national programmes and supportive inter-national partnerships.
2. It has a special/particular focus on South Asia and North Africa regions, and its secretariat facilitates the allocation of major portion of financial resources to these regions.
3. It is committed to bottom-up approach, encouraging the participation of local people in combating the desertification.

Select the correct answer using the code given below.

(a) 1 only
(b) 2 and 3 only
(c) 1 and 3 only
(d) 1, 2 and 3

Ans: c

Exp: The convention aims to promote effective action through innovative national programmes and supportive international partnerships. committed to a bottom-up approach, encouraging the participation of local people in combating desertification and land degradation.

Q. 14. Recently, which one of the following currencies has been proposed to be added to the basket of IMF’s SDR?

(a) Rouble
(b) Rand
(c) Indian Rupee
(d) Renminbi

Ans: d

Exp: Chinese renminbi (RMB) will be added as the fifth currency in IMF’s SDR.

Q. 15. With reference to the International Monetary and Financial Committee (IMFC), consider the following statements:

1. IMFC discusses matters of concern affecting the global economy, and advises the International Monetary Fund (IMF) on the direction of its work.
2. The World Bank participates as observer in IMFC’s meetings.

Which of the statements given above is/are correct?

(a) 1 only
(b) 2 only
(c) Both 1 and 2
(d) Neither 1 nor 2

Ans: c

Exp: The IMFC advises and reports to the IMF Board of Governors on the supervision and management of the international monetary and financial system, including on responses to unfolding events that may disrupt the system. It also considers proposals by the Executive Board to amend the Articles of Agreement and advises on any other matters that may be referred to it by the Board of Governors. Although the IMFC has no formal decision-making powers, in practice, it has become a key instrument for providing strategic direction to the work and policies of the Fund.

World Bank acts as observer in the meetings.

Q. 16. ‘Rashtriya Garima Abhiyaan’ is a national campaign to
(a) rehabilitate the homeless and destitute persons and provide then with suitable sources of livelihood
(b) release the sex workers from the practice and provide them with alternative sources of livelihood
(c) eradicate the practice of manual scavenging and rehabilitate the manual scavengers
(d) release the bonded labourers free their bondage and rehabilitate them

Ans: c

Exp: it is based on Eradication of inhuman practice of manual scavenging and comprehensive rehabilitation of manual scavengers in India.

Q. 17. With reference to the cultural history of medieval India, consider the following statements:

1. Siddhas (Sittars) of Tamil region were monotheistic and condemned idolatry.
2.Lingayats of Kannada region questioned the theory of rebirth and rejected the caste hierarchy

Which of the statements given above is/are correct?

(a) 1 only
(b) 2 only
(c) Both 1 and 2
(d) Neither 1 nor 2

Ans: To be updated…

Q. 18. Which of the following best describes the term “import cover”, sometimes seen in the news?

(a) It is the ratio of value of imports to the Gross Domestic product of a country
(b) It is the total value of imports of a country in a year.
© It is the ratio between the value of exports and that of imports between two countries
(d)It is the number of months of imports that could be paid for by a country’s international reserves

Ans: d

Q. 19. Consider the following pairs:

Community sometimes mentioned in the news In the affairs of

1. Kurd : Bangladesh
2. Madhesi : Nepal
3. Robingya : Myanmar

Which of the pairs given above is/are correctly matched?

(a) 1 and 2
(b) 2 only
(c) 2 and 3
(d) 3 only

Ans: c

Exp: Kurds inhabit a mountainous region straddling the borders of Turkey, Iraq, Syria, Iran and Armenia. They make up the fourth-largest ethnic group in the Middle East.

Q. 20. With reference to ‘Organization for the prohibition of Chemical Weapons (OPCW)’ consider the following statements:

1. It is an organization of European Union in working relation with NATO and WHO
2. It monitors chemical industry to prevent new weapons free emerging
3. It provides assistance and protection to Stated Parties against chemical weapons threats.

Which of the statements given above is/are correct?

(a) 1 only
(b) 2 and 3 only
(c) 1 and 3 only
(d) 1,2 and 3

Ans: b

Exp: The Organisation for the Prohibition of Chemical Weapons (OPCW) is an intergovernmental organisation not under European Union.

Q. 21. With reference to ‘Pradhan Mantri Fasal Bima Yojana’, consider the following statements:

1. Under this scheme, farmers with have to pay a uniform premium of two percent for any crop they cultivate in any season of the year.
2. This scheme covers post-harvest losses arising out of cyclones and unseasonal rains.

Which of the statements given about is/are correct?

(a) 1 only
(b) 2 only
(c) Both 1 and 2
(d) Neither 1 nor 2

Ans: b

Exp: Under the scheme there will be a uniform premium of only 2% to be paid by farmers for all Kharif crops and 1.5% for all Rabi crops. In case of annual commercial and horticultural crops, the premium to be paid by farmers will be only 5%. Post-harvest losses arising out of cyclone and unseasonal rain have been covered nationally.

Q. 22. In which of the following regions of India are you most likely to come across the ‘Great Indian Hornbill’ in its natural habitat?

a) Sand deserts of northwest India
b) Higher Himalayas of Jammu and Kashmir
c) Salt marshes of western Gujarat
d) Western Ghats

Ans: d

Exp: India has nine hornbill species, of which four are found in the Western Ghats: Indian Grey Hornbill (endemic to India), the Malabar Grey Hornbill (endemic to the Western Ghats), Malabar Pied Hornbill (endemic to India and Sri Lanka) and the widely distributed but endangered Great Hornbill. India also has one species that has one of the smallest ranges of any hornbill: the Narcondam Hornbill, found only on the island of Narcondam.

Q. 23. Which of the following are the key features of ‘National Ganga River Basin Authority (NGRBA)’?

1. River basin is the unit of planning and management.
2. It spearheads the river conservation efforts at the national level.
3. One of the Chief Ministers of the State through which the Ganga flows becomes the Chairman of NGRBA on rotation basis.

Select the correct answer using the code given below.

(a) 1 and 2 only
(b) 2 and 3 only
(c) 1 and 3 only
(d) 1, 2 and 3

Ans: a

Exp: National Ganga River Basin Authority (NGRBA) will spearhead river conservation efforts at the national level. Implementation will be by the State Agencies and Urban Local Bodies. It is a unit of planning and management.

Q. 24. Why does the Government of India promote the use of ‘Neem-coated Urea’ in agriculture?

(a) Release of Neem oil in the soil increases nitrogen fixation by the soil microorganisms
(b) Neem coating slows down the rate of dissolution of urea in the soil
(c) Nitrous oxide, which is a greenhouse gas, is not at all released into atmosphere by crop fields
(d) It is a combination of a weedicide and a fertilizer for particular crops

Ans: b

Exp: When ordinary urea is applied, it gets converted to ammonium carbamate. Some of this gets converted to ammonia gas in what is called ammonia volatilisation. The rest of the ammonium carbamate undergoes chemical transfor-mation and nitrates are formed. Some of these are absorbed by the plants. The rest are either leached into the underground water or are denitrified to gaseous nitrogen and nitrous oxide under anaerobic conditions (absence of oxygen). Neem has properties that check nitrogen loss at each stage. It slows down the process of nitrate formation and hence excess nitrate is not available for denitrification.

Q. 25. Consider the following statements:

1. The Chief Secretary in a State is appointed by the Governor of that State.
2. The Chief Secretary in a State has a fixed tenure

Which of the statements given above is/are correct?

(a) 1 only
(b) 2 only
(c) Both 1 and 2
(d) Neither 1 nor 2

Ans: d

Exp: The chief secretary is the head of the entire state secretariat. S/he is appointed by the Chief Minister. It has no fixed tenure.

26. With reference to ‘Stand Up India Scheme’, which of the following statements is/are correct?

1. Its purpose is to promote entrepreneurship among SC/ST and women entrepreneurs.
2. It provides for refinance through SIDBI.

Select the correct answer using the code given below.

(a) 1 only
(b) 2 only
(c) Both 1 and 2
(d) Neither 1 nor 2

Ans: c

Exp: The “Stand up India Scheme” is being launched now to promote entrepreneurship among Scheduled Caste/Schedule Tribe and Women for loans in the range of Rs. 10 Lakhs to Rs. 100 Lakhs. The Scheme is expected to benefit large number of such entrepreneurs, as it is intended to facilitate at least two such projects per bank branch (Scheduled Commercial Bank) on an average one for each category of entrepreneur. Refinance window through Small Industries Development Bank of India (SIDBI) with an initial amount of Rs. 10,000 crore will be provided.

Q. 27. The FAO accords the status of ‘Globally Important Agricultural Heritage Systems (GIAHS)’ to traditional agricultural systems. What is the overall goal of this initiative?

1. To provide modern technology, training in modern farming methods and financial support to local communities of identified GIAHS so as to greatly enhance their agricultural productivity
2. To identify and safeguard eco-friendly traditional farm practices and their associated landscapes, agricultural biodiversity and knowledge systems of the local communities
3. To provide Geographical Indication status to all the varieties of agricultural produce in such indentified GIAHS

Select the correct answer using the code given below.

(a) 1 and 3 only
(b) 2 only
(c) 2 and 3 only
(d) 1, 2 and 3

Ans: b

Q. 28. Which of the following is/are tributary/tributaries of Brahmaputra?
1. Dibang
2. Kameng
3. Lohit

Select the correct answer using the code given below:

(a) 1 only
(b) 2 and 3 only
(c) 1 and 3 only
(d) 1, 2 and 3

Ans: d

Exp: Tributaries- Dibang River, Lohit River, Dhansiri River, Kolong River, Kameng River, Manas River, Raidak River, Jaldhaka River, Teesta River, Subansiri River.

Q. 29. The term ‘Core Banking Solution’ is sometimes such in the news. Which of the following statements best describes/describe this term?

1. It is a networking of a bank’s branches which enables customers to operate their accounts from any branch of the bank on its network regardless of where they open their accounts.
2. It is an effort to increase RBI’s control over commercial banks through computerization.
3. It is a detailed procedure by which a bank with huge non-performing assets is taken over by another bank.

Select the correct answer using the code given below:

(a) 1 only
(b) 2 and 3 only
(c) 1 and 3 only
(d) 1, 2 and 3

Ans: A

Exp: Core Banking Solution (CBS) is networking of branches, which enables Customers to operate their accounts, and avail banking services from any branch of the Bank on CBS network, regardless of where he maintains his account. The customer is no more the customer of a Branch.

Q. 30. Consider the following pairs:

Terms sometimes seen in the news Their origin

1. Annex-I Countries : Cartagena Protocol
2. Certified Emissions Reductions : Nagoya Protocol
3. Clean Development Mechanism : Kyoto Protocol

Which of the pairs given above is/are correctly matched?

(a) 1 and 2 only
(b) 2 and 3 only
(c) 3 only
(d) 1, 2 and 3

Ans: C

Exp: All the terms are associated with Kyoto Protocol. Certified Emission Reductions (CERs) are a type of emissions unit (or carbon credits) issued by the Clean Development Mechanism (CDM) Executive Board for emission reductions achieved by CDM projects and verified by a DOE (Designated Operational Entity) under the rules of the Kyoto Protocol.

Annex I parties are industrialized countries belonging to the Organization for Economic Cooperation and Development (OECD) and countries designated as Economies in Transition under the Framework Convention on Climate Change (FCCC), that pledged to reduce their greenhouse gas emissions to 1990 levels by the year 2000.

Q. 31. In the context of the developments in bioinformatics, the term ‘transcriptome’, sometimes seen in the news, refer to

(a) a range of enzymes used in genome editing
(b) the full range of mRNA molecules expressed by an organism
(c) the description of the mechanism of gene expression
(d)A mechanism of genetic mutation taking place in cells

Ans: b

Exp: The sum total of all the messenger RNA molecules expressed from the genes of an organism.

Q. 32. ‘Mission Indradhanush’ launched by the Government of India pertains to

(a) Immunization of children and pregnant women
(b) Construction of smart cities across the country
(c) India’ own search for the Earth-like planets in outer space
(d) New Educational policy

Ans: a

Exp: Mission Indradhanush aims to ensure full immunization with all available vaccines for children up to two years and pregnant women.

Q. 33. Which of the following best describe the aim of ‘Green India Mission’ of the Government of India?

1. Incorporating environment benefits and costs into the Union and State Budgets thereby implementing the ‘green accounting’.
2. Launching the second green revolution to enhance agriculture output so as to ensure food security to one and all in the future.
3. Restoring and enhancing forest cover and responding to climate change by a combination of adaptation and mitigation measures.

Select the correct answer using the code given below

(a) 1 only
(b) 2 and 3 Only
(c) 3 Only
(d) 1, 2 and 3

Ans: c

Exp: The objectives are:

• To increase forest/tree cover to the extent of 5 million hectares (mha) and improve quality of forest/tree cover on another 5 mha of forest/non-forest lands;
• To improve/enhance eco-system services like carbon sequestration and storage (in forests and other ecosystems), hydrological services and biodiversity; along with provisioning services like fuel, fodder, and timber and non-timber forest produces (NTFPs); and
• To increase forest based livelihood income of about 3 million households.

Q. 34. With reference to pre-packaged items in India, it is mandatory to the manufacturer to put which of the following information on the main label, as per the Food Safety and Standards (Packaging and Labeling) Regulations, 2011?

1. List of ingredients including additives
2. Nutrition information
3. Recommendations, if any made by the medical profession about the possibility of any allergic reactions
4. Vegetarian/non-vegetarian

Select the correct answer using the code given below

a) 1, 2 and 3
b) 2, 3 and 4
c) 1, 2 and 4
d) 1 and 4 Only

Ans: c

Q. 35. ‘Project Loon’, sometimes seen in the news, is related to

a) Waste management technology
b) Wireless communication technology
c) Solar power production technology
d) Water conservation technology

Ans: b

Exp: Project Loon is a research and development project being developed by X(formerly Google X) with the mission of providing Internet access to rural and remote areas.

Q. 36. ‘Net metering’ is sometimes seen in the news in the context of promoting the

a) production and use of solar energy by the households/consumers
b) use of piped natural gas in the kitchens of households
c) installation of CNG kits in motor- cars
d) installation of water meters in urban households

Ans: a

Exp: Net metering is a billing mechanism that credits solar energy system owners for the electricity they add to the grid.

Q. 37. India’s ranking in the ‘Ease of Doing Business Index’ is sometimes seen in the news. Which of the following has declared that ranking?

a) Organization for Economic Cooperation and Development (OECD)
b) World Economic Forum
c) World Bank
d) World Trade Organization (WTO)

Ans: c

Q. 38. Banjaras during the medieval period of Indian history were generally

a) Agriculturists
b) Warriors
c) Weaves
d) Traders

Ans: d

Q. 39. Who of the following had first deciphered the edicts of Emperor Ashoka?
a) Georg Buhier
b) James Prinsep
c) Max Muller
d) William Jones

Ans: b

Q. 40. With reference to the ‘Gram Nyayalaya Act’, which of the following statements is/are correct?
1. As per the Act, Gram Nyayalayas can hear only civil cases and not criminal cases
2. The Act allows local social activists as mediators/reconciliators.

Select the correct answer using the code given below:

(a) 1 only
(b) 2 only
(c) Both 1 and 2
(d) Neither 1 nor 2

Ans: b

Exp: As per the Act, Gram Nyayalayas can hear both criminal and civil cases and appeals in civil cases will have to be disposed of in six months. The Act also makes the judicial process participatory and decentralized because it allows appointment of local social activists and lawyers as mediators/reconciliators.

Q. 41. With reference to the ‘Trans-Pacific Partnership’, consider the following statements:

1. It is an agreement among all the Pacific Rim countries except China and Russia.
2. It is a strategic alliance for the purpose of maritime security only.

Which of the statements given above is/are correct?

(a) 1 only
(b) 2 only
(c) Both 1 and 2
(d) Neither 1 nor 2

Ans: d

Exp: The TPP involves 12 Pacific Rim nations – Australia, Brunei, Canada, Chile, Japan, Malaysia, Mexico, New Zealand, Peru, Singapore, the United States and Vietnam. The pact aims to deepen economic ties between these nations, slashing tariffs and fostering trade to boost growth.

Q. 42. Consider the following statements:

The India-Africa Summit

1. Held in 2015 was the third such Summit
2. Was actually initiated by Jawaharlal Nehru in 1951

Which of the statements given above is/are correct?

(a) 1 only
(b) 2 only
(c) Both 1 and 2
(d) Neither 1 nor 2

Ans: a

Exp: It was first held in 2008.

Q. 43. What is/are the purpose/purposes of the ‘Marginal Cost of Funds based Lending Rate (MCLR)’ announced by RBI?

1. These guidelines help improve the transparency in the methodology followed by banks for determining the interest rates on advances
2. These guidelines help ensure availability of bank credit & interest rates which are fair to the borrowers as well as the banks

Select the correct answer using the code given below:

(a) 1 only
(b) 2 only
(c) Both 1 and 2
(d) Neither 1 nor 2

Ans: c

Exp: MCLR help in improving the transmission of policy rates into the lending rates of banks, these guidelines are expected to improve transparency in the methodology followed by banks for determining interest rates on advances. These guidelines are also expected to ensure availability of bank credit at interest rates which are fair to the borrowers as well as the banks. Further, marginal cost of pricing of loans will help the banks to become more competitive and enhance their long run value and contribution to economic growth.

Q. 44. What is/are unique about ‘Kharai camel’ a breed found in India?

1. It is capable of swimming up to three kilometers in seawater.
2. It survives by grazing on mangrows.
3. It lives in the wild and cannot be domesticated.

Selected the correct answer using the code given below:

(a) 1 and 2 only
(b) 3 only
(c) 1 and 3 only
(d) 1, 2 and 3

Ans: d

Exp: Kharai Camel or Swimming Camels are found only in Gujarat’s Bhuj area. It has been recently recognized as a separate breed (one among nine such breeds found in India) of camel for better conservation. This camel is adapted to the extreme climate of Ran of Kachh where shallow seas and high salinity is prevalent. Kharai Camel can live in both coastal and dry ecosystems. It grazes on saline / mangrove trees and is tolerant to high saline water. It can swim up to three kilometers into the sea in search of mangroves, their primary food. The camel is distinct from other camels because of its rounded back, long and thin legs and small feet.

Q. 45. Recently, our scientists have discovered a new and distinct species of banana plant which attains a height of about 11 metres and has orange-coloured fruit pulp. In which part of India has it been discovered?

(a) Anadaman Islands
(b) Anamalai Forests
(c ) Maikala Hills
(d)Tropical rain forest of northeast

Ans: a

Exp: The species have been found in Andaman and nicobar. The new species is about 11 metres high, whereas as the usual banana species is about three to four metres high. The fruit lux of the new species is about one metres, which is thrice the size of regular species.

Q. 46. Which one of the following is the best description of ‘INS Astradharini’, that was in the news recently?

(a) Amphibious warfare ship
(b) Nuclear-powered submarine
(c) Torpedo launch and recovery vessel
(d) Nuclear-powered aircraft carrier

Ans: c

Exp: It is Indian Navy’s first totally indigenously-designed and built torpedo launch and recovery vessel.

47. What is ‘Greased Lightning-10 (GL-10)’, recently in the news?

(a) Electric plane tested by NASA
(b) Solar-powered two-seater aircraft designed by Japan
(c) Space observatory launched by China
(d) Reusable rocket designed by ISRO

Ans: a

Exp: The GL-10 or Greased Lightning is a 28kg, battery-powered drone prototype that can switch between helicopter and airplane modes, giving the aircraft the maneuverability and vertical take-off capabilities of a chopper, as well as the speed and endurance of a plane designed by NASA.

Q. 48. With reference to ‘Initiative for Nutritional Security through intensive Millets Promotion’, which of the following statements is/are correct?

1. This initiative aims to demonstrate the improved production and post-harvest technologies, and to demonstrate value addition techniques, in an integrated manner, with cluster approach.
2. Poor, small, marginal and tribal farmers have larger stake in this scheme.
3. An important objective of the scheme is to encourage farmers of commercial crops to shift to millet cultivation by offering them free kits of critical inputs of nutrients and micro-irrigation equipment.

Select the correct answer using the code given below.

(a) 1 only
(b) 2 and 3 only
(c) 1 and 2 only
(d) 1, 2 and 3

Ans: a

Exp: The scheme aims to demonstrate the improved production and post-harvest technologies in an integrated manner with visible impact to catalyze increased production of millets in the country. Besides increasing production of millets, the Scheme through processing and value addition techniques is expected to generate consumer demand for millet based food products.

Q. 49. The ‘Swadeshi’ and ‘Boycott’ adopted as methods of struggle for the first time during the

(a) agitation against the Partition of Bengal
(b) Home Rule Movement
(c) Non-Cooperation Movement
(d) visit of the Simon Commission to India

Ans: a

Q. 50. With reference to the religious history of India, consider the following statements:

1. The concept of Bodhisattva is central to Hinayana sect of Buddhism.
2. Bodhisattva is a compassionate one on his way to enlightenment.
3. Bodhisattva delays achieving his own salvation to help all sentient beings on their path to it.

Which of the statements given above is/are correct?

(a) 1 only
(b) 2 and 3 only
(c) 2 only
(d) 1, 2 and 3

Ans: b

Exp: Bodhisattva is central to Mahayana Sect.

Q. 51. ‘Doctors Without Borders (Medecins Sans Frontiercs)’, often in the news, is

a) A division of World Health Organization
b) A non-governmental international organization
c) And inter-government agency sponsored by European Union
d) A specialized agency of the United Nations

Ans: b

Exp: Doctors Without Borders, is an international humanitarian-aid non-governmental organization (NGO) and Nobel Peace Prize laureate, best known for its projects in war-torn regions and developing countries facing endemic diseases.

Q. 52. With reference to an initiative called ‘The Economics of Ecosystems and Biodiversity (TEEB)’, which of the following statements is/are correct?

1. It is initiative hosted by UNEP, IMF and World Economic Forum.
2. It is a global initiative that focuses on drawing attention to the economic benefits of biodiversity.
3. It presents an approach that can help decision-makers recognize, demonstrate and capture the value of ecosystems and biodiversity.

Select the correct answer using the code given below

a) 1 and 2 only
b) 3 Only
c) 2 and 3 only
d) 1, 2 and 3

Ans: c

Exp: The Economics of Ecosystems and Biodiversity (TEEB) is an initiative hosted by the United Nations Environment Programme (UNEP).

Q. 53. With reference to ‘Red Sanders’, sometimes seen in the news, consider the following statements:

1. It is a tree species found in a part of South India.
2. It is one of the most important trees in the tropical rain forest areas of South India.

Which of the statements given above is/are correct?

a) 1 only
b) 2 Only
c) Both 1 and 2
d) Neither 1 nor 2

Ans: a

Exp: It is a species of thorny deciduous region.

54. Which of following statements is/are correct?
Proper design and effective implementation of UN-REDD+ Programme can significantly contribute to

1. Protection of biodiversity
2. Resilience of forest ecosystems
3. Poverty reduction

Select the correct answer using the code given below

a) 1 and 2 only
b) 3 Only
c) 2 and 3 only
d) 1, 2 and 3

Ans: a

Q. 55. What is ‘Greenhouse Gas Protocol?

a) It is an international accounting tool for government and business leaders to understand, quantify and manage greenhouse gas emissions
b) It is an initiative of the United Nations to offer financial incentives to developing countries to reduce greenhouse gas emissions and to adopt eco-friendly technologies
c) It is an inter-governments agreement ratified by all the member countries of the United Nations to reduce greenhouse gas emissions to specified levels by the year 2022
d) It is one of the multilateral REDD-initiatives hosted by the World Bank

Ans: a

Exp: The Greenhouse Gas Protocol (GHGP) provides accounting and reporting standards, sector guidance, calculation tools, and trainings for business and government. It establishes a comprehensive, global, standardized framework for measuring and managing emissions from private and public sector operations, value chains, products, cities, and policies.

Q. 56. With reference to ‘Financial Stability and Development Council’, consider the following statements:

1. It is an organ of NITI Aayog.
2. It is headed by the Union Finance Minister.
3. It monitors macro prudential super-vision of the economy.

Which of the statements given above is/are correct?

a) 1 and 2 only
b) 3 Only
c) 2 and 3 only
d) 1, 2 and 3

Ans: c

Exp: The Chairman of the Council is the Finance Minister and its members include the heads of financial sector Regulators (RBI, SEBI, PFRDA, IRDA & FMC {now with SEBI}) Finance Secretary and/or Secretary, Department of Economic Affairs, Secretary, Department of Financial Services, and Chief Economic Adviser. The Council can invite experts to its meeting if required. The FSDC Secretariat is in the Department of Economic Affairs.

The council will act as a co-ordination agency between the various financial sector regulators- the RBI, SEBI, IRDA and the PFRDA. This Council would monitor macro-prudential supervision of the economy, including the functioning of large financial conglomerates, and address inter-regulatory coordination issues.

Q. 57. With reference to ‘Agenda 21’, sometimes seen in the news, consider the following statements:

1. It is a global action plan for sustainable development.
2. It originated in the World Summit on Sustainable Development held in Johannesburg in 2002

Which of the statements given above is/are correct?

(a) 1 only
(b) 2 only
(c) Both 1 and 2
(d) Neither 1 nor 2

Ans: a

Exp: It was initiated in Earth Summit (UN Conference on Environment and Development) held in Rio de Janeiro, Brazil, in 1992.

Q. 58. Satya Shodhak Samaj organized

(a) a movement for upliftment of tribals in Bihar
(b) a temple-entry movement in Gujarat
(c ) an anti-caste movement in Maharashtra
(e) A peasant movement in Punjab

Ans: c

Exp: The main aim was to liberate the social shudra and untouchable castes from exploitation and oppression.

Q. 59. Which of the following statements is/are correct?

Viruses can infect

1. Bacteria
2. Fungi
3. Plants

Select the correct answer using the code given below:

(a) 1 and 2 only
(b) 3 only
(c) 1 and 3 only
(d) 1, 2 and 3

Ans: d

Q. 60. The term ‘Base Erosion and profit shifting’ is sometimes seen in the news in the context of

(a) mining operation by multinational companies in resource-rich by backward areas
(b) curbing of the tax evasion by multinational companies
(c )exploitation of genetic resources of a country by multinational companie
(d ) Lack of consideration of environmental costs in the planning of development projects

Ans: b

Exp: Base erosion and profit shifting (BEPS) refers to tax avoidance strategies that exploit gaps and mismatches in tax rules to artificially shift profits to low or no-tax locations.

Q. 61. Recently, India’s first ‘National Investment and Manufacturing Zone’ was proposed to be set up in

(a) Andhra Pradesh
(b) Gujarat
(c ) Maharashtra
(d)Uttar Pradesh

Ans: a

Exp: Andhra Pradesh is set to house India’s first national investment and manufacturing zone. The NIMZ will comprise a special purpose vehicle and a developer, along with the state and the central governments as stakeholders.

Q. 62. What is/are the purpose of ‘District Mineral Foundations” in India?

1. Promoting mineral explorative activities in mineral-rich districts
2. Protecting the interests of the persons affected by mining operations
3. Authorizing State Governments to issue licenses for mineral exploration

Select the correct answer using the code given below:

(a) 1 and 2 only
(b) 2 only
(c) 1 and 3 only
(d) 1, 2 and 3

Ans: b

Exp: District Mineral Foundation (DMF) is a trust set up as a non-profit body, in those districts affected by the mining works, to work for the interest and benefit of persons and areas affected by mining related operations. It is funded through the contributions from miners.

Q. 63. ‘SWAYAM’, an imitative of the Government of India, aims at

(a) Promoting the Self Help Groups in rural areas
(b) Providing financial and technical assistance to young start-up entrepreneurs
(c) Promoting the education and health of adolescent girls
(d) Providing affordable and quality education to the citizens for free

Ans: d

Exp: SWAYAM is an online platform of the Ministry of HRD through which online programmes/courses will be offered to students in India.

Q. 64. The Montague-Chelmsford Proposals were related to

(a) social reforms
(b) educational reforms
(c) reforms in police administration
(d) constitutional reforms

Ans: d

Q. 65. What is/are common to the two historical places known as Ajanta and Mahabalipuram?
1. Both were built in the same period.
2. Both belong to the same religious denomination.
3. Both have rock-cut monuments.

Select the correct answer using the code given below.

(a) 1 and 2 only
(b) 3 only
(c) 1 and 3 only
(d) None of the statements given above is correct

Ans: b

Exp: Both are rock cut monuments.

Q. 66. With reference to ‘Bitcoins’ sometimes seen in the news, which of the following statements is/are correct?

1. Bitcoins are tracked by the Central Banks of the countries.
2. Anyone with a Bitcoin address can send and receive Bitcoins from anyone else with a Bitcoin address.
3. Online payments can be sent without either side knowing the identity of the other.

Select the correct answer using the code given below.

(a) 1 and 2 only
(b) 2 and 3 only
(c) 3 only
(d) 1, 2 and 3

Ans: b

Exp: Bitcoin are not maintained by the Central Bank. Bitcoins are created as a reward for payment processing work in which users offer their computing power to verify and record payments into a public ledger.

Q. 67. Consider the following statements:

1. New Development Bank has been set up by APEC.
2. The headquarters of New Development Bank is in Shanghai.

Which of the statements given above is/are correct?

(a) 1 only
(b) 2 only
(c) Both 1 and 2
(d) Neither 1 nor 2

Ans: b

Exp: New Development Bank has been established by the BRICS.

Q. 68. ‘Gadgil Committee Report’ and ‘Kasturirangan Committee Report’, sometimes seen in the news, are related to

(a) Constitutional reforms
(b) Ganga Action Plan
(c) Linking of rivers
(d) Protection of Western Ghats

Ans: d

Q. 69. Consider the following:
1. Calcutta Unitarian Committee
2. Tabernacle of New Dispensation
3. Indian Reforms Association

Keshab Chandra Sen is associated with the establishment of which of the above?

(a) 1 and 3 only
(b) 2 and 3 only
(c) 3 only
(d) 1, 2 and 3

Ans: b

Exp: Raja Ram Mohan Roy is associated with Calcutta Unitarian Committee

Q. 70. Recently, which of the following States has explored the possibility of constructing an artificial inland port to be connected to sea by a long navigational channel?

(a) Andhra Pradesh
(b) Chhattisgarh
(c) Karnataka
(d) Rajasthan

Ans: d

Exp: The Rajasthan government is exploring ways to develop an artificial inland port in Jalore by bringing in Arabian Sea water into Rajasthan through Gujarat. If implemented, Rajasthan will become accessible by water transport and will be able to shed the tag of a landlocked state.

Q. 71.    With reference to the Agreement at the UNFCCC Meeting in Paris in 2015, which of the following statements is/are correct?

1.    The Agreement was signed by all the member countries of the UN and it will go into effect in 2017.
2.    The Agreement aims to limit the greenhouse gas emissions so that the rise in average global temperature by the end of this century does not exceed 20C or even 1.50C above pre-industrial levels.
3.    Developed countries acknowledged their historical responsibility in global warming and committed to donate S 1000 billion a year from 2020 to help developing countries to cope with climate change.

Select the correct answer using the code given below.

(a)    1 and 3 only
(b)    2 only
(c)    2 and 3 only
(d)    1, 2 and 3

Ans: (b)

Exp: Parties to the U.N. Framework Convention on Climate Change (UNFCCC) reached a landmark agreement on December 12 in Paris, charting a fundamentally new course in the two-decade-old global climate effort. An historic agreement to combat climate change and unleash actions and investment towards a low carbon, resilient and sustainable future was agreed by 195 nations in Paris. The universal agreement’s main aim is to keep a global temperature rise this century well below 2 degrees Celsius and to drive efforts to limit the temperature increase even further to 1.5 degrees Celsius above pre-industrial levels. UNFCCC Conference of the Parties, or COP 21 agreed to extend the current goal of mobilizing $100 billion a year in support by 2020 through 2025, with a new, higher goal to be set for the period after 2025. Option (1) is separately correct but wrong with option (3)

Q. 72.    Consider the following statements:

1.    The Sustainable Development Goals were first proposed in 1972 by a global think tank called the ‘Club of Rome’.
2.    The Sustainable Development Goals have to be achieved by 2030.

Which of the statements given above is/are correct?

(a)    1 only
(b)    2 only
(c)    Both 1 and 2
(d)    Neither 1 nor 2

Ans: (b)

Exp: The Sustainable Development Goals (SDGs), officially known as Transforming our world: the 2030 Agenda for Sustainable Development, are an intergovernmental set of aspiration Goals with 169 targets. The Goals are contained in paragraph 54 United Nations Resolution A/RES/70/1 of 25 September 2015. The history of the SDGs can be traced to 1972 when governments met under the auspices of the United Nations Human and Environment Conference, to consider the rights of the human family to a healthy and productive environment. and not the Club of Rome which is a global think tank that deals with a variety of international political issues. Founded in 1968 at Accademia dei Lincei in Rome, Italy, It raised considerable public attention in 1972 with its report The Limits to Growth. The club states that its mission is “to act as a global catalyst for change through the identification and analysis of the crucial problems facing humanity.

Q. 73.    A recent movie titled The Man Who Knew Infinity is based on the biography of

(a)    S. Ramanujan
(b)    S. Chandrasekhar
(c)    S. N. Bose
(d)    C. V. Raman

Ans :(a )

Exp: The Man Who Knew Infinity is a 2015 British biographical drama film based on the 1991 book of the same name by Robert Kanigel. The film stars Dev Patel as the real-life Srinivasa Ramanujan, a mathematician who after growing up poor in Madras, India, earns admittance to Cambridge University during World War I, where he becomes a pioneer in mathematical theories with the guidance of his professor, G. H. Hardy (played by Jeremy Irons).

Q. 74.    Consider the following statements:

1.    The minimum age prescribed for any person to be a member of Panchayat is 25 years.
2.    A Panchayat reconstituted after premature dissolution continues only for the remainder period.
Which of the statements given above is/are correct?

a.    1 only
b.    2 only
c.    Both 1 and 2
d.    Neither 1 nor 2

Ans: (b)

Exp:  If you are a candidate for the seat of Member or Sarpanch of Gram Panchayat, you must be a registered voter in the electoral roll of that Gram Panchayat. (b) You must not be less than 21 years of age. Every Panchayat, unless sooner dissolved under any law for the time being in force, shall continue for five years from the date appointed for its first meeting and so longer. A Panchayat constituted upon the dissolution of a Panchayat before the expiration of its duration shall continue only for the remainder of the period for which the dissolved Panchayat would have continued under clause (1) had it not been so dissolved.

Q. 75.    India is an important member of the ‘International Thermonuclear Experimental Reactor’. If this experiment succeeds, what is the immediate advantage for India?

(a)    It can use thorium in place of uranium for power generation
(b)    It can attain a global role in satellite navigation
(c)    It can drastically improve the efficiency of its fission reactors in power generation
(d)    It can build fusion reactors for power generation

Ans: (d)

Exp: A mega international nuclear fusion research & engineering project, ITER is currently building the world’s largest experimental tokamak nuclear fusion reactor. A tokamak is a device that uses a magnetic field to confine plasma (fourth state of matter) in the shape of a torus. The ITER project aims to make the long-awaited transition from experimental studies of plasma physics to full-scale electricity-producing fusion power plants. It is seen as a method for electricity production from fusion energy — one for the future. The most vital aim is to produce at least 10 times more thermal energy than that required to operate it. This energy could be converted into electricity in future power-producing reactors. Scientists.

Q. 76.    In the context of the history of India, consider the following pairs:

Term                Description

1.    Eripatti    :    Land, revenue from which was set apart for the maintenance of the village tank
2.    Taniyurs    :    Villages donated to a single Brahmin or a group of Brahmins
3.    Ghatikas    :    Colleges generally attached to the temples

Which of the pairs given above is/are correctly matched?

(a)     1 and 2
(b)    3 only
(c)    2 and 3
(d)    1 and 3

Ans: (a)

Exp : Taniyurs- During the Chola regime Taniyurs developed out of major Brahmadeyas and temple settlements and came to include several hamlets and revenue villages.
Eripatti -A special category of land, eripatti or tank land, was known only in south India. the revenue from which was set apart for the maintenance of the village tank.
Ghatika: The Pallava kings of the ancient Tamil country who ruled from their capital city Kanchipuram   from the 4th-9th centuries A.D were known as patrons of scholars and set up Ghatikas. The Ghatikas were the most important educational institutions in South India during ancient times and the most important Ghatika of the Pallavas was located in Kanchipuram. Inscriptions reveal that many of these kings looked after this institution with great care. It was well-known as an important educational centre even in the 4th century A.D and an epigraph of a Kadamba king (of ancient Karnataka) records that his ancestors, a scholar named Mayura Sharman, along with his teacher Vira Sharman came to this institution from faraway Goa. Ghatikas were separate from temples.

Q. 77.    Consider the following statements:

1.    The International Solar Alliance was launched at the United Nations Climate Change Conference in 2015.
2.    The Alliance includes all the member countries of the United Nations.

Which of the statements given above is/are correct?

(a)    1 only
(b)    2 only
(c)    Both 1 and 2
(d)    Neither 1 nor 2

Ans: (a)

Exp: India and France launched an International Solar Alliance to boost solar energy in developing countries. The initiative was launched at the UN Climate Change Conference in Paris on 30 November by Indian Prime Minister Narendera Modi and French President Francois Hollande. The alliance includes around 120 countries that support the “Declaration on the occasion to launch the international solar alliance of countries dedicated to the promotion of solar energy.”

Q. 78.    ‘European Stability Mechanism’, sometimes seen in the news, is an

(a)    agency  created by EU to deal with the impact of millions of refugees arriving from Middle East
(b)    agency of EU that provides financial assistance to eurozone countries
(c)    agency of EU to deal with all the bilateral and multilateral agreements on trade
(d)    agency of EU to deal with the conflict arising among the member countries

Ans: (b)

Exp: The European Stability Mechanism is a European Union agency that provides financial assistance, in the form of loans, to eurozone countries or as new capital to banks in difficulty. It is a permanent agency, based in Luxembourg, and has replaced the temporaryEuropean Financial Stability Facility.

Q. 79.    Which of the following is/are the advantage/advantages of practicing drip irrigation?

1.    Reduction in weed
2.    Reduction in soil salinity
3.    Reduction in soil erosion

Select the correct answer using the code given below.

(a)    1 and 2 only
(b)    3 only
(c)    1 and 3 only
(d)    None of the above is an advantage of practicing drip irrigation

Ans: (c )

Exp: It has no role in reduction in soil salinity.

Q. 80.    Regarding ‘DigiLocker’, sometimes seen in the news, which of the following statements is/are correct?

1.    It is a digital locker system offered by the Government under Digital India Programme.
2.    It allows you to access your e-documents irrespective of your physical location.

Select the correct answer using the code given below

a)    1 only
b)    2 Only
c)    Both 1 and 2
d)    Neither 1 nor 2

Ans: (c )

Exp: DigiLocker is a “digital locker” service launched by the Government of India in February 2015 to provide a secure dedicated personal electronic space for storing the documents of resident Indian citizens. DigiLocker is one of the key initiatives under the Digital India Programme. This was released by the Department of Electronics and Information Technology (DeitY), Government of India. The storage space (maximum 10 MB at the time of launching & now upgraded to 1GB) is linked to the Unique Identification Authority of India (Aadhaar number) of the user. The space can be utilized for storing personal documents like University certificates, Permanent account number (PAN) cards, voter id cards, etc., and the URIs of the e-documents issued by various issuer departments. The idea is that this should minimise the need for physical documents; if your birth and education certificates are online, and you apply for a passport, then the Passport Office could use your Aadhaar number to request the DIGILocker for your details, without needing you to carry a large file of documents for the application.

Q. 81.    Recently, linking of which of the following rivers was undertaking?

a)    Cauvery and Tungabhadra
b)    Godavari and Krishna
c)    Mahanadi and Son
d)    Narmada and Tapti

Ans: b

Exp: Andhra Pradesh has taken the historic step of interlinking its two major rivers, Godavari and Krishna

Q. 82.    In the cities of our country, which among the following atmospheric gases are normally considered in calculating the value of Air Quality Index?

1.    Carbon dioxide
2.    Carbon monoxide
3.    Nitrogen dioxide
4.    Sulphur Dioxide
5.    Methane

Select the correct answer using the code given below

a)    1, 2 and 3 only
b)    2, 3 and 4 Only
c)    1, 4 and 5 Only
d)    1, 2, 3, 4 and 5

Ans: b

Exp: The Air quality index in India was recently developed by the IIT Kanpur. The Index considers eight pollutants — PM10 (particulate matter 10), PM2.5(particulate matter 2.5), NO2 (Nitrogen dioxide), SO2 (Sulphur dioxide), CO (Carbon monoxide), O3 (Ozone), NH3 (Ammoniac) and Pb(Lead)).

Q. 83.    With reference to ‘Astrosat’, the astronomical observatory launched by India, which of the following statements is/are correct?

1.    Other than USA and Russia, India is the only country to have launched a similar observatory into space.
2.    Astrosat is a 2000 kg satellite placed in an orbit at 1650 km above the surface of the Earth.

Select the correct answer using the code given below

a)    1 only
b)    2 Only
c)    Both 1 and 2
d)    Neither 1 nor 2

Ans: d

Exp: ASTROSAT is India’s first dedicated. This scientific satellite mission endeavours for a more detailed understanding of our universe.  ASTROSAT is designed to observe the universe in the Visible, Ultraviolet, low and high energy X-ray regions of the electromagnetic spectrum simultaneously with the help of its five payloads. The ASTROSAT spacecraft has a launch mass of 1650multi wavelength space observatorykg, including 868 kg of payload mass. Other space agencies have wavelength space observatory.

Q. 84.    With reference to the economic history of medieval India the term ‘Araghatta’ refers to

a)    bonded labour
b)    Land grants made to military officers
c)    Waterwheel used in the irrigation of land
d)    Wasteland converted to cultivated land

Ans: c

Exp: The ‘ara-ghatta’ or rope-pot system of lifting water from open wells was probably invented in erstwhile India of the past.
Q. 85.    With reference to the cultural history of India, the memorizing of chronicles, dynastic histories and epic tales was the profession of who of the following?

a)    Shramana
b)    Parivraajaka
c)    Agrahaarika
d)    Magadha

Ans: d

Q. 86.    Which of the following is not a member of ‘Gulf Cooperation Council’?

a)    Iran
b)    Saudi Arabia
c)    Oman
d)    Kuwait

Ans: a

Exp:  GCC is a political and economic alliance of six Middle Eastern countries—Saudi Arabia, Kuwait, the United Arab Emirates, Qatar, Bahrain, and Oman.

Q. 87.    What is/are the purpose/purposes of Government’s ‘Sovereign Gold Bond Scheme’ and Gold Monetization Scheme?

1.    To bring the idle gold lying with India households into the economy
2.    To promote FDI in the gold and jewelry sector
3.    To reduce India’s dependence on gold imports

Select the correct answer using the code given below

a)    1 Only
b)    2 and 3 Only
c)    1 and 3 only
d)    1, 2 and 3

Ans: c

Exp: Gold Bond and Gold Monetisation schemes combined purposes is to reduce India’s gold imports and bring all the gold lying idle with individuals and households in India into the economy.However, the schemes aren’t targeted at Foreign investors or boosting foreign investment.

Q. 88.    Recently, for the first time in our country, which of the following State has declared a particular butterfly as ‘State Butterfly’?

(a)    Arunachal Pradesh
(b)    Himachal Pradesh
(c)    Karnataka
(d)    Maharashtra

Ans: d

Exp: Maharashtra became the first State in the country to have a ‘Official State butterfly’ by declaring the Blue Mormon, the second largest butterfly found in India, as State butterfly.
The Blue Mormon is a large, swallowtail butterfly found primarily in Sri Lanka and India, mainly restricted to the Western Ghats of Maharashtra, South India and coastal belts.

Q. 89.    Consider the following statements:

The Mangalyaan launched by ISRO

1.    is also called the Mars Orbiter Mission
2.    Made India the second country to have a spacecraft orbit the Mars after USA
3.    Made India the only country to be successful in making its spacecraft orbits the Mars in its very first attempt.

Which of the statements given above is/are correct?

(a)    1 only
(b)    2 and 3 only
(c)    1 and 3 only
(d)    1, 2 and 3

Ans: c

Exp: ISRO has become the fourth space agency to reach Mars, after the Soviet space program, NASA, and the European Space Agency.

Q. 90.    What was the main reason for the split in the Indian National Congress at Surat in 1907?

(a) Introduction of communalism into Indian politics by Lord Minto.
(b) Extremists’ lack of faith in the capacity of the moderates to negotiate with the British Government
(c) Foundation of Muslim League
(d) Aurobindo Ghosh’s inability to be elected as the President of the Indian national Congress

Ans: b

Exp: Moderates believed in the policy of settlement of minor issues with the government by deliberations. But the extremists believed in agitation, strikes and boycotts to force their demands. Some nationalists led by LokmanyaTilak agitated against the moderate behaviour of congress against British rule.

 

Q. 91.    The plan of Sir Stafford Cripps envisaged that after the Second World War

(a)    India should be granted complete independence
(b)    India should be partitioned into two before granting independence
(c)    India should be made a republic with the condition that she will join the Commonwealth
(d)    India should be given Dominion status

Ans: d

Exp: Cripps Mission proposed to give dominion status after the war as well as elections to be held after the war.

92.    Consider the following pairs:

Famous place    Region
1.    Bodhgaya    : Baghelkhand
2.    Khajuraho    : Bundelkhand
3.    Shirdi    : Vidarbha
4.    Nasik (Nashik)    : Malwa
5.    Tirupati    : Rayalaseema

Which of the pairs given above are correctly matched?

(a)    1, 2 and 4
(b)    2, 3, 4 and 5
(c)    2 and 5 only
(d)    1, 3, 4 and 5

Ans: c

Exp: Malwa region includes districts of western Madhya Pradesh and parts of south-eastern Rajasthan.
Baghelkhand covers the northeastern regions ofMadhya Pradesh and a small area of western Uttar Pradesh.
Vidarbha is the eastern region of the Indian state of Maharashtra, comprising Nagpur Division and Amravati Division.

Q. 93.    The Parliament of India acquires the power to legislate on any item in the State List in the national interest if a resolution to that effect is passes by the

(a)    LokSabha by a simple majority of its total membership
(b)    LokSabha by a majority of not less than two-thirds of its total membership
(c)    RajyaSabha by a simple majority of its total membership
(d)    RajyaSabha by a majority of not less than two-thirds of its members present and voting

Ans: d

Exp: If the Council of States has declared by resolution supported by not less than two thirds of the members present and voting that it is necessary or expedient in national interest that Parliament should make laws with respect to any matter enumerated in the State List specified in the resolution, it shall be lawful for Parliament to make laws for the whole or any part of the territory of India with respect to that matter while the resolution remains in force.

Q. 94.    ‘Belt and road Initiative’ is sometimes mentioned in the news in the context of the affairs of

(a)    African Union
(b)    Brazil
(c)    European union
(d)    China

Ans: d

Exp: One Belt, One Road or the Belt and Road Initiativeis same. It is a development strategy and framework, proposed by Chinese paramount leader Xi Jinping that focuses on connectivity and cooperation among countries primarily between the People’s Republic of China and the rest of Eurasia, which consists of two main components, the land-based “Silk Road Economic Belt” (SREB) and oceangoing “Maritime Silk Road” (MSR).

Q. 95.    PradhanMantri MUDRA Yojana is aimed at

(a)    Bringing the small entrepreneurs into formal financial system
(b)    Providing loans to poor farmers for cultivating particular crops
(c)    Providing pensions to old and destitute persons
(d)    Funding the voluntary organization involved in the promotion of skill development and employment generation

Ans: a

Exp: The main objectives of Mudra Bank are, to encourage entrepreneurs and small business units to expand their capabilities and operations, to reduce over indebtedness and to provide formal system of credit (finance).

Q. 96.    In which of the following regions of India are shale gas resources found?

1.    Cambay Basin
2.    Cauvery basin
3.    Krishna-Godavari Basin

Select the correct answer using the code given below:

(a)    1 and 2 only
(b)    3 only
(c)    2 and 3 only
(d)    1, 2 and 3

Ans: d

Exp: Research is under process for presence of shale gas in Cambay basin at Mehsana, Ahmedabad and Bharuch districts of Gujarat, Cauvery basin at Nagapattinam in Tamil Nadu andin KG Basin at East and West Godavari districts of Andhra Pradesh.

Q. 97.    ‘Global Financial Stability Report’ is prepared by the

(a)    European Central Bank
(b)    International Monetary Fund
(c)    International Bank for Reconstruction and Development
(d)    Organization for Economic Cooperation and Development

Ans: b

Exp: The Global Financial Stability Report provides an assessment of the global financial system and markets, and addresses emerging market financing in a global context. It is published by IMF.

Q. 98.    Regarding ‘Atal Pension Yojana’, which of the following statements is/are correct?

1.    It is a  minimum guaranteed pension scheme mainly targeted at unorganized sector workers
2.    Only one member of a family can join the scheme
3.    Same amount of pension is guaranteed for the spouse for life after subscriber’s death.

Select the correct answer using the code given below:

(a)    1 only
(b)    2 and 3 only
(c)    1 and 3 only
(d)    1, 2 and 3

Ans: c

Exp: Atal Pension Yojana (APY) is open to all bank account holders who are not members of any statutory social security scheme. The minimum age of joining APY is 18 years and maximum age is 40 years. Therefore, minimum period of contribution by the subscriber under APY would be 20 years or more. No specification of family members. In case of death of subscriber pension would be available to the spouse and on the death of both of them (subscriber and spouse), the pension corpus would be returned to his nominee.
Q. 99.    The term ‘Regional Comprehensive Economic Partnership’ often appears in the news in the context of the affairs of a group of countries known as

(a)    G20
(b)    ASEAN
(c)    SCO
(d)    SAARC

Ans: b

Exp: Regional Comprehensive Economic Partnership (RCEP) is a proposed free trade agreement (FTA) between the ten member states of the Association of Southeast Asian Nations (ASEAN) (Brunei, Burma (Myanmar), Cambodia, Indonesia, Laos, Malaysia, thePhilippines, Singapore, Thailand, Vietnam) and the six states with which ASEAN has existing FTAs (Australia, China, India, Japan,South Korea and New Zealand).

Q. 100.    On which of the following can you find the Bureau of Energy Efficiency Star Label?

1.    Ceiling fans
2.    Electric geysers
3.    Tubular fluorescent lamps

Select the correct answer using the code given below:

(a)    1 and 2 only
(b)    3 only
(c)    2 and 3 only
(d)    1, 2 and 3

Ans: d

Exp: It is invoked for equipment/appliances Room Air Conditioner, Ceiling Fan, Colour Television, Computer, Direct Cool Refrigerator, Distribution Transformer, Domestic Gas Stove, Frost Free Refrigerator, General Purpose Industrial Motor, Monoset Pump, OpenwellSubmersible Pump Set, Stationary Type Water Heater, Submersible Pump Set, Tfl, Ballast, Solid State Inverter, Office Automation Products, Diesel Engine Driven Mono-setpumps For Agricultural Purposes, Diesel Generator Set, Led Lamps, Inverter Ac.

 

Keep visiting the page for rest of the questions… All the answer key will be updated till evening…

Prelims Exam Trend (2014 & 15)

The objective type format of Prelims Paper requires an aspirant to choose the correct answer from the codes given and generally the statements and codes are confusing in nature rather than providing an easy access to correct answer. This has been done to check the grasp of the topic by the aspirant.

Hence, aspirant requires a conceptual clarity of each subject along with grasp of multi-dimensions or says micro-topics of each subject so that intensive coverage of subject can be done.

Here, we are analyzing the trend of Paper 1 (2014 and 2015) i.e. General Studies to provide a broad overview about the subject wise division in the paper to the aspirants.

The broad subject-wise question distribution is as follows:

S.No. Subject Number of questions asked in 2015 Number of questions asked in 2014 
1. History and Culture 20 17
2. Geography 14 15
3. Polity 14 13
4. Economy 10 21
5. Environment 18 10
6. Science and technology 16 09
7. Current affairs 08 15

             

 

Manoj K. Jha

PT 2016: The Right Time is Now !

  • by Manoj K. Jha

This year’s preliminary examination will be a different game altogether. A tough examination has became tougher as one has to attempt only the GS.  In this light there are two pertinent questions which demand immediate answer:

 1.    What should be the right strategy to minimize the risk and maximize the possibility of qualifying the first stage?

 2.    How to manage time as one can’t leave PT preparation for the last minute and also can’t  put all efforts for PT at the cost of GS Mains, optional and gradual improvement of writing ability?

In light of the above questions one needs to understand that demand for PT is much different from GS Mains.

PT is more about range of information. But the point of issue is that there is no specific syllabus and demarcation line. This requires widespread knowledge both conceptually and factually. The whole process of content procurement for this stage has to be gradual even if one candidate is preparing from last 1,2 or 3 years. Along with it there is a misconception that in G.S aspirant has to just mug up the facts but unfortunately this mugging up can hardly help in clearing the exam. We have analyzed the new pattern and realized that preparation should be in much deep and micro level. Developing micro topic listing helps a lot in demarcating the possible boundary. We are also trying to do the same in our PT test series.

You can download micro level topics listing of GS for PT at: http://iasscore.in/iasscore-online-test.php

GS for PT is tough; most of the students are worried about it, as there is no fixed length and breath. However, at the same time they also show trait of careless attitude by carrying loose mode of preparation. Aspirants generally go through N.C.E.R.T. books and some crash materials. N.C.E.R.T. books are certainly important, but only to begin with to have conceptual clarity. You cannot depend solely on these books. In the changing scenario one needs to walk extra miles. Figure out the areas and keep attempting question to know the level of preparation.

Another tough question for this year’s Prelim is – what is likely to be the cut-off? This theme always has been a subject of high speculation and gossip. This year too aspirants are worried for the same. A frequent question, I face is – what would be safe marks in the PT 2016? Well, if we look at the previous years’ scoring pattern, one can easily notice that larger count of student scored approximately 110 in GS (2013, & 2014). However CSAT was a deciding factor then. But under the new pattern GS has become the only factor. Cut off for 2015 is yet to come. Even if we know the figure, it is going to be only a psychological rubbing to our imagination for the future cutoff.

So my suggestion is simple just   kick start your preparation of prelims along with mains and gradually bring PT on the main track three months before the ‘D’ day.

Manoj K. Jha

Step-up the game with super 30 topics

  • by Manoj K. Jha

GS for PT is tough; most of the students are worried about it, as there is no fixed length and breath. However, at the same time they also show traits of careless attitude by carrying loose mode of preparation. Aspirants generally go through N.C.E.R.T. books and some crash materials. N.C.E.R.T. books are certainly important, but only to begin with. You cannot depend solely on these books. Even we use to categorize the PT questions for the sake of our convenience with a purpose of decoding the trend. But loose and general categorization does not help always.

As the stakes have gone high, every single mark matters in Civil Services Exam. Earlier ever if, one scored even 45-50% in written examination, he/she could be confident to get a place in final selection. However, this year’s preliminary examination will be a different game altogether. A tough examination became tougher, as one has to attempt only the GS. CSAT is now qualifying in nature.

A tough question for this year’s Prelim is – what is likely to be the cut-off? This topic always has been a subject of high speculation and gossip. This year too aspirants are worried for the same. A frequent question, I face is – what would be safe marks in the PT ? Well, if we look at the previous years’ scoring pattern, one can easily notice that larger count of student scored approximately 110 in GS. However CSAT was a deciding factor then. But under the new pattern GS has become the only factor.

GS for PT is tough; most of the students are worried about it, as there is no fixed length and breath. However, at the same time they also show traits of careless attitude by carrying loose mode of preparation. Aspirants generally go through N.C.E.R.T. books and some crash materials. N.C.E.R.T. books are certainly important, but only to begin with. You cannot depend solely on these books. Even we use to categorize the PT questions for the sake of our convenience with a purpose of decoding the trend. But loose and general categorization does not help always.

For example: Analyze the question of last year’s Prelims and try to encircle total number of questions asked from Geography. In the first glimpse you may find that approximately 20 question were asked. But a careful scrutiny will give you another picture. 4 questions are from physical geography and related areas and rest questions are of very general nature and this does not demand to go through very popular bulky books. On the basis of such observation you may find that there are 30-35 areas where UPSC focuses most of the time, however, weights of the subject related question may vary year to year. To make your preparation more focused for this year’s PT, I am trying to figure out important areas where UPSC focuses all the time. You can also verify it by going through last few years questions.

The lists of areas are as follows:

1. Biodiversity and conservation strategy
2. Environmental problems (special focus on climate change, pollution, etc)
3. Environment and global initiative issues
4. Biotechnology
5. Nutrition, Health and diseases
6. Agriculture
7. Water resource and management
8. Infrastructure
9. Energy
10. Forest
11. Welfare schemes and programmes
12. Inclusion issues and initiatives
13. Recent development in the field of Space, Defence and IT
14. Culture
15. Buddhism and Jainism
16. Issues related to governance
17. Statutory and regulatory bodies
18. Parliament – functions, responsibilities and issues
19. Preamble, Fundamental Rights, DPSP and Fundamental Duties
20. Amendments
21. Constitutional Bodies
22. Indian Constitution: significant provisions and basic structure
23. Local governance
24. Demography
25. Industry and Trade
26. Monetary and Fiscal Policy
27. National income concept
28. Banking
29. India Year Book

• Land and the people
• National Symbols
• Agriculture
• Culture and Tourism
• Basic education Data
• Defence
• Education
• Environment
• Food and Civil Supplies
• Rural and Urban development
• Scientific and technological developments
• A leap forward – New Initiatives of the Government

30. Economic Survey

• State of the Economy: An overview
• Prices, Agriculture and Food Management
• Climate Change and Sustainable Development
• Social Infrastructure, Employment, and Human Development
• ‘Wiping every tear from every eye’: the JAM Number Trinity Solution
• What to Make in India? Manufacturing or Services?
• A National Market for Agricultural Commodities- Some Issues and the Way Forward
• From Carbon Subsidy to Carbon Tax: India’s Green Actions
• The Fourteenth Finance Commission (FFC) – Implications for Fiscal Federalism in India?

Click Here to Download Demography Notes

PRELIMS 2014 PAPER ANALYSIS: GENERAL STUDIES

The Preliminary Examination consists of two papers of Objective type (multiple choice questions) and carry a maximum of 400 marks. This examination is meant to serve as a screening test only; the marks obtained in the Preliminary Examination by the candidates who are declared qualified for admission to the Main Examination will not be counted for determining their final order of merit. The number of candidates to be admitted to the Main Examination will be about twelve to thirteen times the total approximate number of vacancies to be filled in the year in the various Services and Posts. Only those candidates who are declared by the Commission to have qualified in the Preliminary Examination in the year will be eligible for admission to the Main Examination of that year provided they are otherwise eligible for admission to the Main Examination.

Here, we are analyzing the trend of Paper 1 i.e. General Studies to provide a broad overview about the topics covered in the paper to the aspirants.

The broad subject-wise question distribution is as follows:

S. No. Subject  Number of question asked
1 History and culture 20
2 Geography 14
3 Polity 14
4 Economy 10
5 Environment 18
6 Science and technology 16
7 Current affairs 8


Now, the topic-wise question distribution is as follows:

1

History and culture

• Modern history 6
• Culture 15
• Ancient history 1
• Medieval history 1

2

Geography

• Physical geography 3
• Indian physiology 1
• Agriculture 3
• Economic and social geography 3
• Map 5

3

Environment

• Basic concepts 4
• Environmental issues 3
• Environmental governance 4
• Environmental conservation 7

4

Science and technology

5

• General science 10
• Technology 7

Basic Economy

• Banking – 4
• Finance – 1
• Fiscal – 3
• International trade and data – 2

6

Basic Polity

• Based on Articles of Constitution – 2
• Organs of Government – 7
• Organizations – 2

7

READ ALSO: CURRENT AFFAIRS: THE REAL GAME CHANGER FOR PT 15′
BY MANOJ K. JHA

Manoj K. Jha

How to read India Year Book?

  • by Manoj K. Jha

India Year Book is published by Ministry of Information and Broadcasting. It provides a detailed knowledge about India and various sectors of the economy. But it is too bulky, detailed and complicated. which makes it difficult for an aspirant to read and grasp all the points.

India Year Book is published by Ministry of Information and Broadcasting. It provides a detailed knowledge about India and various sectors of the economy. But it is too bulky, detailed and complicated…which makes it difficult for an aspirant to read and grasp all the points.

Firstly we will discuss the questions that are linked to India Year Book in Prelims paper 2014, then will discuss the strategy “how to effectively utilize the India Year Book” to clear IAS Prelims.

1.    Consider the following rivers:

1. Barak
2. Lohit
3. Subansiri

Which of the above flows / flow through Arunachal Pradesh?

(a) 1 only
(b) 2 and 3 only
(c) 1 and 3 only
(d) 1, 2 and 3

(Chapter Land the People: The Brahmaputra rises in Tibet, where it is known as Tsangpo and runs a long distance till it crosses over into India in Arunachal Pradesh under the name of Dihang. Near Passighat, the Debang and Lohit join the river Brahmaputra and the combined river runs all along the Assam in a narrow valley. It crosses into Bangladesh downstream of Dhubri. The principal tributaries of Brahmaputra in India are the Subansiri, Jia Bhareli, Dhansiri, Puthimari, Pagladiya and the Manas….)

2.    Chaitra 1 of the national calendar based on the Saka Era corresponds to which one of the following dates of the Gregorian calendar in a normal year of 365 days?

(a) 22 March (or 21st March)
(b) 15th May (or 16th May)
(c) 31st March (or  30th March)
(d) 21st April (or 20th April

(Chapter National Symbols: Dates of the national calendar have a permanent correspondence with dates of the Gregorian calendar, 1 Chaitra falling on 22 March normally and on 21 March in leap year.)

3.    The national motto of India, ‘Satyameva Jayate’ inscribed below the Emblem of India is taken from

(a) Katha Upanishad
(b) Chandogya Upanishad
(c) Aitareya Upanishad
(d) Mundaka Upanishad
(Chapter National Symbols: The words Satyameva Jayate from Mundaka Upanishad, meaning ‘Truth Alone Triumphs’, are inscribed below the abacus in Devanagari script.)

4.     In the Constitution of India, promotion of international peace and security is included in the

(a) Preamble to the Constitution
(b) Directive Principles of State Policy
(c) Fundamental Duties
(d) Ninth Schedule

(Chapter The Polity: Some of the other important DPSP relate to provision of opportunities and facilities for children to develop in a healthy manner…., promotion of international peace and security, just and honourable relations between nations, respect for international law, treaty obligations and settlement of international disputes by arbitration.)

5.    Which of the following is / are the function/functions of the Cabinet Secretariat?

1.  Preparation of agenda for Cabinet Meetings
2.  Secretarial assistance to Cabinet Committees
3.  Allocation of financial resources to the Ministries

Select the correct answer using the code given below.

(a) 1 only
(b) 2 and 3 only
(c) 1 and 2 only
(d) 1, 2 and 3

(Chapter The Polity: The Cabinet Secretariat is responsible for the administration of the Government of India (Transaction of Business) Rules, 1961 and the Government of India (Allocation of Business) Rules 1961, facilitating smooth transaction of business in Ministries/ Departments of the Government by ensuring adherence to these rules. The Secretariat assists in decision-making in Government by ensuring Inter-Ministerial coordination, ironing out differences amongst Ministries/Departments and evolving consensus through the instrumentality of the standing and ad hoc Committees of Secretaries.)

The above questions and explanations from India Year Book shows that for clearing Prelims reading few chapters are must.

Hereby, we are listing the important chapters and what to read in that…

Chapter 1: Land and the People: This chapter covers the physiography of India alongwith Demographic terminologies and Census 2011 data. Read this chapter thoroughly as it will help in grasping facts related to Indian physiography. Questions on demography and Census are common in UPSC so read the terminologies thoroughly.

Chapter 2: National symbols: This chapter covers the National symbols of India as National flag, National anthem, national fruit, flower, calendar, etc. It is a very important chapter as questions are directly asked on national symbols. Last year also there were two questions on National symbols.

Chapter 3: The Polity: This chapter covers the executive part of the government alongwith rights and basic features of the Constitution. This chapter is helpful for revision of Polity for Prelims. The topics and covered in short and to the point.

Chapter 4: Agriculture: This chapter covers the programmes and policies related to agriculture in India, different agricultural sectors and recent initiatives. Read only the introduction part and then move to new programmes and policies launched by GOI. Rest will be covered in Economics so no need to read the whole chapter.

Chapter 5: Art and Culture: This chapter covers the Intangible and Tangible Cultural heritage of India, along with Tourism. Read about Cultural Institutions mentioned in India Year Book but stick to those institutions which are covered in newspaper in past six months. Read the Medical tourism topic also.

Chapter 6: Basic Economic Data: This chapter covers the statistical institutions in India. Make notes about organizations and reports published by them. This question can come in the form of match the following.

Chapter 7: Commerce: It can be ignored as it is helpful for Mains.

Chapter 8: Communication: This chapter covers the three communication forms i.e. Posts, Telecommunications and Information Technology. Read the telecommunication part to know about the technological terms related to telecom industry, no need to read about the tariff policies and other detailed policies related to telecom sector. Also read the cyber security topic in detail.

Chapter 9:
Defence: This chapter covers the security scenario of India, defence undertakings, training for defence services, etc. This chapter is very important as UPSC asks direct questions related to it. Focus more on new missiles/testing of missiles.

Chapter 10: Education: This chapter covers the policies related to elementary, secondary and higher education. In this chapter focus on salient features of the schemes related to education. Don’t read detailed explanations of each scheme. Also read the topics related to copyright.

Chapter 11:
Energy: This chapter covers the conventional and non conventional energy sector in India. Read the energy sources present in India mainly related to geographic position alongwith initiatives in renewable energy sector. Rest of the chapter is important for Mains.

Chapter 12:
Environment: This chapter is very important as UPSC has started asking more detailed questions in environment. Make a list of institutions related to environment and their status whether statutory, quasi judicial, etc in nature. Read the following topics: biodiversity, hazardous waste, wetland, E-waste in detail.

Chapter 13:
Finance: In this Chapter the economic terms associated with Banking, Budget, Insurance, taxes. This will be helpful in tackling the economy questions in Prelims. Along with that the new steps initiated by Government in different sectors will be helpful in Mains also.

Chapter 14: Corporate Affairs: In this chapter read about Companies Act, definition of Limited Liability Partnership Act, SFIO, CCI, CAT, etc as the questions can be asked on the functions the different bodies.

Chapter 15: Food and Civil Supplies: It is a very important chapter as in covers food security issues and welfare schemes related to it. The questions on schemes are quiet common in UPSC. Whereas, due to the recent controversy of food safety, the Quality assurance topic is very important.

Chapter 16: Health and Family welfare: This chapter is also very important as it covers schemes related to Health. Further the government is emphasizing more on AYUSH so read that topic thoroughly.

Chapter 17: Housing: The programmes and policies related to Housing and the rating criteria of houses must be read.

Chapter 18: India and the World: It can be ignored as it is helpful for Mains. Current issues related to it should be covered from newspapers.

Chapter 19: Industry: Read the recent steps initiated by the GOI for simplifying the business environment and FDI policy. Further read about CPSEs and terminologies related to it.

Chapter 20: Law and Justice: The basics of judiciary have already been read in Polity part so read the new initiaitives and alternative form of justice mechanism as questions on Lok Adalat, Gram Nyalayas are very common. Along with this read the enforcement agencies mandate for maintaining law and order. Further in the election section voting rights to the citizens of India living abroad is must.

Chapter 21: Labour and Employment: This chapter is very important as government has initiated labour reforms. Read the mandates of each Acts related to labour laws social security schemes provided to them. Skill development initiatives proposed by GOI in budget should be thoroughly read.

Chapter 22: Mass communication: It can be ignored just read the chronology of the different institutions as questions can be asked on that.

Chapter 23: Planning: Read the important features of the XII Five Year Plan. Further read the Niti Aayog, composition, functions, etc from newspapers.

Chapter 24: Rural and Urban Development: This Chapter covers the Major programmes for rural and urban development as MGNREGA, NRLM, Land Reforms, Local government programmes, and planned urbanization elements.  Read the vision and mission of programmes and policies in detail.

Chapter 25:
Science and Technological Development: Read the Science, Technology and Innovation Policy of the government; Nuclear; Space programme of India. Concentrate more on the latest technologies related to it.

Chapter 26: Transport: Read this chapter in sync with geography.

Chapter 27: Water resources: This chapter covers the information about National Water Policy, Groundwater resource, flood management and institutions related to it. Read the Nanami Gange Programme, inter-state river issues (tribunal for issue between different states).

Chapter 28:
Welfare: This chapter covers the welfare schemes for the SC/ST/OBC/Minorities/Women/Children. Read the schemes related to each in brief with special focus on Minorities, Women and Children.

Chapter 29: Youth Affairs and Sports: Read the National Youth Policy along with new initiatives of the government for promotion of sports.

Chapter 30: State and Union Territories: The whole chapter can be ignored except North East states and Telengana.

Chapter 31: Diary of national events: It can be ignored.

Chapter 32: A Leap Forward – Initiatives of the Government: This chapter covers the new programmes and policies launched by the new government. This chapter is very important as it covers the current schemes as Swachch bharat Mission, Jan Dhan Yojana, Sansad Adarsh Gram Yojana, etc.

Click Here to Download India Year Book – Agriculture

 

Manoj K. Jha

How IAS examination questions are framed?

  • by Manoj K. Jha

Can u guess how IAS examination questions are framed? What are the sources they opt for framing the questions?

Can u guess how IAS examination questions are framed? What are the sources they opt for framing the questions? No surprise, we will provide u the exact source along with book and pages number. We will start with the details of Reading Comprehension of last three year IAS prelims exam. Have a look….

COMPREHENSION PASSAGES: TITLES & SOURCES

A. SOCIAL

1. The Concept of a “Creative Society” [CSAT-2011]
[Book: “Readings in Indian Government and Politics: Class, Caste, Gender (Volume of Essays)” edited by Manoranjan Mohanty; Publisher: SAGE; ‘Chapter-15: Conclusion—Social Movements in a Creative Society’ by Manoranjan Mohanty (The Creative Society: Page-390)]

B. POLITICAL

2. Deepening of Democracy in India: [CSAT-2012]
Transfer of Power from Upper Caste Elites to the Marginalised Subaltern Groups [CSAT-2012]
[Book: “Indian Politics and Society Since Independence: Events, Processes and Ideology” by Bidyut Chakrabarty; Publisher: Routledge; Conclusion: “Growing Democratization” (Pages-171-172)]

3. Democracy and Development—Do they Go Together? [CSAT-2012]
[Book: “The Idea of Justice” by Amartya Sen; Publisher: Penguin; Chapter: ‘Democracy and Development’]

4. Democracy Not Exclusive Western Concept, But Development of Institutional Structure [CSAT-2013] of Democracy—A Western Achievement
[Book: “The Idea of Justice” by Amartya Sen; Publisher: Penguin; Part Four: “Public Reasoning and Democracy”; Chapter-15: ‘Democracy as Public Reason’]

C. ECONOMIC

5. Indian Economy—A Great Paradox [CSAT-SAMPLE]
[Book: “Contemporary India: Economy, Society, Politics”; Publisher: PEARSON; Part-I, Chapter-2: The Evolution of Development Strategy Since Independence by Ambuja Kumar Tripathy]

6. An Enabling State for Inclusive Growth—The Need of the Hour [CSAT-2011]
[The Economic Survey of India 2009-10, Chapter-2: Micro-foundations of Inclusive Growth, Paragraph 2.5]

7. The Potential of Farm Collectivities in Transition Economies [CSAT-2012]
[Working Paper No. 305 (2010) by Bina Agarwal: “Rethinking Agricultural Production Collectivities”, Institute of Economic Growth, Delhi University; Chapter-7: In Conclusion (Page-31)]

8. Competition Law to Facilitate a Level-Playing Field Among Foreign and Domestic Firms [CSAT-2012]
[Article in The Economic Times dated 04 Nov 2009: “Competition Law and Inclusive Growth” by Madhav Mehra]

9. Indian Financial Markets Integrated With International Financial Markets [CSAT-2013]
[The Economic Survey of India 2011-12, Chapter-5: Financial Intermediation and Markets (Introduction)]

10. Corporate Governance [CSAT-2013]
[Article in The Economic Times dated 18 January 2009: “Corporate Governance” by Lisa Mary Thomson]

D. SCIENTIFIC/ECOLOGY/ENVIRONMENT

11. Effects of Modifications to Earth’s Ecosystem, [CSAT-2011]
Necessitated by a Rapidly Expanding Human Population
[Textbook: “Life-The Science of Biology” (8th Edition) co-published by Sinauer Associates Inc. and WH Freeman; Chapter-56: “Ecosystems and the Global Ecology”—56.4 ‘What Services do Ecosystems Provide’ (Page-1221)]

12. The Influence of Keystone Species on Species Richness in an Ecosystem [CSAT-2011]
[Textbook: “Life-The Science of Biology” (8th Edition) co-published by Sinauer Associates Inc. and WH Freeman; Chapter-55: “Communities and Ecosystems”— (Pages-1062-63) ‘Individual Species May Influence Community Processes’]

13. Introduction of Exotic Species in New Geographic Areas and Its Impact on Biodiversity [CSAT-2012]
[Textbook: “Essentials of Ecology” by Colin R Townsend, Michael Begon, John L Harper; Publisher: John Wiley & Sons; Article 14.2.2 Habitat Disruption (Page-462)]

14. Use of Chemical Pesticides in Agriculture—Is it Sustainable? [CSAT-2012]
[Website: www.trinitygreenconsultancy.com; Category: “Species Richness”; Article: ‘Evolution of Resistance to Pesticides” (posted 01 Nov 2012)]

15. Climate-Smart Development Strategies of Adaptation and Mitigation – The Need of the Hour [CSAT-2012]
[The World Bank Report: “World Development Report—Development and Climate Change” (2010); Chapter-1 (Pages-44-45)—‘Growth: Changing Carbon Footprints and Vulnerabilities’]

16. Habitat Fragmentation—The Most Serious Threat to Biodiversity Conservation [CSAT-2013]
[Article in Deccan Herald dated 17 August 2008: “Mining has spelt ecological disaster” by Praveen Bhargav]

17. Excessive & Indiscriminate Use of Chemical Fertilizers in Agriculture— [CSAT-2013]
A Serious Threat to Aquatic Ecosystems
[Textbook: “Essentials of Ecology” by Colin R Townsend, Michael Begon, John L Harper; Publisher: John Wiley & Sons; Chapter 13: Habitat Degradation; Article 13.2.2 Intensive Cropping (Pages-429 & 430)]

E. EDUCATION

18. Will the Right to Education Remain on Paper or Become a Reality, [CSAT-2011]
Particularly so for Girls?
[Article in The Hindu dated 02 April 2010: “India’s Children Have a Precarious Right” by Krishna Kumar.]

F. ETHICS & PHILOSOPHY

19. A Moral Act is an Act of Will [CSAT-2011]
[Book: “Ethical Religion” by Mohandas Karamchand Gandhi; Chapter-3: “What is Moral Action”]

20. India’s Spiritual Greatness—Myth or Reality? [CSAT-2011]
[Book: “Essays on Ancient India” by Raj Kumar; Publisher: Discovery Publishing House]

21. Ever Increasing Man’s Materialistic Needs Leads to Endless Miseries; [CSAT-2013]Only Solution to Change Man’s Character—Make Him Pure, Educated and Spiritually Strong
[Book: “The Indispensable Vivekananda: An Anthology for Our Times” by Swami Vivekananda;

Editor: Amiya P Sen; Publisher: Permanent Black; Chapter III. b. “Karma Yoga: Service Before Self” (Page-203)]

Manoj K. Jha

Current Affairs: the real game changer for PT 15

  • by Manoj K. Jha

To succeed in UPSC Exam, a thorough knowledge of current affairs is required. In the examination, sometimes the questions are directly related to Current Affairs or sometimes indirectly related to the current events or news. Sometimes even the traditional part of the General Studies questions are those questions, which were in news during even, last ten to twelve months. If we analyze Prelims questions, at least 20 to 25% questions are directly related to Current Affairs. The percentage is likely to increase further in future.

Prelims Preparatory series Part -2

UPSC is often notoriously termed as Unpredictable Public Service Commission, because of its ability to retain surprises regarding the questions year-after-year. Since I have been vilified in past for predicting the upcoming changes and forewarning candidates regarding them and have already invited criticism, I now ask you a question:

What is there to unfold now? Nothing new is there now, except the change in nature of questions and focus areas.

There is a clue to this question – Will ‘Current Affairs’ be the real game changer for PT 2015?

I will also soon come up with a post related to the format of the questions, till then you may find this useful…

To succeed in UPSC Exam, a thorough knowledge of current affairs is required. In the examination, sometimes the questions are directly related to Current Affairs or sometimes indirectly related to the current events or news. Sometimes even the traditional part of the General Studies questions are those questions, which were in news during even, last ten to twelve months. If we analyze Prelims questions, at least 20 to 25% questions are directly related to Current Affairs. The percentage is likely to increase further in future.

Current Affairs for IAS is an area which is by far the most important aspect of the GS paper and also perhaps the most difficult area to master. Important, because good number of questions, whether they are in the prelims or the mains, draw their inspiration from what is happening in the contemporary context, and difficult, because of its vastness and difficulty in identifying what is important and what is not.

Most students have been found to be complaining that they have not been able to recall and reproduce, whatever they have studied in the current affairs, and in the sense that the factual part that is useful in the prelims cannot be recalled as far as nature of choices is concerned and the analytical part which is useful in the mains, cannot be related and incorporated in the descriptive answers.

However, the factual versus conceptual demand for GS is now an outdated story. Concepts are paramount. People have been suggesting several strategies to prepare for current affairs, but very few people have really decoded the challenge. Those who have managed to do so, are now busy calling the shots in bureaucracy. On the same line, this year’s prelims will begin a new chapter for candidates, as a historical first after the latest change.

So how to start now….

Firstly, the preparation of the current affairs is not a separate activity carried out in isolation with other topics; rather the concepts in all other areas together contribute both to the understanding and in augmenting the relevance of whatever we study in current affairs apart from its retention factor.

Secondly the study of current affairs has to be essentially based on those issues, which are of contemporary importance. Here preparation of prelims will be different from the mains one.

Third, but most crucial is the listing of topics and ability to comprehends facts that could be possible there in the given choices. This is by far the most important aspect of studying current affairs. This alone shall ensure that whatever is studied is not only methodically arranged in the mind, but is also logically reproduced in the exam.

By adopting this approach a student can even create his personal notes extensively, replete with references for current happenings on all the major issues of the time. However, this approach is best carried out only when a student is thorough with all the concepts across the various disciplines and is in a position to identify the areas where these concepts apply and contribute to enrich the overall understanding.

IAS Score

Is comprehension a reason to worry about???

  • by IAS Score

Of the late Reading Comprehension has become a big headache for the students appearing in the Civil Services Preliminary Examination.

Of the late Reading Comprehension has become a big headache for the students appearing in the Civil Services Preliminary Examination. Majority people who couldn’t make it for the mains either left huge marks unattempted only because they couldn’t manage the time while trying to solve the questions of Reading Comprehension or they couldn’t grasp the basic essence of the passages and marked the answers wrong. It is true that given the weightage UPSC gives to the Reading Comprehensions (nearly 40% to 45% of Paper II) it is absurd that one can leave them while attempting Prelim Paper II (CSAT), along with this the other sections of the paper can also not be ignored. We have already discussed about the various sources of Comprehensions given by UPSC in the previous three years of examinations but to solve them what you need is a resounding strategy. Here we tried to solve this problem for you:-

1) Broaden your database (knowledge)

What section of the newspaper do you turn to while reading a newspaper? Is it the sports section, the editorials, the celebrity column or some other section? I go with the sports section first. Why? Because I like to read news and reports about the sports events.

Similarly, the section that you like to read the most is the one you are most comfortable with.

Do you ever get bored reading that section or do you ever have difficulty understanding what’s written in that section? The answer, of course, is no (or else you would not read it).

Reading essays on topics which you understand and know about is very easy as compared to reading on alien topics. Hence, it is very important to develop a broad background of knowledge in various fields. Remember that essays from any field can come in Reading Comprehension questions.

You can improve your knowledge base by reading books, newspapers, magazines, articles and periodicals regularly. Start taking interest in what is happening around the world. This would not only help you develop your reading skills but also help you with your General Studies (especially what we call ‘current affairs’) and Personal Interview stage.

2) Learn to understand the structure of Paragraphs

Most often, the paragraphs used in Reading Comprehension passages are taken from articles or periodicals written by good writers. These writers generally have a well defined structure of constructing a paragraph. Their paragraphs will mostly have a proper beginning, middle part and an end section. Knowing the functions of the respective parts can help you understand what the passage is trying to convey. For example, the first sentence will most probably be constructed in such a way that it gives the reader the main idea of what is to come in the passage.

3) Choose your Passages Carefully

Accuracy holds the key in Reading Comprehension as well as Verbal Ability as a whole. So, try and finish the easy passages first instead of wasting time on tough passages.

How do I know if a passage is easy?

Keep two things in mind:

a) Is the topic of the passage something you are familiar with?

b) Are the question direct or twisted?

Solve passages having direct questions first. It will be better if the passage is on a subject which you know about.

4) Focus on Comprehension, instead of speed

While reading speed is a key factor in saving time, it is not the most important criteria. One can read a passage very fast without really understanding what it is about. Hence, you need to make sure that you pay more attention to reading the passage instead of Reading Speed.

You should be able to comprehend the passage fast. This will help you a lot in solving the questions. On the other hand, if you just read fast and do not comprehend properly, you may have to revert to the passage again and again.

5) Keep noting the main points of the passage

As long as possible, try to note down the main points of the passage while you are reading it.

Noting down the main points would help you in case you forget something. This will prevent the unpleasant experience of searching for ‘that point I read somewhere’.

Follow these and I am sure you will come out with comparative ease….

IAS Score

Preparing Data Interpretation and Data Sufficiency for CSAT

  • by IAS Score

Data Interpretation is one of the easy sections of one day competitive Examinations. It is an extension of Mathematical skill and accuracy

By Dipak Mishra
Data Interpretation is one of the easy sections of one day competitive Examinations. It is an extension of Mathematical skill and accuracy. In the CSAT scenario, it has become important because more often than not, the civil services officers have voluminous data in different forms, from which they are required to churn out information and help the concerned authorities in formulating policy decisions.
Data Interpretation tests your speed, decision making capability and analysing data. It consists of a good number of graphs, charts and tables from which you will have to analyse data. The key to cracking this area is to quickly identify the key pieces of data that you will require to work on the questions asked. It is not unknown for question-setters to try and bewilder students with a large amount of data, most of it unnecessary. As a rule, the more the data presented, the easier the questions that follow, so don’t lose heart if you see a table with 10 columns occupying one whole page. On the other hand, several seemingly innocuous questions may stump you.
Different Data Forms
The different data forms that usually confront the students are
Table Graphs
Tables are often used in reports, magazines and newspaper to present a set of numerical facts. They enable the reader to make comparisons and to draw quick conclusions. It is one of the easiest and
most accurate ways of presenting data. They require much closer reading than graphs of other forms and hence are difficult and time consuming to interpret.
One of the main purposes of tables is to make complicated information easier to understand. The advantage of presenting data in a table is that one can see the information at a glance.

Pie Charts
They derive their name from its shape, like that of a pie divided into various portions. They always represent data in the form of a percentage of the total, with the total percentage being 100. In such a chart, the length of the arc (and therefore the angle each sector subtends at the centre) is proportional to the quantity it represents. Such charts are often used in the corporate world and in newspapers. Since a circle comprises 360 degrees, each percent of a pie-chart is equal to 360 divided by 100, or 3.6 degrees. This fact will be important for the calculations you are expected to perform.

Bar Graphs
Bar graphs represent data in the form of columns or bars. Bar graphs can be horizontal or vertical. The length of the bar is proportional to the data value represented by it.

Line Graphs
Line graph represents data in the form of straight lines that connect various data values. Both line graphs and bar graphs are used to convey same things and hence can be used inter-changeably. For example, a line graph can be generated by joining the tip of the bar graph.

Caselets
In caselets, the mathematical data is represented in the form of a paragraph. Hence extracting data and establishing relationships between different data values becomes difficult. However caselets are very popular with the examiners.

Combined Data Sets
Data is represented in two or more different types of data sets. It could be combination of a table and a graph or two or more similar graphs. You may have to correlate the data in different data sets to solve these questions. Thus interpreting data takes time. These type of sets are very commonly asked. However based on experience, we feel that that if such a set comes in CSAT, then it would not be heavy on data and be an easy set to interpret with the focus on correlation of data.

Some tips to score well in Data Interpretation:
Careful Reading and Analyzing

The first and the most important step in solving any Data Interpretation question is to read the question carefully. Many a times, the data given below the graph turns out to be more important than most of the numbers in the graphs.

The next step should be to analyze the given graph/data carefully. Do not try to see the questions first and find out the answers accordingly. You will waste your time if you follow that method. Try to understand the graph. Instead do the reverse.What is the graph all about? Which years does it cover? Is the data in absolute terms or in percentage terms? What do the two axis signify?, etc. Look at the statistics for each graph, chart, table or pie diagram. Look carefully at the labels. Make sure you understand the central theme of the data.
Worry less about data given
Understand the question, which might contain lots of data that is unrelated and not required for answering the questions. When you look at the question you may get discouraged by the lengthy tables or by the amount of information below the graphs. But, if you were to understand what the data is about and then look at the question, you may find that you only have to use part of the data. Don’t get disheartened by the amount of data and the only thing important is whether you can correlate the data between the graphs properly.

Skip Calculative Questions
Some questions require lots of calculation in order to be solved. These questions are known as the speedbreakers. Such questions are best left alone, at least in the first round. Once you have solved all the easy questions then go to the difficult ones. If you try such questions in the first round, you will lose your precious time on them and may not be able to attempt some simple questions that may follow. Also there are many unnecessary calculations that we do, which might cost us a precious few seconds per question. Sometimes, there are many steps that can be skipped but we still do it as we are trained to solve in a step by step method. Learn to skip those steps.

Attention to the minor details
This is related to the first step which we discussed. Sometimes, the questions will use a different unit for the question and another unit for the data. For example, the data given may be about sales volume in Millions. However, the question may ask about sales volume in lakhs. Also you should have an idea about sales volume and sales value.If you do not pay close attention to the unit, you may chose the wrong answer.
Sometimes there are questions which will ask us to find out some data for which information may not be available. Always be alert enough to see whether the data given is enough to answer the question or not and do not go forward with answering the questions based on assumptions. We will look into this in our next part of discussion.

Approximate properly
CSAT is not about finding the exact answer always. Most of the times, the options given are far enough from each other to give you enough room for approximation. So, for example, if you are asked to divide 642678/161335, you can easily approximate that the answer will be somewhere around 4 by looking at the first 2 numbers only. 64/16 is 4. So, instead of dividing it and finding the accurate answer, try to find an appproximate answer. This will give you the correct answer more often than not. If however, the options are close, you will still be able to eliminate 1 or 2 options easily.

Lets attempt one set and try to apply those steps

The pie chart shows the distribution of Rs 6 lakhs spent by a construction contractor on different items.

1. The amount spent on cement is

a.    Rs.2,00,000
b.    Rs.1,60,000
c.    Rs.1,20,000
d.    Rs.1,00,000

Total is Rs. 6,00,000 (=360 degrees of the circle).Cement is 72 Degrees.
Apply the ratio principle. Hence 72/360= M/6 lakhs where M is the amount spent on cement. HenceM=(72*6)/360=1.2 lakhs.

2. The amount spent on labour exceeds the amount spent on steel by

A.    5% of the total cost
B.    66.66% of the total cost
C.    12% of the total cost
D.    15% of the total cost

There is no need to find absolute values. Just observe the degrees. The difference between Labour and Steel is 90-54 or 36 degrees. And total cost is (6 lakh=) 360 degrees.

So the percentage= 36/360 x 100 =10% of the total cost. But if the options would have omitted the word  “of the total cost”, then the answer would have been (90-54)/54 × 100= 66.66%.The base would have been the cost of steel, in that case.Notice the wrong option B to create confusion in the mind.

3. The amount spent on cement and steel is what percent of the cost on supervision?

A.    70%
B.    42.94%
C.    50%
D.    233.33%

Again no need to find absolute values. Just observe the degrees. The total degrees is 126 on cement and steel while it is 54 for supervision. It is asking, “what percent of the total cost on supervision?”

So cost of supervision is the “base”%.. Hence the required value=(126/54) x 100 =233.33%.

If we reverse the base(take the base as cement and steel) , it becomes 42.94%. If we take the total cost as the base the value becomes 126/180 x 100=70%.If we take the total cost of three items vis-à-vis the total cost as the base, then the value becomes 180/360 x 100=50%.These are the 4 options given in the question as observed here.

DATA  SUFFICIENCY

Data sufficiency tests your skill on the amount data you require to take a decision. Quite a number  of times, you may have to arrive at a decision on the amount of data available, which might be more /less than you actually require. If  you ask your block officer under you as to “ How many families have 2 or more girls in this area” and he/she starts reeling off figures of the population of the block according to last census, infant mortality rate, families having children, population growth etc..that will not serve your purpose. The focus should be only on girl child and not children. A family can have girls and boys, but the question asks” How many families have 2 or more girls in this area”. That family can have boys, but whether it has 2 or more girls  should be the aim that you should focus on.

Format of Data sufficiency question

The format of the question would be something like this.
1.    Is integer x positive?

1.    x > 9
2.    x2 > 81
Mark

(A) if one statement alone but not on other statement alone is sufficient to answer the question
(B) EACH statement ALONE is sufficient to answer the question asked
(C) Both statements (1) and (2) TOGETHER are sufficient to answer the question asked; but NEITHER statement ALONE is sufficient
(D) Statements (1) and (2) TOGETHER are NOT sufficient to answer the question asked, and additional data specific to the problem are needed.

Data Sufficiency questions hinge on whether a statement is sufficient to answer a question. A statement is sufficient when it guarantees exactly one answer to that question. For example, in the question:

Is integer x positive?

The statement “x > 9” would be sufficient, as any number greater than 9 is also greater than 0 and therefore positive.

The statement “x2 > 81”, however, would not be sufficient, as there are two potential values of x: 9 (which gives the answer “yes, x is positive”) and -9 (which gives the answer “no, x is not positive”). The correct option is A.

Your job, then, is to determine when a statement is sufficient to provide exactly one answer to the overarching question.

Some tips to score well in Data Sufficiency:

Don’t solve the question.

Data Sufficiency questions simply ask whether you COULD solve the question given the information in the statements. Don’t waste valuable time actually solving unless you are uncertain of sufficiency! Data sufficiency questions are not supposed to involve long and drawn-out calculations. If you find yourself calulating, there is probably something you’re not seeing. Remember that variables can equal a variety of values: negatives, positives, integers, fractions, zero. Don’t simplify when you don’t know what a variable can equal, and don’t assume variables are positive integers!

For “yes/no” questions, focus on whether you are getting a firm yes or no.

There is no “correct” answer. Our task is merely to determine when we have enough information. It is possible for one statement to answer the question “yes” and the other to answer the question “no” and have BOTH statements be independently sufficient.

Focus on this question

2.    Is x=4?

1.X3= -64
2.X2=9
Mark

(A) if one statement alone but not on other statement alone is sufficient to answer the question
(B) EACH statement ALONE is sufficient to answer the question asked
(C) Both statements (1) and (2) TOGETHER are sufficient to answer the question asked; but NEITHER statement ALONE is sufficient
(D) Statements (1) and (2) TOGETHER are NOT sufficient to answer the question asked, and additional data specific to the problem are needed.

Here from the 1st statement we get X=-4 which answers the question that X IS NOT 4.

The 2nd statement shows that X is either 3 or -3 and hence not 4.Hence X IS NOT 4.The correct option is B. Note that NO is also an answer to a question.

Note down the variables needed for answering the question BEFORE looking at the statements.

Try to write down your own prediction first. It helps you to eliminate the statements more quickly if you have something to compare their information to, and forces you to spend more time thinking critically. Don’t rush this step!

Remove biasness in mind.

Once you determine the type of question and have analyzed the information given, analyze the first two statements independently of each other. If you have used Statement 1 to answer the question, try to “forget” statement 1 when you move on to statement 2. Don’t underestimate how challenging this can be – try not to mentally “carry over” any info from one statement to the next.

Use the process of elimination to narrow down the choices methodically.

If Statement 1 is sufficient, eliminate C and D. The only two options are A and B. Conversely if 1 is NOT sufficient, eliminate B. Simply by appropriately analyzing the first statement, you can eliminate two answer choices! If statement 1 is challenging for you, you can start by analyzing statement 2. If statement 2 is insufficient, then choices B can immediately be eliminated. It doesn’t matter which statement you analyze first, as long as you start by looking at them individually. Also in some , the question itself gives some of the information which you require. It might be possible that same information is rephrased and again given in one of the statements. You should reach this conclusion that “THE STATEMENT IS REDUNDANT”, and do not use that statement to answer the question.

IAS Score

Preparing for General Mental ability and Basic Numeracy for CSAT

  • by IAS Score

Quantitative section takes about 10-12 % of the CSAT paper. The number of questions varied from 3 to 7 to 11 till last year.

BY DIPAK MISHRA

Quantitative section takes about 10-12 % of the CSAT paper. The number of questions varied from 3 to 7 to 11 till last year. The questions were relatively of lesser difficulty level, hence it is easy to crack this part of the paper. However a few core skills need to be developed. These include a combination of accuracy and speed, the ability to perform simple calculations under pressure and the ability to use reasoning while solving mathematics questions.

Considering the nature and scope of these skills, practice should become imperative for any student. The basic understanding and grasp of concepts is vital and should never be looked over. Along with these, one need to develop a certain set of advanced skills and learn a few short cuts so as to minimize the time you spend on each question.

Given below are some of the tips to master this section along with some examples below to reinforce those tips.

► Learn to master the basics: Majority of questions are asked from the Arithmetic section with the odd questions asked from the Modern Maths section ( viz. Permutation/Combination, Probability and Set Theory). Hence focus should be to master the basics of these sections. Develop strong fundamentals for each of the above mentioned categories. Remember weakness in one area can have a negative effect on multiple sections because most of the time the question requires understanding of concepts of two different areas. In addition to this, one should always know the amount of weightage one should give to each sub areas of quantitative ability while preparing. For example, recent trends show that numbers, time speed distance, permutations and combinations are some topics that are frequently tested in the examination. Questions from topics like Mensuration, time and work and probability vary from year to year. However questions from topics like sets, clocks, pipes and cisterns, are hardly been seen in recent years. This is not to discount the importance of these topics but one should always pay emphasis to topics which are in vogue.

► Focus on application: The formulae plays an important part in attacking a question, but more often than not,the questions that are asked is a combination of formulae, hence the focus should be on application of formulae. Also exceptions to the formulae should also be kept in mind while solving the questions. An example below shows this

1. The LCM of 2 numbers is 272, while their HCF is 8.If one of the numbers is 16, what is the other number?

The formulae is Product of 2 numbers = their LCM × HCF. Applying this we get the other number to be 136.So a straight-forward question.

2. The LCM of 2 numbers is 200, while their HCF is 2.If one of the numbers is 16, what is the other number?

Applying the same formulae, we get the other number to be 16.But the HCF of 25 and 16 is 1, but the question says the HCF to be 2. Hence this question is a wrong question. No such set of numbers exist.

3. The LCM of 2 numbers is 14, while their HCF is 5.If one of the numbers is 35, what is the other number?

This question is again a wrong question, since HCF is always a factor of LCM and here 5 is not a factor of 14.

► Judgmental ability: Always judge the question before attempting the section and try to find out the easy ones on the basis of your skills, ability and strengths. Some problems which may be easy for others may be hard for you and vice versa. Also have a basic idea as to what the question is actually trying to ask. The question might contain information and combination of options, which will make it a little confusing to mark the correct option. An example below shows this

The rate of inflation per year  is 1000%. What will be the cost of an article 2yrs from now, which costs 10 units now? A) 1000  b) 1100 c)1210 d) 100

The inflation is 1000%, which means the price rises by 10 times and hence becomes 11times. Hence the price will be 10×11×11=1210. To confuse you, the question gives options which are 10 times and 100 times of 10.

► Practice and Practice: The way you practice will gives you an edge over others. Given below are some practice tips while going through a quantitative aptitude text book.

1.While  going through a particular topic in a text book, Clear the basics part first by going through the fundamentals of that topic..Fundamentals include the formulae and application of that as well.

2. Practice the examples first. Go through the first 2-3 examples by going through the solutions understanding each step thoroughly. Read carefully and try to understand what it is really being asked in each question.

3. Try doing the next 2-3 examples yourself. If you are not successful, look at the solution .Try to analyse as to why you are making the mistake. Either you are making a calculation error or there is a problem with the funda itself. If you are making a silly calculation mistake, promise yourself to be extra careful. If there is problem in understanding of the fundamentals itself, then go through topic once again to clear the chinks .

4.  Try doing 2-3 very difficult problems on that topic. Give youself a particular time in which to solve them. Scale it up to 10-12 difficulty level problems and then see your accuracy as well as speed on that topic. Practice  as much as possible on that topic.

5. After you have done sufficient number of topics in a particular subarea (Arithmetic, P&C, 3D Mensuration etc), try doing a test on that. To avoid being taken by surprise, it is important to do a number of tests on different patterns, within the given time frame. Analysis of each test is a must so to know the scope of improvement in that particular area. The analysis must have these parameters

a. Which questions to select?

b. How much time to invest in a particular question?

c. Where am I making a mistake? Follow step 3 as given above, accordingly.

At each step, needless to say time plays a big role in the preparation, so be focused on time. Maintain a time discipline, but don’t be rigid right at the beginning of your preparation.

► Test taking strategies:  As you know that in spite of a good theory background and n-number of mock tests, the examinee still fails to devise a proper strategy of attempting a test, mainly because of the fear of the unknown. It is like a cricket captain deciding to field in a final match after winning the toss because past records suggest that(may be a dew factor), but absence of a strategy if the ground conditions change on that day, will ensure that his previous efforts comes to nothing.

As already stated previously, CSAT tests 3 skills

1. Time management   2. Decision making   3. Pressure handling capability.

Having known that you are going devote 25 mins to this section of 8 questions (let’s assume), devise a process in which to make the best use of that 25 mins. By the time you get to sit in CSAT, you will have this knowledge about the general feel of the section (Mocks play a big role in this endeavor). If most of the questions you scan are confusing to solve, try to identify the easy questions (sitters), which you can attempt straightway. If you go through any paper of past years, you will find that at least 1/3rd of questions can be attempted if one has a reasonable knowledge of the fundas. There might be some questions which seem pretty straight forward when you started but soon you understand that there are twists to it. The percentages of those questions are very less. Leave the questions which are taking more than 2-3 mins to solve.

While solving the question, put down all the given information in a neat and structured manner as possible, after which you will be in a position to decide whether you can proceed further or whether you should leave that question and move on. Also, try to check for alternative approaches like elimination/substitution of choices, which often save a lot of time and pay rich dividends. These can be mastered thorough practice.

Given below are 3 problems which will give us an insight about how to choose a question.

1. The probability of Suresh solving a problem is 2/5  , Ramesh solving it is  1/3 and Aakash solving it is  . If all three try to solve the problem, calculate the probability of problem being solved.

A] 31/35  B] 29/35  C] 8/105 D] 1

2. There are eight dice each having 6 faces, which are numbered with the first six odd natural numbers. These dice are thrown simultaneously; find the number of ways in which the sum of the numbers on the faces of the dice equals 43.

A] 10C6 B] 5040 C] 106 D] None of these

3. In how many ways can 30 identical balls be divided into 3 groups?

A] 146 B] 75 C] 76 D] 91

1. This problem says that the problem has to be solved by at least 1 person. So we will not multiply the probabilities. There is one option, which has exactly this option pertaining to

 

2.This question looks like there can be number of cases but you have to know that the sum of 6 odd numbers cannot be odd. So there is no such case which has the sum to be 43. So this is basically a number system question being rephrased into a P&C Question. The correct option is D.HENCE THIS IS AN EASY QUESTION.

3. This problem at the outset looks to be an easy question, but go within there can be a number of cases which are enumerated below.

The number of balls in all the groups can be same in only one case: (10, 10, 10). The number of balls in exactly two groups can be same in these cases: (1, 1, 28), (2, 2, 26), …, (14, 14, 2). These are 14 cases. The number of balls in the three groups can be different in these cases:

First group has 1 ball: (1, 2, 27), (1, 3, 26), …,.(1, 14, 15) (These are 13 cases)

First group has 2 balls: (2, 3, 25), (2, 4, 24), …, (2, 13, 15) (These are 11 cases) (we cannot take (2, 14, 14) which has been counted earlier)

First group has 3 balls: (3, 4, 23), (3, 5, 22), …, (3, 13, 14) (These are 10 cases)

First group has 4 balls: (4, 5, 21), (4, 6, 20), …, (4, 12, 14) (These are 8 cases)

And so on.

So, total number of cases = 1 + 14 + (13 + 11 + 10 + 8 + 7 + 5 + 4 + 2 + 1) = 76.

THIS LOOKS TO BE A STRAIGHT-FORWARD QUESTION BUT TURNS OUT TO BE A DIFFICULT QUESTION.

Hope I cleared some of the doubts regarding the GMA section for CSAT.

Practice books which will help you in getting a grasp of topics are:

1. Quicker Maths By M. Tyra

2. Quantitative Aptitude by Abhijit Guha

 

Manoj K. Jha

Human Geography Syllabus

  • by Manoj K. Jha

IAS aspirants are always in dilemma regarding the selection of topics and detailing of micro topics of related subjects

By Manoj K. Jha

IAS aspirants are always in a dilemma regarding the selection of topics and detailing of micro topics of related subjects. It has been a challenge especially for the IAS Prelims paper, as range of information matters for prelims exam. Keeping in mind the concerns, we are presenting the list of important topics for prelims 2014. We will cover all the subjects one-by-one. Here we are presenting topics for human geography ….

Human Geography topics for PT 14

Population
• General Demographic Scene
• Comparison of India’s population with world countries
• Characteristic of India’s demographic profile and related issues
• Population Growth – different periods & characteristics.
1. Period of Stagnant population
2. Period of Steady Growth
3. Period of Rapid High Growth
4. Period of High Growth with Definite Signs of slowing down.

The Demographic Transition – and related issues, challenges etc

National Population Policy & issues of population stabilization

Its long term, short term and medium term objective

States/UTs in descending or ascending order of Decadal Growth Rate

Rural – urban differences in population growth etc

Distribution and Density of Population : Major factors that affect it:-

Spatial Distribution of Population

States/UTs in descending/ascending order of population

Density

Comparison with other world countries

State level patterns

Spatial distribution Areas –

Population composition
• It comprises physical, socio-cultural and economic attributes of population like ethnicity, tribes, languages, religion etc.

Ethnic composition
• Common racial groups
• The Negritos
• Proto Austral rids
• The Mongoloids
• The Mediterranean
• The Brachycephals
• The Nordics

Tribal population : Types- distribution

Tribal economy
• Hunting, Fishing and Gathering
• Shifting Cultivation and lumbering –different names
• Sedentary cultivation, and Animal Husbandry
• Contemporary issues and developments

Language and Dialect Groups
• Classification of Indian languages
• Indo European Family
• Dravidian Family
• Austric Family
• Sino Tibetan Family

Linguistic Regions

Religious Composition

Population and proportion by Religious communities

Sex composition
• Sex Ratio in India during different census
• UTs/state level patterns in India
• Contemporary issues related to sex ratio

Migration Pattern
• Components of migration
• Definition by census

Causes of Migration
• Marriage, Employment, Education, Lack of Security Push and Pull factors

Consequence of Migration
(Demographic, socio-economic consequence and recent examples)

Rural Settlements
• Classification of villages
• Settlement types
• Rural Settlement Pattern
• Common issues related to rural settlement

Trends in Urban Area
• Definition by census
• Urbanization in India
• Decennial growth of urbanization
• Compared with other countries of the world
• Spatial pattern of urbanization

Size-class composition of Urban Population
• Town, city, metros, mega cities
• Metropolitan or million plus cities in India
• Important terminology related to urban growth

Problems of urbanization in India
• Urban sprawl, Overcrowding, Housing, unemployment, slums and squatter settlements etc.
• Slums distribution, issues and policies

Human development
• UNDP’s HD Report
• Indicators of Human Development
• HDI of India comparison with other countries

Health Indicators
• Measured in terms of BR, DR, Nutrition life-expectancy
• IMR, MMR
• Life Expectancy

Literacy
• Census Definition
• Regional variation in Literacy Rate
• Literacy during different decades
• Literacy Programme in India

Economic geography
• Water Resources
• Inter linkage of Rivers
• Perennial and non perennial rivers
• Link canal scheme
• Interstate River Water Disputes in India
• International Cooperation
• Contemporary development with reference to water issues i.e. Indus water Treaty, India-Bangladesh Co-operation, Indo-China etc.

Irrigation
• Geographical factors favouring irrigation
• Sources of irrigation
• Depending on topography, soils, rainfall, availability of surface or ground water, nature of rivers, requirement of crops etc.
• Tank irrigation, wells and Tube wells, Canals – their distribution in India with highest percentage
• Issues related to irrigation and initiatives

Multipurpose Projects
• Dams on different rivers
• Purposes and problems

Livestock and animal husbandry
• Distribution
• Livestock Products
• Milk – Operation flood etc
• Issues related to animal husbandry

Sericulture
• Production rank of India with other countries

Fisheries
• Fish production in India – ranking in world
• Marine Fisheries and Freshwater Fisheries – their Distribution in India – largest producing states

Land utilization
• Depends on physical factors and human factors
• Different types of uses of land
• Net sown areas, Forest, Pasture, etc.

Agriculture
• Dependency of India population on agriculture and contribution of agriculture to GDP.
• Problems of Indian Agriculture
• Small and fragmented landholding, seeds, manures, Fertilizers, and Biocides etc.

Agriculture Regions
• Rice- Jute – Tea Region, wheat and sugarcane region etc.

Agricultural revolutions
Intercrop imbalances, regional disparities etc

Major Indian crops
• Food crops
• Cash corps
• Plantation crops
• Horticulture
• Different conditions required for growth of these crops
• Its distribution in different states, highest producer

Mineral Resources
• Different mineral belt of India
• North Eastern Peninsular belt
• Central Belt
• Southern Belt
• Northwestern Belt

Types of Minerals –
1) Metallic – iron –ore, Manganese, cooper, etc
2) Non Metallic – limestone, potash etc

Iron ore
• Types
• Distribution and Production, largest producing area in India.
• India’s status in world countries
• Likewise for different types of minerals in India

Energy Resources
• Conventional sources of energy
• Coal – varieties – distribution in India
• And status in world countries

Petroleum or Mineral Oil
• List oil producing area, distribution
• Pipeline
• Distribution, import etc

Hydro electricity
• Important hydroelectric plants in Different states of India
• Recent development related to hydro electricity

Thermal – NTPC
• Nuclear Energy
• Programme in India
• Producing area.

Non- Conventional Energy Sources
• Solar
• Wind – first wind farm
• Biogas
• Small hydropower
• Geothermal energy, Tidal Energy, wave energy – their largest producer and distribution in India

Manufacturing Industries
• Industrial development in the Planning Era –
• Rise of modern industry
• Factors that influences the location of industries
• Geographical factors – Raw materials, power, labours transport, water, market etc.
• Non geographical factors – Capital, Govt., policies etc.
• Classification of Industries based on different criteria – (i) labour (ii) Raw Material (iii) ownership
• Miscellaneous industries – village, cottage, Ancillary Industries, Basic Industries etc.

Industries – Growth and Development, their largest producer and distribution in India, their problems.

Silk Industry – India enjoys the distinction of being the only country producing all the

five known commercial varieties of silk.

Iron and steel… Etc.

Fertilizers – Growth and Distribution

Agro based, forest based industry.

Major Industrial Region in India
• Mumbai – Pune,

• Hugli,

• Gujarat Industrial Region

Transport
• Railways & Roadways
• Growth and Development
• Different plans associated with it
• Nagpur plan
• BoT
• Classification of Roads.
• National Highways – largest, smallest.
• (NHDP) National Highways Development Project.
• NHDP – consist of different components.
• Golden Quadrilateral, North South Corridor
• State Highways – state wise length of Roads
• Border Roads
• Geographical Distribution of Roads

Air Transport

Water transport and Ports
• Water Transport
• National water ways
• Shipping
• Coastal shipping
• Ports – major and minor

IAS Score

Important Topics of General Science and Science & Tech.

  • by IAS Score

It has always been a dilemma for the aspirants, while charting the syllabus for General Science. It is more so while detailing the areas of recent development, the micro topics related to this area, for prelim, has been pretty vast.

By Manoj K. Jha

It has always been a dilemma for the aspirants of civil services examination, while charting the syllabus for General Science. It is more so while detailing the areas of recent development, the micro topics related to this area, for prelim, has been pretty vast (especially since the range of information matters for prelim). Keeping in mind the concerns, we are presenting the list of important topics for prelims 2014……

S & T Special Issue

• Institutional Structure for S & T in India
a) Dept of S & T
b) SEED
c) NCSTC
d) NSTEDB
e) CSIR
f) SERC
g) Technology dev Board
h) National Accreditation Board

• Schemes for dev of S & T
a) Different Fellowships Programmes
b) INSPIRE
c) FAST
d) Kishore Vaigyanik Protsahan
e) International Travel Support Scheme

• S & T policy 2003
• S & T policy 2013

• Nanotechnology
a) What is Nanotechnology with brief introduction of  “ how it works”
b) Applications
c) Nano Mission of India
d) Different capacity building projects of India
e) Examples of some research outputs
f) Nano knowledge city
g) Initiatives by different companies
h) Drawbacks

• Telecommunications
a) Basic concepts as:
i. What is analog and digital?
ii. What are the different multiplexing techniques?
iii. Optical fibre system
iv. Networking devices as Modem, ISDN, PSTN, etc.
v. What is spectrum?
vi. How mobile phone works?
b) GSM and CDMA
c) Different Generations of Mobile
d) Types of mobile batteries
e) Wi-Fi; Wimax; Bluetooth; WiBro; WAP; GPRS
f) What are Broadband and its uses?
g) Satellite Phone
h) iPhone and iPad
i) GPS navigation
j) Radiation emission guidelines set for Mobile Handsets, Mobile Base Stations
k) Indian Telecom Industry with brief history; National Telecom Policy 1994, 1999, 2011
l) TRAI
m) Mobile Number Portability
n) TCCCPR Bill 2010
o) What is USOF?
p) Spectrum Licencing
q) Telecomm applications for the socio-economic dev of the masses

• Electronics
a) Brief introduction of Semiconductors
b) How semiconductor works
c) What are photovoltaic devices
d) LED and OLED
e) Plasma
f) LCD, CFL, CRT
g) HDTV
h) Triple play
i) Lasers
j) 3-D and Animation
k) Superconductors
l) Government initiatives for dev of electronic industry in India as Electronics Hardware Technology Park (EHTP) Scheme, Export Promotion Capital Goods (EPCG) Scheme, National Skill Dev, etc.
m) R & D in Electronics
n) Electronics Policy 2011

• IT
a) Computers and their Generations
b) Supercomputers
c) Networking and types
d) Internet
e) Basic terms used in internet as www; http; HTML; URL; IP address etc
f) IPV6
g) IP Telephony
h) ocial networking sites and its impact
i) Computer crimes
j) Cyber laws
k) ypes of computer viruses
l) Recent controversy on online content
m) Digital signature
n) UIDAI project: use of IT in fingerprinting, iris scanning etc
o) ATM
p) AI
q) Digital Convergence
r) BPO: advent of it in India
s) E-governance: different initiatives by government
t) National Policy on IT
u) Cloud computing
v) Internet banking
w) E-learning
x) National Knowledge Network
y) AGRINET
z) NIC
aa) Robotics

• Biotechnology
a) What are genes?
b) What is biotechnology?
c) ts branches
d) Bio fingerprinting
e) Bio sensors
f) Genetic engineering
g) Recombinant technology
h) Gene mutation
i) Human genome project
j) Gene sequence of arhar, neem etc
k) Bio piracy
l) Bio-grid
m) Bt cotton
n) Bt Brinjal and controversy
o) Cloning
p) Stem cells
q) Applications in agriculture, industry, food, health etc.
r) Institutional structure for dev of biotechnology in India
s) Initiatives by government for the dev of biotechnology
t) Mission mode programmes
u) Bioinformatics
v) International collaborations
w) Latest achievements
x) XIth Five Year Plan

• Nuclear Technology
a) Basic concepts as:
i. What is radioactivity?
ii. Applications of radioactivity
iii. Nuclear fission
iv. Nuclear fusion
v. What is coolant, modulator, fuel used etc
vi. How nuclear power generates
vii. ifferent types of nuclear reactors as fast breeder, boiling water reactor etc
viii. Civilian and military use of nuclear technology
b) verview of different countries involved in nuclear technology with intro of ITER.
c) Disadvantages with focus on nuclear accidents and long term effect of radiation on masses
d) Institutional structure for dev of nuclear technology in India
e) India nuclear policy
f) Nuclear programme
g) Recent controversy of establishing nuclear reactors
h) Safety parameters issued by government
i) Uranium mines in India
j) Technologies and applications for societal dev in India

• Defence
a) Basic concepts as what is missile and how it works?
b) Stealth technology with intro of RADAR
c) Difference between ballistic and cruise missile
d) What is Mach speed
e) Division of types of missiles in relation with India missile programme
f) Anti ballistic missiles and how it works
g) BRAHMOS
h) Different technological dev in  weapons and aircrafts for Air force
i) Brief intro of different naval ships with their use
j) Institutional structure for R & D in defence
k) Role of DRDO for overall dev of defence system other than weapons as frozen foods, clothes etc
l) Chemical and biological weapons
m) Dirty bombs and convention to ban it
n) New technological dev to trace bombs
o) What is IED
p) Defense Technology Road Map of India

• Space
a) Brief history of space technology in India
b) How a satellite works
c) Different parts of satellites in brief
d) ISRO
e) Different Centres
f) Launch vehicle types
g) Propulsion systems
h) Cryogenic rockets
i) Reusable technology
j) Different satellite orbits as GTO, LEO,MEO, GEO
k) International space station
l) New ISS launched by China
m) VSAT
n) Telemedicine
o) INSAT and its applications
p) Examples of some latest INSAT satellites
q) Remote Sensing Satellites and its applications
r) Examples of some latest Remote sensing Satellites
s) New launches
t) Future programmes of India
u) Tele education
v) DTH
w) International collaborations
x) Space as a source of mass social dev
y) Environmental effects of satellites
z) Some space programmes of other countries as JUNO, KEPLAR, CHANGE 2 etc

• Energy
a) Overview of fossil fuels as coal, crude oil
b) New technologies to decrease its pollution content
c) Thermal power plants, how it works and disadvantages
d) Hydroelectric power plants, working and drawbacks
e) Carbon sequestration
f) Carbon footprint concept
g) New policies of government to make fossil energy resources more greener
h) Renewable energy resources and its advantages over non renewable
i) Solar energy
j) Wind energy
k) Fuel cells
l) Geothermal energy
m) Tidal and wave energy
n) Biofuel, biogas, etc
o) Hydrogen fuel and its uses
p) Some new inventions

• Biology
a) Cell theory
b) Skeletal and muscular system
c) Circulatory system
d) Nervous system
e) Respiratory system
f) Sensory organs
g) Reproductive system
h) Digestive system
i) Endocrine and exocrine glands
j) Diseases and vaccines
k) Diet
l) Some facts and figures

Manoj K. Jha

CSAT is qualifying now ! What Next ?

  • by Manoj K. Jha

Atlast, the government has cleared the air on the controversy of CSAT exam… it has approved the decision for making the General Studies Paper-II(CSAT) in the Civil Services(Preliminary) Examination a qualifying paper with a minimum qualifying marks fixed at 33%…. further the English Language comprehension skill portion will continue to remain excluded.

Atlast, the government has cleared the air on the controversy of CSAT…it has approved the decision for making the General Studies Paper-II(CSAT) in the Civil Services(Preliminary) Examination a qualifying paper with a minimum qualifying marks fixed at 33%…. further the English Language comprehension skill portion will continue to remain excluded.

Thus, as the weightage of General Studies Paper-II (CSAT) has decreased, the weightage of General Studies Paper-I has increased tremendously. Now this paper will decide whether aspirants are able to come one step closer to goal of becoming a Civil Servant or not.

And as we know, the pattern of questions of General Studies has changed from the past few years; it required a multi-dimensional strategy to cover the topics mentioned in the syllabus. But after changes an aspirant can’t leave any topic of the syllabus because there will be no CSAT for covering up it. He has to be thorough in basic concepts along with current issues to manage the cutthroat competition.

CSAT is qualifying now. What next? How to tackle GS now? GS for prelims have always been a nightmare both for the freshers as well as experienced campaigners. This is primarily because of the vastness of the syllabus wherein almost everything under the sun is covered under it and is fit to be studied and known. Add to it the objective nature of the examination, which makes it important to go through even the minutest details of everything in the syllabus and to be able to recollect and reproduce it accurately in the examination.

This view, though very popular but do not however provide the best picture of the actual nature of the exam. GS for prelims is not actually a synonym of arbitrariness and randomness. A thorough analysis of the paper shall reveal that it is much simple and somewhat predictable. Further recent changes incorporated by the UPSC on the prelims exam has not only made the GS paper much simpler but has drastically reduced the overall effort which was required for clearing the exam provided the candidate follows the right approach and puts in sufficient effort.

However on the other side if a candidate is lackadaisical in putting the necessary effort and fails to appreciate the direction in which the changes are pointing to the task has become even more herculean.

However to elaborate the point further we need to first examine as to what have been the nature of the changes which have become a part of the GS paper during the last three years. Though some people may differ from this view but that is mainly because barring a few of them, most of these changes have been very slowly manifesting themselves and contributing to the evolution of the exam.

More and more questions now are framed around concepts rather than events or fixtures and a candidate is expected to work more on the following lines which is by and large a four step process.

Firstly, it is expected to master all the important concepts in a holistic manner across all the disciplines be it history, polity, economy, geography, science and tech etc.

Secondly, it needs to understand the   theories/principles/fundamental around which the concepts are woven to offer explanations.

Thirdly, it needs to incorporate interdisciplinary concepts to explore more theories and explanations and thus further enriching understanding.

Finally, it needs to apply these theories and concepts to real situations and discover their validity or falsity for a given set of values.

This four-step process though sounds very stupendous but is in-fact foundation to all types of learning. Further following this four step process not only ensures that a candidate clears his GS paper in the prelims but also is very much on track as far as preparations for mains are concerned.

In consonance with this four-step process, there are several noticeable trends, which are visible in the prelims GS paper. Foremost, we find that unlike in the past, the nature of questions asked in the GS paper are more multifaceted that is within the same question, the UPSC tends to identify whether the candidate has a grasp over a concept and whether he is also in a position to apply that concept and give out answers to questions involving theoretical linkages with other concepts. So much so that even some questions from history envisage a candidate to use his knowledge of the social, economic and cultural variables of the time, to answer the questions rather than information like the genealogy of the kings and important dates, that is to indicate a definitive shift from the fact based pattern of asking questions.

Furthermore the increased emphasis on concepts and understanding can be seen in some questions like that on Karl Marx and dialectical materialism (CSAT 2011), which otherwise is a core concept of sociology and political science, and not considered a part of GS, but is of ultimate importance to the understanding of various political and economic concepts.

An extension of the same idea in the disciplines of polity and economics is in the form of asking questions in which the application of the concepts and the theoretical perspectives to real practical situations is required thereby testing for the candidates’ grasp in scenarios where such application of theoretical paradigms is not absolute. In other words not only it is important to know the ‘what’ but also the ‘how’ and the ‘why’ aspects in a critical perspective, taking the entire preparation to the advanced level, closer to that of the mains exam.

This also means that studying NCERT is necessary but not sufficient as the candidate has to cultivate the skill of understanding concepts beyond the conventional perspective, which is something he cannot acquire simply by gazing over NCERT which is more knowledge intensive rather than application intensive.

In that sense only candidates who have managed to take their preparation to the next level run a chance of clearing the Prelims. All this however has proved to be a blessing in disguise for the smart candidate who knows his lessons well as very much of the questions can now be answered merely by applying common sense and also one becomes more comfortable in hitting at the right answers to those questions which offer more than one correct answer or present very close alternatives or involve application of some level of value based subjectivity in coming to the right answers.

Now the question is as to how to maximize the probability of answering the most number of questions in the paper. Foremost is adhering to the wide extensive approach to which there is no substitute. A candidate must be thorough with the various important concepts across all disciplines in conjugation with popular theoretical paradigms.

Once a wide extensive coverage is achieved, it should be followed by a selective intensive approach in which the keyword is ‘current affairs’. Now the word current affairs itself is so broad that some candidates find the application of the selective intensive approach almost impossible. But an avid observer will notice that there is definite definition of ‘relevant’ current affairs and irrelevant current affairs, which we can very well deduce from the previous year question papers.

Having said all this it is equally important to mention in the end, that a candidate must always try to remain original and honest during the course of preparation. All we can offer are just vital clues to make your approach focused and streamlined but the best approach for you is the one you think is the best for you.

I had already written about this change on 22 Jan for Jagranjosh.

http://www.jagranjosh.com/articles/csat-2015-will-upsc-give-another-pleasant-shock-1421921328-1

Prelims Preparatory series Part -1

Manoj K. Jha

Important Topics of Environment and Ecology

  • by Manoj K. Jha

The topic of ‘Ecology, Bio-diversity & Climate-Change’ in Prelim coupled with the topic of ‘Conservation, environmental pollution and degradation, environmental impact assessment’ in Main, creates a comprehensive structure of study.

By Manoj K. Jha

The topic of ‘Ecology, Bio-diversity & Climate-Change’ introduced in Prelim 2011 (ie. With the change of pattern) coupled with the topic of ‘Conservation, environmental pollution and degradation, environmental impact assessment’ in Main (GS Paper III) creates a comprehensive structure of study. The non-conventional nature of the subject along with the fast developments taking place in the field has made it extremely difficult for a student to study it in totality. Given the situation, an attempt has been made to make the task of a student a little bit comfortable by enlisting the topics to be covered in a relatively detailed format. In this exhaustive list topic of contemporary relevance also has been included .

To study ecology and environment one can divide the whole syllabus into five categories ie.

· Fundamentals of Ecology
· Environmental problems
· Contemporary  environmental issues
· Integrated environmental management
· Global environmental governance and India’s response

Below mentioned are the suggested topics for ecology and environment section —-

ENVIRONMENT

1. Ecosystem: ….Structure of An Ecosystem …Function of an Ecosystem….Ecological Pyramids …..Ecological Efficiencies…Biological Magnification..etc

2. Biodiversity: …..Need for Preservation of Bio-diversity….Threats to Bio-diversity …Effects of Loss of Bio-diversity…Bio-diversity Conservation…Bio-diversity in India… Mangrove….National Bio-diversity Strategy and Action Plan

3. Natural Resources: ….Conservation of Forests …..Conservation of Soil….Conservation of Water…..Conservation of Wild Life …..Conservation of Energy Resources…and issues related to it…

4. Energy & Environment: …Fossil Fuels and Environment…Fuel Energy and Need for Sensible Use…..Renewable Energy …Bio-Mass Energy..Solar Energy for Rural India ….Gobar Energy in India….Biogas

5. Environmental Crisis: ….Forms of Environmental Crisis ….Causes of Environmental Crisis …..Environmental Conservation ….Government for the Management

6. Environmental Hazards: …Cyclonic Storms ….Floods …..Landslide …Landslide Zones….Drought  etc

7. Climatic Change:  …The Greenhouse Effect …. Consequences of Climate Change…World Response …UN-Convention on Climate Change

8. Global warming: …Greenhouse Gases….India Enters in Carbon trading…….Weakening North Atlantic Current …Upper Atmosphere thinning …..Ozone Hole Galloping Antarctica Again …Southern Hemisphere Affected More …..Sea Level Rise….environmental refugees, global initiatives..etc

9. Forest:  Forestry & Man….Social Forestry ….Agro-Forestry ….Silviculture

10. Forests & Environment:…..Indian Scenario….Need for Conservation of Forest..Scope of Forest Management…integrated forest management

11. Pollution:….Air Pollution ….Water Pollution …..Oil Pollution at sea …..Radioactive Pollution …..Role of Government in Combating Pollution..Recent developments..etc

12. Land Degradation….Desertification …..Modern Agriculture ….Soil Problem in India ….Wetlands Management…watershed development

13. Water Resources…Configuration of Water Use…..Water Quality Deterioration….Human Issues and Water Resources ….Water Management and Conservation..Water privatization and issues related..

14. Global environmental governance …important international environmental initiatives as..Earth summits, RIO, etc  …and issues related to it

OTHER TOPICS

1. Antarctic Treaty Consultative Meeting
2. Arctic Hurricanes
3. Bharat Stage -IV
4. B K Chaturvedi Committee on Ecological Clearance
5. Black Carbon
6. Carbon Credit
7. Bharti Research Station
8. Biofertilizer
9. Bioremediation
10. Carbon Footprint and Indian Industries
11. Carbon Sequestration
12. Carbon Sink
13. Carbon Tax
14. Cartagena Protocol
15. Census of Marine Life
16. Central Water Commission
17. CFL (Compact Fluorescent Lamps)
18. Clean Development Mechanism
19. Coastal Zone Management
20. Comprehensive Environment Pollution Index
21. Convention On International Trade In Endangered Species Of Wild Fauna And Flora (Cites)
22. Desert Geopark
23. E-8 (Elephant Conservation)
24. Ecologically Sensitive Areas
25. Ecomark
26. Emission Trading
27. Energy Conservation Building Code
28. Energy Saving Certificates
29. Environmental Performance Index
30. Environmental Refugee
31. E.U. Emission Cap (Airlines)
32. Genetic Garden
33. Genetic Pollution
34. Geo-Engineering
36. Geographical Information System
37. Global Carbon Project
38. Global Tiger Forum
39. Green Building
40. Green India Mission
41. Green National Accounts Initiative
42. Green Rating for Integrated Habitat Assessment
43. Haathi Mere Saathi
44. India State of Forest Report 2011
45. Integrated Coastal Zone Management
46. Integrated Coastal Zone Management Project
47. Integrated Watershed Management Programme
48. IUCN Red list
49. Kiribati Islands (Climate Change Impact)
50. Kyoto Protocol
51. Multilateral Environmental Agreements
52. Nagoya Protocol
53. National Action Plan On Climate Change
54. National Centre for Marine Bio-Diversity
55. National Clean Energy Fund
56. National Cyclone Risk Mitigation Project
57. National Disaster Communication Network
58. National Disaster Management Information System
59. National Environment Appellate Authority
60. National Environment Assessment and Monitoring Authority
61. National Environment Policy
62. National Geophysical Research Institute
63. National Mission for Sustaining the Himalayan Ecosystem
64. National Green Tribunal (NGT) or Green Courts
65. National Initiative on Climate Resilient Agriculture
66. National Tiger Conservation Authority (NTCA)
67. Nongovernmental International Panel on Climate Change (NIPCC)
68. Network Project on Organic Farming
69. Ocean Thermal Energy Conversion (OTEC)
70. Oil spill
71. Oil zapper
72. Ramasar Convention
73. Redd
74. Redd+
75. Renewable Energy Certificate (REC)
76. Renewable Purchase Obligation (RPO)
77. Save (Saving Asia’s Vultures from Extinction)
78. Un Convention on Biological Diversity (Uncbd)
79. Un International Year of Forests
80. Western Ghats Ecology Expert Panel (Wgeep)
81. Wwf (World Wildlife Fund)
82. Yeti (Young Ecologists Talk and Interact)
83. Zero Carbon Britain
84. All about Ecologically Sensitive Area
85. Water Degradation in Urban Areas
86. E-Waste Management and Handling
87. Disposal of Untreated Sewage
88. Conservation Of Tiger, Elephant, Snow Leopard, Etc
89. Environmental Aspects Associated With Mining
90. Kasturirangan Report
91. Corporate Social Responsibility And Environment
92. Forest Clearance Issue
93. IUCN updated List
94. Pro-Poor Green Development Initiatives
95. Climate Change and International Response
96. Mining of Sand on River Beds
97. Pollution in Metros
98. Encroachment on National Parks
99. Contamination of Water
100. Commercialization of Agriculture
101. Real Estate Boom and Environment Degradation
102. Haphazard Construction In Hilly Areas And Impact
103. Integrated River Basin Management
104. Trials of Gm Food and Impact
105. Environment Clearance and Development Issue
106. Community Forest Management
107. Environmental Impact Assessment and Environmental Auditing
108. Polythene Bags and Pollution
109. Groundwater Depletion
110. Impact Of Tower Radiation
111. Greenex: India’s First Carbon-Efficient Index

Manoj K. Jha

Important Physical Geography Topics For Ias 2014

  • by Manoj K. Jha

IAS aspirants are always in a dilemma regarding the selection of topics and detailing of micro topics of related subjects. Here we are presenting topics for Physical Geography (World & India) ….

By Manoj K. Jha

IAS aspirants are always in a dilemma regarding the selection of topics and detailing of micro topics of related subjects. It has been a challenge especially for the IAS Prelims paper, as range of information matters for prelims exam. Keeping in mind the concerns, we are presenting the list of important topics for prelims 2014. We will cover all the subjects one-by-one. Here we are presenting topics for Physical Geography (World & India) ….

GEOMORPHOLOGY
• Basic Terms and Concepts (Weathering, Erosion, Mass Wasting, Strait, Isthmus, Gulf, Relief, Bay and Cape, Topography and its various types etc.)
• Earth(Shape, Size, Latitude, Longitude, Rotation, Revolution, Local Time, Standard Time, Earth Zone Rocks – Types of Rocks (Igneous, Sedimentary, Metamorphic) Interior Structure of Earth(Crust, Mantle, Core)
• Forces Which Affect The Earth Crust (Endogenetic, Exogenetic Forces, Mountain Building, Fold And Fault) Continental Drift Theory, Plate Tectonic Theory Etc Volcanism (Types and Distribution) Earthquake(Causes and Distribution) Eutrophication
• Drainage System and Drainage Pattern (Major Drainage Systems, Drainage Pattern, River Capture etc.)
• Biogeography (Genesis, Classification and Distribution of Soils, Soil Profile, Soil Erosion, Degradation and Conservation), (Factors Influencing World Distribution of Plants and Animals, Problems of Deforestation and Conservation Measures, Social Forestry, Agro-Forestry, Biomes and its various types, Major Gene Pool Centers)

CLIMATOLOGY
• Structure and Composition of Atmosphere (Composition of The Atmosphere Including Chemical Composition, Structure of The Atmosphere, Elements of Weather and There Control)
• Winds and Pressure Belt (Types of Wind, Permanent, Periodic and Local Wind) + (Horizontal Distribution of Air Pressure and Pressure Belt, Shifting of Pressure Belt, Pressure Gradient and Circulation, Wind Direction and Related Laws, Tricellular Meridional Circulation of The Atmosphere)
• Mode of Transportation of Heat (Mechanism of Solar Radiation, Distribution of Insolation, Heat Budget Etc.)
• Rainfall Type, Windward Side and Leeward Side (Convectional, Orographic & Cyclonic Rainfall, Global & Zonal Distribution of Rainfall & Its Regime,
• Humidity(Absolute, Specific and Relative), (Condensation & Adiabetic Change of Temperature)
• Clouds, Fog, Dew, Mist Haze and Smog
• Airmasses and Fronts
• Frontogenesis and Frontolysis
• Cyclone (Tropical and Temperate)

OCEANOGRAPHY
• Salinity and Its Distribution
• Tides
• Current (Atlantic, Pacific and Indian)
• Coral Reefs and Atoll

PHYSICAL GEOGRAPHY OF INDIA
• Physiographic Regions of India (Himalaya, Great Plains, Plateau, Coastal Plain and Islands)
• Climate of India (Monsoon and Its Mechanism, El-Nino & La-Nino, Southern Oscillation and Walker Cell, Rainfall and Distribution in India, Climatic Classification of Koppen, Thornthwaite & Trewartha)
• Soils of India (Types, Soil Erosion and Conservation Measure)
• Vegetation of India (Types of Vegetation, Geographycal Distribution, Problems of Forest, Importance of Forest, Forest Products, Mangrove Forest and Social Forestry, Conservation, Wildlife, Preservation of Wildlife)
• Rivers and Tributaries of India(Peninsular and Himalayan River)
• Flood ( Causes, Problems, Prone Ares, Remedy and Forecasting)
• Drought (Types & Causes, Prone Area, Management and Remedy)
• Wetlands (Ramsar Sites, Preservation and Management)

Manoj K. Jha

Sound your ‘Basics’

  • by Manoj K. Jha

Due to the unpredictability of the exam wherein you cannot afford to make selective study purely based on previous years question papers.

Continue reading “Sound your ‘Basics’”

Manoj K. Jha

IAS PT 2014- SPECIAL SERIES-1, GS for Prelims: Changing trends

  • by Manoj K. Jha

GS for prelims have always been a nightmare both for the freshers as well as experienced campaigners. This is primarily because of the vastness of the syllabus wherein almost everything under the sun is covered under it and is fit to be studied and known.

Continue reading “IAS PT 2014- SPECIAL SERIES-1, GS for Prelims: Changing trends”